You are on page 1of 90

Table of Contents

Current Electricity

 Theory ..................................................................................................................................... 2

 Solved Examples .................................................................................................................... 26

 Exercise - 1 : Basic Objective Questions ............................................................................... 43

 Exercise - 2 : Previous Year JEE Mains Questions ................................................................. 54

 Exercise - 3 : Advanced Objective Questions ....................................................................... 62

 Exercise - 4 : Previous Year JEE Advanced Questions .......................................................... 79

 Answer Key ............................................................................................................................. 89


CURRENT ELECTRICITY 2

Current Electricity
1. ELECTRIC CURRENT 3. DRIFT VELOCITY

“The flow of charge in a definite direction constitutes the    


“If u1 , u2 , u3 , ...un are random thermal velocities of n free
electric current and the time rate of flow of charge through
electrons in the metal conductor, then the average thermal
any cross-section of a conductor is the measure of current”.
velocity of electrons is
i.e.,
   
u1  u2  u3  ...  un 
net charge flown q dq 0
Electric current , I=   n
time taken t dt
As a result, there will be no net flow of electrons of charge
1. Though the “electric current represents the direction of flow
in one particular direction in a metal conductor, hence no
of positive charge”.
current”.
2. Yet it is treated as a scalar quantity.
“Drift velocity is defined as the average velocity with which
3. Current follows, the laws of scalar addition and not the laws
the free electrons get drifted towards the positive end of
of vector addition.
the conductor under the influence of an external electric
4. Because the angle between the wires carrying currents does
field applied”.
not affect the total current in the circuit.
–4 –1
1. The drift velocity of electons is of the order of 10 ms .
2. CURRENT CARRIERS 2. If V is the potential difference applied across the ends of the
conductor of length l, the magnitude of electric field set up is
(a) Current carriers in solid conductors :
1. In solid conductors like metals, the valence electrons of the Potential difference V
E 
atoms do not remain attached to individual atoms but are length 
free to move throughout the volume of the conductor.
2. Under the effect of an external electric field, the valence
E
electrons move in a definite direction causing electric current
in the conductors.
3. Thus, valence electrons are the current carriers in solid – +
conductors.
(b) Current carriers in liquids :
1. In an electrolyte like CuSO4, NaCl etc., there are positively 3. Each free electrons in the conductor experience a force,
 
and negatively charged ions (like Cu , SO , Na , Cl ).    
4 F  e E.
2. These are forced to move in definite directions under the 4. The acceleration of each electron is
effect of an external electric field, causing electric current. 
3. Thus, in liquids, the current carriers are positively and  eE
a .
negatively charged ions. m
(c) Current carriers in gases : 5. At any instant of time, the velocity acquired by electron

1. Ordinarily, the gases are insulators of electricity. having thermal velocity u1 will be
2. They can be ionized by applying a high potential difference   
v1  u1  a1
at low pressure
3. Thus, positive ions and electrons are the current carriers in where 1 is the time elapsed since it has suffered its last
gases. collision with ion/atom of the conductor.
CURRENT ELECTRICITY 3

6. Similarly, the velocities acquired by other electrons in the


 The drift velocity of electrons is small because of the frequent
conductor will be collisions suffered by electrons.
        
v 2  u 2  a2 , v3  u 3  a3 , ....., v n  u n  an .  The small value of drift velocity produces a large amount of
electric current, due to the presence of extremely large
7. The average velocity of all the free electrons in the conductor
number of free electrons in a conductor. The propagation of
under the effect of external electric field is the drift velocity
 current is almost at the speed of light and involves
vd of the free electrons. electromagnetic process. It is due to this reason that the
   electric bulb glows immediately when switch is on.
 v  v 2  ...  v n
Thus, vd  1  In the absence of electric field, the paths of electrons between
n
successive collisions are straight line while in presence of
      electric field the paths are generally curved.
 u1  a1    u 2  a2   ...  u n  an 

n NA x d
 Free electron density in a metal is given by n 
   A
 u1  u 2  ...  u n    1  2  ...  n   
 a  0  a  a  where N A = Avogrado number, x = number of free
 n  n
electrons per atom, d = density of metal and A = Atomic
weight of metal.
1  2  ...  n
where,   = average time that has elapsed 1. Mobility of charge carrier (), responsible for current is
n
defined as the magnitude of drift velocity of charge per unit
since each electron suffered its last collision with the ion/
electic filed applied, i.e.,
atom of conductor and is called average relaxation time.
–14
8. Its value is the order of 10 second. drift velocity v d q E  / m q 
   
 electric field E E m
9. Putting the value of a in the above relation, we have
 e e
  e E 2. Mobility of electron, e 
vd  me
m
3. The total current in the conducting material is the sum of
eE the currents due to positive current carriers and negative
Average drift speed, vd  
m current carriers.
 vd   e E
The negative sign show that vd is opposite to the direction
2 –1 –1 –1 –1
 4. SI unit of mobility is m S V or ms N C
of E .
3.3 Relation between current and Drift Velocity
3.1 Relaxation time ()

The time interval between two successive collisions of 1. Consider a conductor (say a copper wire) of length l and of
electrons with the positive ions in the metallic lattice is defined uniform area of cross-section
 Volume of the conductor = Al.
mean free path 
as relaxation time    . With 2. If n is the number density of electrons, i.e., the number of
r.m.s. velocity of electrons v rms
free electrons perunit volume of the conductor, then total
rise in temperature vrms increases consequently  decreases. number of free electrons in the conducture = Aln.

3.2 Mobility 3. Then total charge on all the free electrons in the conductor,

q  Ane
Drift velocity per unit electric field is called mobility of electron i.e.
4. The electric field set up across the conductor is given by
v m2
  d It’s unit is E = V/l (in magnitude)
E volt  sec
5. Due to this field, the free electrons present in the conductor
 If cross-section is constant, I  J i.e. for a given cross-sectional will begin to move with a drift velocity vd towards the left
area, greater the current density, larger will be current. hand side as shown in figure
CURRENT ELECTRICITY 4

V m
or 
I A n e 2  = R = a constant for a given conductor for a
given value of n, l and at a given temperature. It is known as
the electrical resistance of the conductor.
Thus, V = RI

6. Time taken by the free electrons to cross the conductors, this is Ohm’s law.
t = l/vd (1) Ohm’s law is not a universal law, the substances, which
q Ane obey ohm’s law are known as ohmic substance.
Hence, current, dI  
t t  (2) Graph between V and i for a metallic conductor is a straight
vd line as shown. At different temperatures V-i curves are
different.
or I  A n e vd
V V
 e E  T1
7. Putting the value of vd    , we have
 m  1 T2
2 2
Ane E
I
m 1
2

i i
4. OHM’S LAW
(A) Slope of the line (B) Here tan1 > tan2
Ohm’s law states that the current (I) flowing through a
V
conductor is directly proportional to the potential difference = tan   R So, R1 > R2 i.e., T1 > T2
i
(V) across the ends of the conductor”.
(3) The device or substances which don’t obey ohm’s law
I e.g. gases, crystal rectifiers, thermoionic valve, transistors
etc. are known as non-ohmic or non-linear conductors.
A For these V-i curve is not linear.

V 1
Static resistance R st  
i tan 
O V
V 1
i.e., I  V or V  I or V = RI Dynamic resistance R dyn  
I tan 

V
or  R  constant i Crystal
I rectifier

4.1 Deduction of Ohm’s law

eE
We know that vd   V
m

But E = V/l  vd 
eV
 5. ELECTRICAL RESISTANCE
m
“The electrical resistance of a conductor is the obstruction
Also, I = A n e vd
posed by the conductor to the flow of electric current
2
 eV   A n e   through it”.
 I =An e     V
 m   m  1. i.e., R = V/I
CURRENT ELECTRICITY 5

(vi) Resistivity increases with impurity and mechanical stress.


volt
2. The SI unit of electrical resistance is ohm or . (vii) Magnetic field increases the resistivity of all metals except
amp
iron, cobalt and nickel.
3. Dimensions of electric resistance (viii) Resistivity of certain substances like selenium, cadmium,
sulphides is inversely proportional to intensity of light
Pot. diff . work done/charge falling upon them.
 
current current
V m m 
3. We have, R   2
 2 
2
ML T / AT2 I Ane  ne  A
   M1L2 T 3 A 2 
A

comparing the above relation with the relation, R   .
5.1 Electrical, Resistivity or Specific Resistance A
4. We have, the resistivity of the material of a conductor,
“The resistance of a conductor depends upon the following
factors : m

(i) Length (l) : The resistance (R) of a conductor is directly ne2 
proportional to its length (l), i.e., R  l
(ii) Area of cross-section (A) : The resistance (R) of a 5.3 Conductivity
conductor is inversely proportional to the area of cross- 1
section (A). of the conductor, i.e., R  1/A Reciprocal of resistivity is called conductivity () i.e.   with

(iii) The resistance of conductor also depends upon the nature unit mho/m and dimensions [M 1 L3 T 3 A 2 ] .
of material and temperature of the conductor.
5.4 Conductance
 
From above ; R  or R  .” 1
A A Reciprocal of resistance is known as conductance. C  It’ss
R
5.2 Resistivity () 1
unit is or –1 or “Siemen”.

1 Where  is constant of proportionality and is known as
specific resistance or electrical resistivity of the material of
i
the conductor
2. Specific resistance (or electrical resistivity) of the material
of a conductor is defined as the resistance of a unit length
with unit areas of cross section of the material of the
conductor. V

(i) Unit and dimension : It’s S.I. unit is ohm × m and


5.5 Stretching of Wire
dimension is [ML3T–3A–2]
If a conducting wire stretches, it’s length increases, area of cross-
m
(ii) It’s formula :   section decreases so resistance increases but volume remain
ne 2  constant.
(iii) Resistivity is the intrinsic property of the substance. It is Suppose for a conducting wire before stretching it’s length = l1,
independent of shape and size of the body (i.e. l and A). area of cross–section = A1, radius = r1, diameter = d1, and
(iv) For different substances their resistivity is also different l1
e.g. silver = minimum = 1.6  10 –8  -m and resistance = R1  
A1
fused quartz = maximum  1016  -m
Before stretching After stretching

insulator  alloy  semi-conductor  conductor
(Maximum for fused quartz) (Minimum for silver ) l1 l2

(v) Resistivity depends on the temperature. For metals


 t   0 (1  t) i.e. resitivity increases with temperature.
CURRENT ELECTRICITY 6

Volume remains constant i.e., A1l1 = A2l2 3. Semiconductors : These are those material whose electrical
After stretching length = l2, area of cross-section = A2, conductivity lies inbetween that of insulators and conductors.
Semiconductors can conduct charges but not so easily as is
l in case of conductors. When a small potential difference is
radius = r2, diameter = d2 and resistance  R 2   2
A2 applied across the ends of a semiconductor, a weak current
Ratio of resistances before and after stretching flows through semiconductor due to motion of electrons and
holes.
2 2 4 4
R1 l1 A 2  l1   A 2   r2   d 2  Examples of semiconductors are germanium, silicon etc.
= × =  =  =  = 
R 2 l2 A1  l2   A1   r1   d1  The value of elecrical resistance R increases with rise of
temperature.
2
R1  l1 
(1) If length is given then R  l 2    Rt  R0 increase in resistance
R 2  l2   
R0  t original resistance × rise of temp.

4
1 R r  Thus, temperature coefficient of resistance is defined as the
(2) If radius is given then R  4  1   2  increase in resistance per unit original resistance per degree
r R 2  r1 
celsium or kelvin rise of temperature.

6. CURRENT DENSITY, CONDUCTANCE 1. For metals like silver, copper, etc., the value of a is positive,
therefore, resistance of a metal increases with rise in
AND ELECTRIAL CONDUCTIVITY –1 –1
temperature. The unit of  is K or °C .

6.1 Relation between J,  and E 2. For insulators and semiconductors  is negative,


therefore, the resistance decreases with rise in temperature.
2
 eE  n Ae E 6.2 Non-Ohmic Devices
We know, I = n Aevd = nAe    
m  m
Those devices which do not obey Ohm’s law are called non-
I ne 2 E 1 ohmic devices. For example, vaccum tubes, semiconductor
or  or J  E
A m  diode, liquid electrolyte, transistor etc.
For all non-ohmic devices (where there will be failure of
 J  E
Ohm’s law), V–I graph has one or more of the following
1. Insulators : These are those materials whose electrical characteristics :
conducticity is either very very small or nil. (a) The relation between V and I is non-linear, figure
Insulators do not conduct charges. When a small potential
difference is applied across the two ends of an insulator, the I
current through the insulator is zero.
Examples of insulators are glass, rubber, wood etc.
Variation of R,  with T
2. Conductors : These are those materials whose electrical
conductivity is very high
V
Conductor conduct charges very easily. When a small
potential difference is applied across the two ends of
(b) The relation between V and I depends on the sign of V. It
conductor, a strong current flows through the conductor.
means, if I is the current for a certain value of V, then
For super-conductor, the value of electrical conductivity is
reversing the direction of V, keeping its magnitude fixed,
infinite and electrical resistivity is zero.
does not produce a current of same magnitude I, in the
Examples of conductors are all metals like copper, silver, opposite direction, figure.
aluminium, tungsten etc.
CURRENT ELECTRICITY 7

I (mA)
To remember the value of colour coding used for carbon
resistor, the following sentences are found to be of great
help (where bold letters stand for colours).
B B ROY Green, Britain Very Good Wife Gold Silver.
V V Way of finding the resistance of carbon resistor from its
colour coding.
In the system of colour coding, Strips of different colours
are given on the body of the resistor, figure. The colours on
strips are noted from left to right.
(c) The relation between V and I is not unique, i.e., there is more
than one value of V for the same current I, figure.

I (mA)

A B C R
(i) Colour of the first stip A from the end indicates the first
significant figure of resistance in ohm.
(ii) Colour of the second strip B indicate the second significant
figure of resistance in ohm.
V
(iii) The colour of the third strip C indicates the multiplier,
i.e., the number of zeros that will follow after the two
7. COLOUR CODE FOR CARBON RESISTORS significant figure.
(iv) The colour of fourth strip R indicates the tolerance limit
The colour code for carbon resistance is given in the
of the resistance value of percentage accuracy of resistance.
following table.

Colour code of carbon resistors


8. COMBINATION OF RESISTORS

8.1 Resistances in Series


Colour Letter as No. Mulitplier Colour Tolerance
anAid to Resistors are said to be connected in series, if the same
memory current is flowing through each resistor when some poential
difference is applied across the combination.
0
Black B 0 10 Gold 5%
1
Brown B 1 10 Silver 10%
2
Red R 2 10 No colour 20%
3
Orange O 3 10
4
Yellow Y 4 10
5
Green G 5 10
6
Blue B 6 10 1. Let V be the potential difference applied across A and B
7 using the battery . In series combination, the same current
Violet V 7 10
(say I) will be passing through each resistance.
8
Grey G 8 10
2. Let V1, V2, V3 be the potential difference across R1, R2 and
9
White W 9 10 R3 respectively. According to Ohm’s law
–1
Gold 10 V1 = IR1, V2 = IR2, V3 = IR3
–2
Silver 10 3. Here, V = V1 + V2 + V3 = IR1 + IR2 + IR3 = I (R1 + R2 + R3)
CURRENT ELECTRICITY 8

branches are different and I1, I2, I3 be the current through


the resistances R1, R2 and R3 respectively. Then,
I = I2 + I2 + I3
3. Here, potential difference across each resistor is V, therefore
4. If R s is the equivalent resistance of the given series V = I1R1 = I2 R2 = I3R3
combination of resistances, figure, then the potential
difference across A and B is, V V V
or I1  , I2  , I3 
R1 R2 R3
V = IRs.
We have Putting values, we get
IRs = I (R1 + R2 + R3)
V V V
I  
or R s  R1  R 2  R 3 R1 R 2 R 3

Memory note 4. If Rp is the equivalent resistance of the given parallel


combination of resistance, figure, then
In a series resistance circuit, it should be noted that :
(i) the current is same in every resistor.
(ii) the current in the circuit is independent of the relative
positions of the various resistors in the series.
(iii) the voltage across any resistor is directly proportional to
the resistance of the resistor.
(iv) the total resistance of the circuit is equal to the sum of the V = IRp or I = V/Rp
individual resistances, plus the internal resistance of a cell
we have
if any.
(v) The total resistance in the series circuit is obviously more V V V V 1 1 1 1
     
than the greatest resistance in the circuit. R p R 1 R 2 R 3 or R p R1 R 2 R 3

Thus, the reciprocal of equivalent resistance of a number of


8.2 Resistances in Parallel
resistor connected in parallel is equal to the sum of the
Any number of resistors are said to be connected in parallel reciprocals of the individual resistances.
if potential difference across each of them is the same and
is equal to the applied potential difference. Memory note

In a parallel resistance circuit, it should be noted that :


(i) the potential difference across each resistor is the same
and is equal to the applied potential difference.
(ii) the current through each resistor is inversely proportional
to the resistance of that resistor.
(iii) total current through the parallel combination is the sum
of the individual currents through the various resistors.
(iv) The reciprocal of the total resistance of the parallel
combination is equal to the sum of the reciprocals of the
individual resistances.
1. Let V be the potential difference applied across A and B (v) The total resistances are connected in series, the current
with the help of a battery . through each resistance is same. When the resistance are
in parallel, the pot-diff. accross each resistance is the same
2. Let I be the main current in the circuit from battery. I divides
and not the current.
itself into three unequal parts because the resistances of these
CURRENT ELECTRICITY 9

(iii) Potential drop inside the cell = ir


9. CELL
(iv) Equation of cell E  V  ir (E > V)
The device which converts chemical energy into electrical energy
is known as electric cell. Cell is a source of constant emf but not E 
(v) Internal resistance of the cell r    1  R
constant current. V 
(vi) Power dissipated in external resistance (load)
2
V2  E 
+ P  Vi  i 2 R    .R
A R Rr
Anode –
Cathode
+ – E2
+ – Power delivered will be maximum when R  r so Pmax  .
4r
Symbol of cell

This statement in generalised from is called “maximum
+
power transfer theorem”.
Electrolyte

2
P max = E /4r
(1) Emf of cell (E) : The potential difference across the
P
terminals of a cell when it is not supplying any current is
called it’s emf.
(2) Potential difference (V) : The voltage across the R=r
terminals of a cell when it is supplying current to external R
resistance is called potential difference or terminal
voltage. Potential difference is equal to the product of (vii) When the cell is being charged i.e. current is given to the
current and resistance of that given part i.e. V = iR. cell then E = V – ir and E < V.
(3) Internal resistance (r) : In case of a cell the opposition (2) Open circuit : When no current is taken from the cell it
of electrolyte to the flow of current through it is called is said to be in open circuit.
internal resistance of the cell. The internal resistance of a
cell depends on the distance between electrodes (r  d), R
area of electrodes [r  (1/A)] and nature, concentration C D A B
(r  C) and temperature of electrolyte [r  (1/ temp.)].
A cell is said to be ideal, if it has zero internal resistance.
E, r
9.1 Cell in Various Positions
(i) Current through the circuit i = 0
(1) Closed circuit : Cell supplies a constant current in the
circuit. (ii) Potential difference between A and B, VAB = E
(iii) Potential difference between C and D, VCD = 0
R
(3) Short circuit : If two terminals of cell are join together
V= iR by a thick conducting wire
i

E, r R=0

E
(i) Current given by the cell i 
Rr E, r

(ii) Potential difference across the resistance V  iR


CURRENT ELECTRICITY 10

(i) Maximum current (called short circuit current) flows


plates of cells are connected together their emf’s are added to
E each other while if their similar plates are connected together
momentarily isc 
r their emf’s are subtractive.
(ii) Potential difference V = 0

Memory note E1 E1
E2 E2
1. It is important to note that during charging of a cell, the E eq = E 1 + E 2 Eeq = E1 – E2 (E1 > E2)
positive electrode of the cell is connected to positive r eq = r 1 + r 2 req = r1 + r2
terminal of battery charger and negative electrodes of the
cell is connected to negative terminal of battery charger.
In this process, current flows from positive electrode to (1) Series grouping : In series grouping anode of one cell is
negative electrode through the cell. Refer figure connected to cathode of other cell and so on. If n identical
cells are connected in series
Charger
+ – E, r E, r E, r E, r
I I

i
 V =  + Ir R

Hence, the terminal potential difference becomes greater


than the emf of the cell.
(i) Equivalent emf of the combination E eq  nE
2. The difference of emf and terminal voltage is called lost
voltage as it is not indicated by a voltmeter. It is equal to Ir. (ii) Equivalent internal resistance req  nr

nE
9.2 Distinction between E.M.E. and Potential Difference (iii) Main current = Current from each cell  i 
R  nr
E.M.F. of a Cell Potential Difference (iv) Potential difference across external resistance V  iR

1 The emf of a cells is the 1. The potential difference V


(v) Potential difference across each cell V ' 
maximum potential between the two points is n
difference between the the difference of potential
2
two electrodes of a cell between those two points  nE 
(vi) Power dissipated in the external circuit    .R
when the cell is in the in a closed circuit.  R  nr 
open circuit.
2. It is independent of the 2. It depends upon the resis-  E2 
(vii) Condition for maximum power R  nr and Pmax  n  
resistance of the circuit tance between the two points  4r 
and depends upon the of the circuit and current
(viii) This type of combination is used when nr << R.
nature of electrodes and flowing through the
the nature of electrolyte circuit. (2) Parallel grouping : In parallel grouping all anodes are
of the cell. connected at one point and all cathode are connected together
3. The term emf is used for 3. The potential difference is at other point. If n identical cells are connected in parallel
the source of electric measured between any two
E, r
current. points of the electric circuit.
4. It is a cause. 4. It is an effect. E, r

E, r
9.3 Grouping of Cells
i
R
In series grouping of cell’s their emf’s are additive or subtractive
while their internal resistances are always additive. If dissimilar
CURRENT ELECTRICITY 11

(i) Equivalent emf Eeq = E (iii) Main current flowing through the load
(ii) Equivalent internal resistance R eq  r / n
nE mnE
i 
E nr mR  nr
(iii) Main current i  R
m
R r/n
(iv) Potential difference across external resistance = p.d. (iv) Potential difference across load V = iR
across each cell = V = iR V
(v) Potential difference across each cell V ' 
i n
(v) Current from each cell i ' 
n i
(vi) Current from each cell i ' 
2 n
 E 
(vi) Power dissipated in the circuit P    .R
 R r/n  nr
(vii) Condition for maximum power R  and
m
 E2 
(vii) Condition for max. power is R  r / n and Pmax  n   E2
 4r  Pmax  (mn)
4r
(viii) This type of combination is used when nr >> R
(viii) Total number of cell = mn
Generalized Parallel Battery Memory note

r1 Note that (i) If the wo cells connected in parallel are of the


E1
same emf  and same internal resistance r, then
E2 r2
r  r
eq  
r3 Eeq
rr
E3 r eq

1 1 1 2 r
   or req 
req r r r 2

(ii) If n identical cells are connected in parallel, then the


E1 E 2 E equivalent emf of all the cells is equal to the emf of one
  ... n
r r2 rn 1 1 1 1 cell.
E eq  1 and    ... .
1 1 1 req r1 r2 rn
  ... 1 1 1 n
r1 r2 rn    ...  n terms  or req = r/n
req r r r
(3) Mixed Grouping : If n identical cell’s are connected in a
row and such m row’s are connected in parallel as shown.
10. ELECTRIC CURRENT
E, r E, r E, r
(1) The time rate of flow of charge through any cross-section
1
1 2 n
ΔQ dQ
2 is called current. i  Lim  . If flow is uniform
Δt  0 Δt dt
i
m Q
then i  . Current is a scalar quantity. It’s S.I. unit is
V t
ampere (A) and C.G.S. unit is emu and is called biot (Bi),
R or ab ampere. 1A = (1/10) Bi (ab amp.)
(2) Ampere of current means the flow of 6.25  10 18
(i) Equivalent emf of the combination Eeq = nE
electrons/sec through any cross–section of the conductor.
nr (3) The conventional direction of current is taken to be the
(ii) Equivalent internal resistance of the combination req 
m direction of flow of positive charge, i.e. field and is
CURRENT ELECTRICITY 12

opposite to the direction of flow of negative charge as (i) Solids : In solid conductors like metals current carriers
shown below. are free electrons.
(ii) Liquids : In liquids current carriers are positive and
+ + – – negative ions.
i i
+ + – – (iii) Gases : In gases current carriers are positive ions and
F F free electrons.
(iv) Semi conductor : In semi conductors current carriers are
(4) The net charge in a current carrying conductor is zero. holes and free electrons.
(5) For a given conductor current does not change with (v) The amount of charge flowing through a crossection of a
change in cross-sectional area. In the following figure conductor from t = ti to t = tf is given by :
i1 = i2 = i3
tf
q   I dt
ti

i1 i2 i3
From Graphs
(i) Slope of Q vs t graph gives instantaneous current.

(6) Current due to translatory motion of charge : If n


particle each having a charge q, pass through a given area Q
in time t then

+ +
+ +
t
+ + t'

If n particles each having a charge q pass per second per (ii) Area under the I vs t graph gives net charge flown.
unit area, the current associated with cross-sectional area A
is i = nqA
I
If there are n particle per unit volume each having a charge
q and moving with velocity v, the current thorough, cross
section A is i = nqvA (Qnet)

(7) Current due to rotatory motion of charge : If a point


t
charge q is moving in a circle of radius r with speed v t1 t2

(frequency , angular speed  and time period T) then


q qv qω
corresponding current i = qν = = = 11. KIRCHHOFF’S LAW
T 2πr 2π
11.1 Kirchhoff’s first law or Kirchhoff’s junction law
or Kirchhoff’s current law.
r q
1. the algebraic sum of the currents meeting at a junction in a
closed electric circuit is zero, i.e.,  I  0

(8) Current carriers : The charged particles whose flow in 2. Consider a junction O in the electrical circuit at which
a definite direction constitutes the electric current are the five conductors are meeting. Let I1, I2, I3, I4 and I5 be
the currents in these conductors in directions, shown in
called current carriers. In different situation current
figure,
carriers are different.
CURRENT ELECTRICITY 13

I3 I2

I1
I4
I5

3. Let us adopt the following sign convention : the current


flowing in a conductor towards the junction is taken as
positive and the current flowing away from the junction is We adopt the following sign convention :
taken as negative.
Traverse a closed path of a circuit once completely in
4. According to Kirchhoff’s first law, at junction O clockwise or anticlockwise direction.
(–I1) + (–I2) + I3 + (–I4) + I5 = 0 Difference between Kirchhoff’s I and II laws
or –I1 – I2 + I3 – I4 + I5 = 0
or I  0 First Law Second Law

or I3 + I5 = I 1 + I 2 + I 4 1. This law supports the 1. This law supports the law


law of conservation of of conservation of energy.
5. i.e., total current flowing towards the junction is equal to
charge.
total current flowing out of the junction.
2. According to this law 2. According to this law
6. Current cannot be stored at a junction. It means, no point/
junction in a circuit can act as a source or sink of charge. I  0     IR

7. Kirchhoff’s first law supports law of conservation of 3. This law can be used in 3. This law can be used in
charge. open and closed circuits. closed circuit only.

11.2 Kirchhoff’s Second law or Kirchhoff’s loop law


or Kirchhoff’s voltage law. 12. EXPERIMENTS

The algebraic sum of changes in potential around any closed 12.1 Galvanometer
path of electric circuit (or closed loop) involving resistors
It is an instrument used to detect small current passing through it
and cells in the loop is zero, i.e.,  V  0.
by showing deflection. Galvanometers are of different types e.g.
In a closed loop, the algebraic sum of the emfs and algebraic moving coil galvanometer, moving magnet galvanometer, hot wire
sum of the products of current and resistance in the various galvanometer. In dc circuit usually moving coil galvanometer
arms of the loop is zero, i.e.,     IR  0. are used.
Kirchhoff’s second law supports the law of conservation of (i) It’s symbol : G ; where G is the total
energy, i.e., the net change in the energy of a charge, after
internal resistance of the galvanometer.
the charge completes a closed path must be zero.
(ii) Full scale deflection current : The current required for
Kirchhoff’s second law follows from the fact that the
full scale deflection in a galvanometer is called full scale
electrostatic force is a conservative force and work done by
deflection current and is represented by ig.
it in any closed path is zero.
(iii) Shunt : The small resistance connected in parallel to
Consider a closed electrical circuit as shown in figure.
galvanometer coil, in order to control current flowing
containing two cells of emfs. 1 and 2 and three resistors of
through the galvanometer is known as shunt.
resistances R1, R2 and R3.
CURRENT ELECTRICITY 14

Table : Merits and demerits of shunt i


(c) To pass nth part of main current (i.e. i g  ) through the
n
Merits of shunt Demerits of shunt
G
To protect the galvano- Shunt resistance decreases the galvanometer, required shunt S = .
(n –1)
meter coil from burning . sensitivity of galvanometer.
12.3 Voltmeter
It can be used to convert
any galvanometer into It is a device used to measure potential difference and is always
ammeter of desired range. put in parallel with the ‘circuit element’ across which potential
difference is to be measured.
12.2 Ammeter
V
It is a device used to measure current and is always connected
in series with the ‘element’ through which current is to be R
measured.
+ –
V
R
(i) The reading of a voltmeter is always lesser than true value.
i A (ii) Greater the resistance of voltmeter, more accurate will
be its reading. A voltmeter is said to be ideal if its
– resistance is infinite, i.e., it draws no current from the
+
V circuit element for its operation.
(iii) Conversion of galvanometer into voltmeter : A
(i) The reading of an ammeter is always lesser than actual galvanometer may be converted into a voltmeter by
current in the circuit. connecting a large resistance R in series with the
(ii) Smaller the resistance of an ammeter more accurate will galvanometer as shown in the figure.
be its reading. An ammeter is said to be ideal if its
resistance r is zero. R
G
(iii) Conversion of galvanometer into ammeter : A Vg = igG (V – Vg)
galvanometer may be converted into an ammeter by ig
connecting a low resistance (called shunt S) in parallel to
the galvanometer G as shown in figure. V

(a) Equivalent resistance of the combination = G + R


S
(b) According to ohm’s law Maximum reading of V which
i – ig can be taken V = ig (G + R); which gives
i
G
ig V V 
Required series resistance R = –G = – 1 G
ig  
Ammeter  Vg 
(c) If nth part of applied voltage appeared across galvanometer
GS
(a) Equivalent resistance of the combination  (i.e. Vg 
V
) then required series resistance R = (n – 1) G..
G S
n
(b) G and S are parallel to each other hence both will have
equal potential difference i.e. i g G  (i  i g ) S ; which 12.4 Wheatstone Bridge Principle
gives Wheatstone Bridge Principle states that if four resistances
ig P, Q, R and S are arranged to form a bridge as shown in
Required shunt S = G figure, if galvanometer shows no deflection, the bridge is
(i – ig )
balanced.
CURRENT ELECTRICITY 15

In that case through the galvanometer or in other words VB = VD. In the

P R P R
 balanced condition = , on mutually changing the
Q S Q S
position of cell and galvanometer this condition will not
change.
(ii) Unbalanced bridge : If the bridge is not balanced current
will flow from D to B if VD > VB i.e. (VA  VD )  (VA  VB )
which gives PS > RQ.
(iii) Applications of wheatstone bridge : Meter bridge, post
office box and Carey Foster bridge are instruments based
on the principle of wheatstone bridge and are used to
measure unknown resistance.

12.5 Slide Wire Bridge or Meter Bridge

Proof : A slide wire bridge is a practical form of Wheatstone bridge.


Let I be the total current given out by the cell. On reaching It consists of a wire AC of constantan or manganin of 1
the point A, it is divided into two parts : metre length and of uniform area of cross-section.
1. I1 is flowing through P A meter scale is also fitted on the wooden board parallel to
the length of the wire.
2. (I – I1) through R.
At B, the current I1 is divided into two parts, Ig through the Copper strip fitted on the wooden board in order to provide
galvanometer G and (I1 – Ig) through Q. two gaps in strips.

A current (I – I1 + Ig) through S. Across one gap, a resistance box R and in another gap the
unknown resistance S are connected.
Applying Kirchhoff’s Second Law to the closed circuit
ABDA, we get The positive pole of the battery E is connected to terminal
A and the negative pole of the battery to terminal C through
I1P + Ig G – (I – I1) R = 0 ...(1)
one way key K.
where G is the resistance of galvanometer.
The circuit is now exactly the same as that of the Wheatstone
Again applying Kirchhoff’s Second Law to the closed circuit bridge figure.
BCDB, we get
(I1 – Ig) Q – (I – I1 + Ig) S – IgG = 0 ...(2)
The value of R is adjusted such that the galvanometer shows
no deflection, i.e., Ig = 0. Now, the bridge is balanced. Putting
Ig = 0 in (1) and (2) we have
I1P – (I – I1) R = 0 or I1P = (I – I1) R ...(3)
and I1Q – (I – I1) S = 0 or I1Q = (I – I1) S ...(4)

P R
Dividing (3) by (4), we get 
Q S

Note that in Wheatstone bridge circuit, arms AB and BC


having resistances P and Q form ratio arm. The arm AD,
having a resistance R, is a known variable resistance arm and Adjust the position of jockey on the wire (say at B) where
arm DC, having a resistance S is unknown resistance arm. on pressing, galvanometer shows no deflection.

(i) Balanced bridge : The bridge is said to be balanced when Note the length AB ( = l say) to the wire. Find the length BC
deflection in galvanometer is zero i.e. no current flows ( = 100 – l) of the wire.
CURRENT ELECTRICITY 16

According to Wheatstone bridge principle If I is the current flowing through the wire, then from Ohm’s
law; V = IR; As, R = l/A
P R

Q S   I 
 V  I  K,  where K  
If r is the resistance per cm length of wire, then A  A
P = resistance of the length l of the wire AB = lr or Vl (if I and A are constant)
Q = resistance of the length (100 – l) of the wire BC = (100 – l) r. i.e., potential difference across any portion of potentiometer
wire is directly proportional to length of the wire of that
r R  100    protion.
  or S    R
100    r S    Here, V/l = K = is called potential gradient, i.e., the fall of
Knowing l and R, we can calculate S. potential per unit length of wire.

12.6 Potentiometer and its principle of working 12.7 Determination of Potential Difference
using Potentiometer
Potentiometer is an apparatus used for measuring the emf
A battery of emf  is connected between the end terminals A
of a cells or potential difference between two points in an
and B of potentiometer wire with ammeter A1, resistance
electrical circuit accurately.
box R and key K in series. This circuit is called an auxillary
A potentiometer consists of a long uniform wire generally circuit. The ends of resistance R1 are connected to terminals
made of manganin or constantan, stretched on a wooden A and Jockey J through galvanometer G. A cell 1 and key
board. K1 are connected across R1 as shown in figure.
Its ends are connected to the binding screws A and B. A
meter scale is fixed on the board parallel to the length of the
wire. The potentiometer is provided with a jockey J with
the help of which, the contact can be made at any point on
the wire, figure. A battery  (called driving cell), connected
across A and B sends the current through the wire which is
kept constant by using a rheostat Rh.

Working and Theory : Close key K and take out suitable


resistance R from resistance box so that the fall of potential
across the potentiometer wire is greater than the potential
difference to be measured.
It can be checked by pressing, firstly the jockey J on
potentiometer wire near end A and later on near end B, the
deflections in galvanometer are in opposite directions.

Principle : The working of a potentiometer is based on the Close key K1. The current flows through R1. A potential
fact that the fall of potential across any portion of the wire difference is developed across R1. Adjust the position of
is directly proportional to the length of that portion provided jockey on potentiometer wire where if pressed, the
the wire is of uniform area of cross-section and a constant galvanometer shows no deflection. Let it be when jockey is
current is flowing through it. at J. Note the length AJ (= l) of potentiometer wire. This
would happen when potential difference across R1 is equal
Suppose A and  are respectively the area of cross-section to the fall of potential across the potentiometer wire of length
and specific resistance of the material of the wire. l. If K is the potential gradient of potentiometer wire, then
Let V be the potential difference across the portion of the potential difference across R1, i.e.,
wire of length l whose resistance is R. V = Kl
CURRENT ELECTRICITY 17

If r is the resistance of potentiometer wire of length L, then i.e., 1 = Kl1 ...(1)


current through potentiometer wire is where K is the potential gradient across the potentiometer
 wire.
I
Rr Now remove the plug from the gap between 1 and 3 and
insert in the gap between 2 and 3 of two way key so that
   cells of emf 2 comes into the circuit. Again find the position
Potential drop across potentiometer wire  Ir   r
Rr of jockey on potentiometer wire, where galvanometer shows
no deflection. Let it be at J2. Note the length of the wire AJ2
Potential gradient of potentiometer wire, i.e., fall of potential
( = l2 say). Then
per unit length is
2 = Kl2 ...(2)
  r   r
K  . V   1 1
RrL RrL 
Dividing (1) by (2), we get
2  2
Hence, V can be calculated.

12.8 Comparison of emfs of two cells using Potentiometer 12.9 Precautions of experiment

A battery of emf  is connected between the end terminals A 1. The current in the potentiometer wire from driving cell must
and B of potentiometer wire with rheostat Rh, ammeter A1 be kept constant during experiment.
and key K in series. 2. While adjusting the position of jockey on potentiometer wire,
The positive terminals of both the cells are connected to the edge of jockey should not be rubbed on the wire,
point A of the potentiometer. Their negative terminals are otherwise area of cross-section of wire will not be uniform
connected to two terminals 1 and 2 of two ways key, while and constant.
its common terminal 3 is connected to jockey J through a 3. The current in the potentiometer wire from driving cell
galvanometer G. should not be passed for long time as this would cause
Insert the plug in the gap between the terminals 1 and 3 of heating effect, resulting the change in resistance of wire.
two way key so that the cell of emf 1 is in the circuit. Memory note
Adjust the position of jockey on potentiometer wire, where A balance point is obtained on the potentiometer wire if
if pressed, the galvanometer shows no deflection. Let it be the fall of potential along the potentiometer wire, due to
when jockey be at J1. Note the length AJ1 (= l1 say) of the wire. driving cell is greater than the e.m.f. of the cells to be balanced.
There is no current in arm A1J1. It means the potential of
positive terminal of cell = potential of the point A, and the 12.10 Determination of Internal Resistance
potential of negative terminal of cell = potential of the point J1. of a Cell by Potentiometer Method

To find the internal resistance r of a cell of emf  using


potentiometer, set up the circuit as shown in figure.

Therefore, the e.m.f. of the cell ( =1) is equal to potential


difference between the points A and J1 of the potentiometer
wire.
CURRENT ELECTRICITY 18

Close key K and maintain suitable constant current in the potentiometer wire circuit with the help of rheostat and using
potentiometer wire with the help of rheostat Rh. Adjust the a single cell.
position of jockey on the potentiometer wire where if Difference between Potentiometer and Voltmeter
pressed, the galvanometer show no deflection. Let it be when
jockey is as J1. Note the length AJ1 (= l1) of the potentiometer Potentiometer Voltmere
wire. Now emf of the cell,  = potential difference across
1. It measures the emf of 1. It measures the emf of a
the length l1 of the potentiometer wire.
a cell very accurately. cell approximately.
or  = Kl1 ...(1)
2. While measuring emf it 2. While measuring emf, it
where K is the potential gradient across the wire. does not draw any current drws some current from
Close key K1 and take out suitable resistance R from the from the source of the source of emf.
resistance box in the cell circuit. Again find the position of known emf.
the jockey on the potentiometer wire where galvanometer 3. While measuring emf, 3. While measuring emf the
shows no deflection. Let it be at J2. Note the length of the the resistance of poten- resistance of voltmeter is
wire AJ2 ( = l2 say). As current is being drawn from the cell, tiometer becomes infinite. high but finite.
its terminal potential difference V is balanced and not emf 4. Its sensitivity is high. 4. Its sensitivity is low.
. Therefore, potential difference between two poles of the 5. It is based on null 5. It is based on deflection
cell, V = potential difference across the length l2 of the deflection method. method.
potentiometer wire 6. It can be used for 6. It can be used only to
i.e. V = Kl2 ...(2) various purposes. measure emf or potential
difference.
Dividing (1) by (2), we have

 1
 ...(3) 13. HEATING EFFECT OF CURRENT
V 2

We know that the internal resistance r of a cell of emf , When some potential difference V is applied across a resistance
when a resistance R is connected in its circuit is given by R then the work done by the electric field on charge q to flow
through the circuit in time t will be
V  
r  R    1 R ...(4)
V V  V2 t
W = qV = Vit = i2R  Joule .
R
Putting the value (3) in (4), we get

    i R
r   1  1 R  1 2  R A B

 2  2

Thus, knowing the values of l1, l2 and R, the internal This work appears as thermal energy in the resistor.
resistance r of the cell can be determined. Heat produced by the resistance R is

12.11 Sensitiveness of Potentiometer W Vit i 2 Rt V2 t


H    Cal. This relation is called joules
J 4  2 4  2 4  2R
The sensitiveness of potentiometer means the smallest
heating.
potential difference that can be measured with its help.
Some important relations for solving objective questions are as
The sensitiveness of a potentiometer can be increased by
follow :
decreasing its potential gradient. The same can be achieved.
(i) By increasing the length of potentiometer wire.
(ii) If the potentiometer wire is of fixed length, the potential
gradient can be decreased by reducing the current in the
CURRENT ELECTRICITY 19

of any electrical appliance can be calculated by rated


Condition Graph
VR2
If R and t are constant power and rated voltage i.e. by using R = e.g.
H PR
H  i 2 and H  V
2

220  220
Resistance of 100W, 220 volt bulb is R   484 
i (or V) 100
If i and t are constant (series grouping) H (4) Power consumed (illumination) : An electrical appliance
HR (Bulb, heater, …. etc.) consume rated power (PR) only if
applied voltage (VA) is equal to rated voltage (VR) i.e. If
R
VA2
VA = VR so Pconsumed = PR. If VA < VR then Pconsumed 
If V and t are constant (Parallel grouping) R
H

1 VR2
H also we have R  so
R PR
R

If V, i and R constant H  t H  V2 
Pconsumed (Brightness)   A2  .PR
 VR 

t Pconsumed  (Brightness)
e.g. If 100 W, 220 V bulb operates on 110 volt supply then
13.1 Electric Power
2
The rate at which electrical energy is dissipated into other  110 
Pconsumed     100  25 W
forms of energy is called electrical power i.e.  220 

W V2
P= = Vi = i 2 R =
t R
(1) Units : It’s S.I. unit is Joule/sec or Watt
If VA < VR then % drop in output power
Bigger S.I. units are KW, MW and HP,
(PR  Pconsumed )
remember 1 HP = 746 Watt  100
PR
(2) Rated values : On electrical appliances
(Bulbs, Heater … etc.) For the series combination of bulbs, current through
them will be same so they will consume power in the
ratio of resistance i.e., P  R {By P = i2R) while if they
are connected in parallel i.e. V is constant so power
40 W consumed by them is in the reverse ratio of their
1
220 V resistance i.e. P  .
R

Wattage, voltage, ……. etc. are printed called rated values (5) Thickness of filament of bulb : We know that resistance
e.g. If suppose we have a bulb of 40 W, 220 V then rated VR2
power (PR) = 40 W while rated voltage (VR) = 220 V. It of filament of bulb is given by R  , also R   l ,
PR A
means that on operating the bulb at 220 volt, the power
dissipated will be 40 W or in other words 40 J of electrical 1
hence we can say that A  PR  i.e. If rated
energy will be converted into heat and light per second.  Thickness  R
(3) Resistance of electrical appliance : If variation of power of a bulb is more, thickness of it’s filament is also
resistance with temperature is neglected then resistance more and it’s resistance will be less.
CURRENT ELECTRICITY 20

1
If applied voltage is constant then P(consumed) 
R
VA2 If quantity of water is given n litre then
(By P  ). Hence if different bulbs (electrical
R
4180(4200) n 
appliance) operated at same voltage supply then t
p
1
Pconsumed  PR  thickness 
R
13.2 Electric Energy

The total electric work done or energy supplied by the


source of emf in maintaining the current in an electric circuit
Different bulbs
for a given time is called electric energy consumed in the
25W 100W 1000W
220V 220V 220V circuit.
 Electric energy, W = VIt = P.t
 Electric energy = electric power × time
 Resistance R25 > R100 > R1000 SI unit of electric energy is joule, wherre
 Thickness of filament t1000– > t100 > t40 1 joule = 1 volt × 1 ampere × 1 second = 1 watt × 1 second
 Brightness B1000 > B100 > B25 The commercial unit of electric energy is called a kilowatt-
hour (kWh) or Board to Trade Unit (BOT) or UNIT of
(6) Long distance power transmission : When power is Electricity, in brief, where
transmitted through a power line of resistance R, power-
1 kWh = 1 kilo watt × 1 hour = 1000 watt × 1 hour
loss will be i 2 R
Thus 1 kilo watt hour is the total electric energy consumed
Now if the power P is transmitted at voltage V
when an electrical appliance of power 1 kilo-watt works for
P2 one hours.
P = Vi i.e. i = (P/V) So, Power loss  R
V2 6
1 kWh = 1000 Wh = (1000 W) × (60 × 60 s) = 3.6 × 10 J.
Now as for a given power and line, P and R are constant
Note that the number of units of electricity consumed = No.
so Power loss  (1/V2)
So if power is transmitted at high voltage, power loss watt  hour
of kWh =
will be small and vice-versa. e.g., power loss at 22 kV 1000
is 10 –4 times than at 220 V. This is why long distance
power transmission is carried out at high voltage. Electric energy  VI t  I2 Rt  V 2 t / R

(7) Time taken by heater to boil the water : We know that


13.3 Electricity Consumption
heat required to raise the temperature  of any
substance of mass m and specific heat S is H = m.S. (1) The price of electricity consumed is calculated on the
basis of electrical energy and not on the basis of electrical
Here heat produced by the heater = Heat required to raise power.
the temp.  of water.
(2) The unit Joule for energy is very small hence a big
J(m.S.) practical unit is considered known as kilowatt hour
i.e. p  t = J  m.S.  t 
p (KWH) or board of trade unit (B.T.U.) or simple unit.
{J = 4.18 or 4.2 J/cal) (3) 1 KWH or 1 unit is the quantity of electrical energy which
4180 ( or 4200) m  dissipates in one hour in an electrical circuit when the
for m kg water t  electrical power in the circuit is 1 KWH thus
p
{S = 1000 cal/kgoC) 1 KWH = 1000 W  3600 sec = 3.6  106 J.
CURRENT ELECTRICITY 21

(4) Important formulae to calculate the no. of consumed units If they are connected If they are connected

Total watt  Total hours in series in parallel


is n 
1000 1 1 1
  PP = P1 + P2
PS P1 P2
13.4 Combination of Bulbs (or Electrical Appliances)
1 1 1 H P H1 H 2
     
Bulbs (Heater etc.) Bulbs (Heater etc.) HS / tS H1 / t1 H2 / t 2 tp t1 t2
are in series are in parallel
 HS=H1= H2  H p = H 1 = H2
(1) Total power consumed (1) Total power consumed 1 1 1
so ts = t1+ t2 so  
1 1 1 t p t1 t 2
   .... Ptotal = P1 + P2 + P3 .... + Pn
Ptotal P1 P2 i.e. time taken by i.e. time taken by parallel
combinationto boil the combination to boil the
P1 same quantity of water same quantity of water
P1 P2
t1 t 2
P2 ts = t1 + t2 tp 
Supply t1  t 2
Supply

(3) If three identical bulbs are connected in series as shown


(2) In ‘n’ bulbs are identical, (2) If ‘n’ identical bulbs are in figure then on closing the switch S. Bulb C short
circuited and hence illumination of bulbs A and B
P increases
Ptotal  in parallel. Ptotal = nP
N
A B C
Pconsumed  Brightness  Pconsumed  Brightness 

+ S
1 1 V–
V  R   PR  i 
Prated R
i.e. in series combination i.e. in parallel combination
bulb of lesser wattage will bulb of greater wattage will Reason : Voltage on A and B increased.
give more bright light and give more bright light and (4) If three bulbs A, B and C are connected in mixed
p.d. appeared across it will more current will pass combination as shown, then illumination of bulb A
be more. through it. decreases if either B or C gets fused

Some Standard Cases for Series and Parallel Combination A

P
(1) If n identical bulbs first connected in series so PS  and
n V
B C
P
then connected in parallel. So PP = nP hence P  n 2
PS
Reason : Voltage on A decreases.
(2) An electric kettle has two coils when one coil is switched
on it takes time t1 to boil water and when the second coil (5) If two identical bulb A and B are connected in parallel
is switched on it takes time t2 to boil the same water. with ammeter A and key K as shown in figure.
CURRENT ELECTRICITY 22

It should be remembered that on pressing key reading of 14. ELECTRICAL CONDUCTING


ammeter becomes twice.
MATERIALS FOR SPECIFIC USE
A
K (1) Filament of electric bulb : Is made up of tungsten which
has high resistivity, high melting point.
V (2) Element of heating devices (such as heater, geyser or
A B
press) : Is made up of nichrome which has high resistivity
and high melting point.
(3) Resistances of resistance boxes (standard resistances) :
Reason : Total resistance becomes half.
Are made up of alloys (manganin, constantan or nichrome)
Concepts these materials have moderate resistivity which is
practically independent of temperature so that the
When a heavy current appliance such us motor, heater
specified value of resistance does not alter with minor
or geyser is switched on, it will draw a heavy current changes in temperature.
from the source so that terminal voltage of source
decreases. Hence power consumed by the bulb (4) Fuse-wire : Is made up of tin-lead alloy (63% tin + 37%
decreases, so the light of bulb becomes less. lead). It should have low melting point and high resistivity.
It is used in series as a safety device in an electric circuit
and is designed so as to melt and thereby open the circuit
if the current exceeds a predetermined value due to some
Heater

r fault. The function of a fuse is independent of its length.

K Safe current of fuse wire relates with it’s radius as i  r 3/2 .


(5) Thermistors : A thermistor is a heat sensitive resistor
usually prepared from oxides of various metals such as
13.5 Some aspects of heating effects of current nickel, copper, cobalt, iron etc. These compounds are also
semi-conductor. For thermistors  is very high which
1. The wire supplying current to an electric lamp are not may be positive or negative. The resistance of thermistors
practically heated while the filament of lamp becomes white changes very rapidly with change of temperature.
hot.
We know that in series connections the heat produced due i
to a current in a conductor is proportional to its resistance
(i.e. H  R). The filament of the lamp and the supply wires
are in series. The resistance of the wire supplying the current
to the lamp is very small as compared to that of the filament V
of the lamp. Therefore, there is more heating effect in the
filament of the lamp than that in the supply wires. Due to it, Thermistors are used to detect small temperature change
the filament of the lamp becomes white hot whereas the and to measure very low temperature.
wires remain practically unheated.
15. SUPER CONDUCTIVITY
2. Electric Iron
Prof. K. Onnes, in 1911, discovered that certain metals and alloys
3. Electric Arc
at very low temperature lose their resistance considerably. This
4. Incandescent electric lamp phenomenon is known as super-conductivity. As the temperature
decreases, the resistance of the material also decreases, but when
F the temperature reaches a certain critical value (called critical
temperature or transition temperature), the resistance of the
B material completely disappears i.e., it becomes zero. Then the
material behaves as if it is a super-conductor and there will be
flow of electrons without any resistance whatsoever. The critical
temperature is different for different materials. It has been found
5. Fuse wire
CURRENT ELECTRICITY 23

that mercury at critical temperature 4.2 K, lead at 7.25 K and V = potential difference across the conductor and l =
niobium at critical temperature 9.2 K become super-conductors. length of the conductor. Electric field out side the current
A team of scientists discovered that an alloy of plutonium, cobalt carrying conductor is zero.
and gallium exhibits super conductivity at temperatures below
18.5 K. Since 1987, many superconductors have been prepared + + + + + +
with critical temperature upto 125 K, as listed below + –
Ein = 0 Ein = V/l
Bi2Ca2Sr2Cu3O10 at 105 K and Tl2Ca2Ba2Cu3O10 at 125 K.
+ + + + + +
The super-conductivity shown by materials can be verified by
simple experiment. If a current is once set up in a closed ring of
super-conducting material, it continues flowing for several weeks 1
4. For a given conductor JA = i = constant so that J 
after the source of e.m.f. has been withdrawn. A
The cause of super-conductivity is that, the free electrons in super- i.e., J1 A1 = J2 A2 ; this is called equation of continuity
conductor are no longer independent but become mutually
dependent and coherent when the critical temperature is reached.
i
The ionic vibrations which could deflect free electrons in metals J1
J2
are unable to deflect this coherent or co-operative cloud of i
electrons in super-conductors. It means the coherent cloud of A1 A2
electrons makes no collisions with ions of the super-conductor
and, as such, there is no resistance offered by the super-conductor
to the flow of electrons. 5. The drift velocity of electrons is small because of the
frequent Collisions suffered by electrons.
Super-conductivity is a very interesting field of research all over
the world these days. The scientists have been working actively 6. The small value of drift velocity produces a large amount
to prepare super-conductor at room temperature and they have of electric current, due to the presence of extremely large
met with some success only. number of free electrons in a conductor. The propagation
of current is almost at the speed of light and involves
Application of super conductors
electromagnetic process. It is due to this reason that the
1. Super conductors are used for making very strong electric bulb glows immediately when switch is on.
electromagnets.
7. In the absence of electric field, the paths of electrons
2. Super conductivity is playing an important role in material
science research and high energy partical physics. between successive collisions are straight line while in
presence of electric field the paths are generally curved.
3. Super conductivity is used to produce very high speed
computers. NA x d
8. Free electron density in a metal is given by n 
4. Super conductors are used for the transmission of electric A
power. where N A = Avogadro number, x = number of free
electrons per atom, d = density of metal and A = Atomic
TIPS AND TRICKS weight of metal.
9. In the absence of radiation loss, the time in which a fuse will
1. Human body, though has a large resistance of the order of
melt does not depends on it’s length but varies with radius
k (say 10 k), is very sensitive to minute currents even
as low as a few mA. Electrocution, excites and disorders as t  r 4 .
the nervous system of the body and hence one fails to 10. If length (l) and mass (m) of a conducting wire is given
control the activity of the body.
2
2. dc flows uniformly throughout the cross-section of then R  .
m
conductor while ac mainly flows through the outer surface
area of the conductor. This is known as skin effect. V
11. Macroscopic form of Ohm’s law is R  , while it’ss
3. It is worth noting that electric field inside a charged i
conductor is zero, but it is non zero inside a current microscopic form is J =  E.
12. After stretching if length increases by n times then
V
carrying conductor and is given by E  where resistance will increase by n 2 times i.e. R 2  n 2 R1 .

CURRENT ELECTRICITY 24

Similarly if radius be reduced to 1/n times then area of 24. Resistance of a conducting body is not unique but
cross-section decreases 1/n2 times so the resistance depends on it’s length and area of cross-section i.e. how
becomes n4 times i.e. R 2  n 4 R1 . the potential difference is applied. See the following
figures
13. After stretching if length of a conductor increases by x%
then resistance will increases by 2x % (valid only if x < 10%)
14. Decoration of lightning in festivals is an example of series
grouping whereas all household appliances connected in
parallel grouping. b b
15. Using n conductors of equal resistance, the number of
possible combinations is 2n – 1. a a
16. If the resistance of n conductors are totally different, then
the number of possible combinations will be 2n. Length = a Length = b
17. If n identical resistances are first connected in series and Area of cross-section = b  c Area of cross-section = a  c
then in parallel, the ratio of the equivalent resistance is
Resistance R   
b 
Resistance R   
a 
Rpn2  
given by  .  bc ac
Rs 1
25. Some standard results for equivalent resistance
18. If a wire of resistance R, cut in n equal parts and then
these parts are collected to form a bundle then equivalent
R1 R2
R
resistance of combination will be 2 .
n A B
R5
19. If equivalent resistance of R1 and R2 in series and parallel
be Rs and Rp respectively then
R3 R4
1
R1  R s  R s2  4R s R p  and
2  
R1R 2 (R 3  R 4 )  (R1  R 2 )R 3 R 4  R5 (R1  R 2 )(R3  R 4 )
1 R AB 
R 2   R s  R s2  4R s R p  R 5 (R1  R 2  R 3  R 4 )  (R1  R 3 )(R 2  R 4 )
2 
20. If a skeleton cube is made with 12 equal resistance each
R1 R2
having resistance R then the net resistance across

A B
H R3
G

E
F
R2 R1
D C

A B 2R1R 2  R 3 (R1  R 2 )
R AB 
2R 3  R1  R 2
5
21. The longest diagonal (EC or AG)  R
6 R1 R1 R1 R1
A

3
22. The diagonal of face (e.g. AC, ED, ....)  R R3 R3 R3 R3
4
B
7 R2 R2 R2 R2
23. A side (e.g. AB, BC.....)  R
12
CURRENT ELECTRICITY 25

32. If n identical cells are connected in a loop in order, then


1 1 1/ 2
R AB  (R1  R 2 )  (R1  R 2 ) 2  4R 3 (R1  R 2 )  emf between any two points is zero.
2 2

E, r
R1 R1 R1 R1
A
E, r E, r

R2 R2 R2 Close
loop
B E, r

E, r
n cell
1  R 
R AB  R1 1  1  4  2 
2   R1   33. In parallel grouping of two identical cell having no internal

resistance
26. It is a common misconception that “current in the circuit
will be maximum when power consumed by the load is R R
maximum.”
E E
27. Actually current i = E/(R + r) is maximum (= E/r) when
R = min = 0 with PL = (E/r)2 × 0 = 0 min. while power E E
consumed by the load E2R/(R + r)2 is maximum (= E2/4r)
Eeq = E Eeq = 0
when R = r and i  (E / 2r)  max ( E / r).

28. Emf is independent of the resistance of the circuit and 34. When two cell’s of different emf and no internal resistance
depends upon the nature of electrolyte of the cell while are connected in parallel then equivalent emf is
potential difference depends upon the resistance between indeterminate, note that connecting a wire with a cell with
the two points of the circuit and current flowing through no resistance is equivalent to short circuiting. Therefore
the circuit. the total current that will be flowing will be infinity.
29. Whenever a cell or battery is present in a branch there R
must be some resistance (internal or external or both)
present in that branch. In practical situation it always E1
happen because we can never have an ideal cell or battery
with zero resistance. E2

30. In series grouping of identical cells. If one cell is wrongly


connected then it will cancel out the effect of two cells 35. In the parallel combination of non-identical cell’s if they
e.g. If in the combination of n identical cells (each having are connected with reversed polarity as shown then
emf E and internal resistance r) if x cell are wrongly equivalent emf

connected then equivalent emf Eeq  (n  2 x ) E and E1r2  E 2 r1


E eq 
equivalent internal resistance req  nr . r1  r2

31. Graphical view of open circuit and closed circuit of a


cell. i1 E1, r 2

V Vmax ; E; i = 0 i i2 E2, r2

imax = E/r; V = 0 i
CURRENT ELECTRICITY 26

36. Wheatstone bridge is most sensitive if all the arms of 39. The measurement of resistance by Wheatstone bridge is
bridge have equal resistances i.e. P = Q = R = S not affected by the internal resistance of the cell.
37. If the temperature of the conductor placed in the right 40. In case of zero deflection in the galvanometer current
gap of metre bridge is increased, then the balancing length flows in the primary circuit of the potentiometer, not in
decreases and the jockey moves towards left. the galvanometer circuit.
38. In Wheatstone bridge to avoid inductive effects the battery 41. A potentiometer can act as an ideal voltmeter.
key should be pressed first and the galvanometer key
afterwards.

SOLVED EXAMPLES

Example - 1
2
2 × 10 m/s. This figure indicates the random vibrational
(a) Estimate the average drift speed of conduction
speeds of copper atoms in a conductor. Note that the drift
electrons in a copper wire of cross-sectional area
–7 2 –5
1.0 × 10 m carrying a current to 1.5 A. Assume that speed of electrons is much smaller, about 10 times the
each copper contributes roughly one conduction typical thermal speed at ordinary temperatures.
3 3
electron. The density of copper is 9.0 × 10 kg/m , (ii) An electric field travelling along the conductor has a
and its atomic mass is 63.5 u. speed of an electromagnetic wave, namely equal to 3.0 ×
8 –1
(b) Compare the drift speed obtained above with, 10 ms . This drift speed is, in comparison, extremely small;
–11
smaller by a factor of 10 .
(i) thermal speeds of copper atoms at ordinary
temperatures, Example - 2
(ii) speed of propagation of electric field along the
(a) In above example, the electron drift speed is
conductor which causes the drift motion. –1
estimated to be only a few mm s for currents in
Sol. (a) The direction of drift velocity of conduction electrons is the range of a few amperes ? How then is current
opposite to the electric field direction, i.e., electrons drift in established almost the instant a circuit is closed ?
the direction of increasing potential. The drift speed vd is (b) The electron drift arises due to the force
given by Eq. vd = (1/neA) experienced by electrons in the electric drift arises
–19 –7 2
Now, e = 1.6 × 10 C, A = 1.0 × 10 m , I = 1.5 A. The due to the force experienced by a electrons in the
density of conduction electrons, n is equal to the number of electric field inside the conductor. But force should
atoms per cubic metre (assuming one conduction electron causes acceleration. Why then do the electrons
per Cu atom as reasonable from its valence electrons count acquire a steady average drift speed ?
3
of one). A cubic metre of copper has a mass of 9.0 × 10 kg. (c) If the electron drift speed is so small, and the
23
Since 6.0 × 10 copper atoms have a mass of 63.5 g, electrons’ charge is small, how can we still obtain
large amounts of current in a conductors ?
6.0  1023
n  9.0  106 = 8.5 × 1028 m–3 (d) When electrons drfit in a metal from lower to higher
63.5
potential, does it means that all the ‘free’ electrons
which gives,
of the metal are moving in the same direction ?
1.5
vd  (e) Are the paths of electrons straight lines between
8.5  10  1.6  1019  1.0  10 7
28
successive collisions (with the positive ions of the
–3 –1 –1
= 1.1 × 10 ms = 1.1 mms metal) in the (i) absence of electric field, (ii) presence
(b) (i) At a temperature T, the thermal speed of a copper atom of electric field ?
2
of mass M is obtained from [<(1/2) mv > =(3/2) kBT] and
Sol. (i) Electric field is established throughout the circuit, almost
is thus typically of the order of k B T / M , where kB is the instantly (with the speed of light) causing at every point a
Boltzmann constant. For copper at 300 K, this is about
CURRENT ELECTRICITY 27

local electron drift. Establishment of a current does not have rule and the symmetry in the problem.
to wait for electrons from one end of the conductor travelling Next take a closed loop, say ABCC’ EA, and apply
to the other end. However, it does take a little while for the Kirchhoff’s second rule :
current to reach its steady value. –IR – (1/2) IR – IR +  = 0
(b) Each ‘free’ electron does accelerate, inceasing its drift speed where R is the resistance of each edge and  the emf of
until it collides with a positive ion of the metal. It loses its battery. Thus,
drift speed after collision but starts to accelerate and
5
increases its drift speed again only to suffer a collision again  IR
and so on. On the average, therefore, electrons acquire only 2
a drift speed. The equivalent resistance Req of the network is

(c) Simple, because the electron number density is enormous,  5


29 –3 R eq   R
~ 10 m . 3I 6
(d) By no means. The drift velocity is superposed over the large For R = 1, Req = (5/6) and for  = 10 V, the total current
random velocities of electrons. (=3I) in the network is
(e) In the absence of electric field, the paths are straight lines; 3I = 10 V /(5/6)  = 12 A, i.e., I = 4A
in the presence of electric field, the paths are, in general The current flowing in each edge can now be read off from
curved. the figure.
Example - 3 Example - 4
A battery of 10 V and negligible internal resistance is A resistance of R  draws current from a potentiometer.
connected across the diagonally opposite corners of a the potentiometer has a total resistance R0  (figure).
cubical network consisting of 12 resistors each of A voltage V is supplied to the potentiometer. Derive an
resistance 1. Determine the equivalent resistance of the experssion for the voltage across R when the sliding
network and the current along each edge of the cube. contact is in the middle of the potentiometer.
I 3I V
D'
C'
I/2
R0
I A C
I/2 I
A' B' B
R
I/2 I/2

I Sol. While the slide is in the middle of the potetiometer only


C
D I/2 half of its resistance R0/2) will be between the points A and
I
B. Hence, the total resistance between A and B, say, R1, will
I/2
be given by the following expression :
A
3I I B E 10V 1 1 1
 
R1 R  R 0 / 2 
Sol. The network is not reducible to a simple series and parallel
combinations of resistors. There is, however, a clear R0R
symmetry in the problem which we can exploit to obtain R1 
R 0  2R
the equivalent resistance of the network.
The paths AA’, AD and AB are obviously symmetrically The total resistance betwen A and C will be sum of resistance
placed in the network. Thus, the current in each must be the between A and B and B and C, i.e., R1 + R0/2
same, say, I. Further, at the corners A’, B and D, the incoming  The current flowing through the potentiometer will be
current I must split equally into the two outgoing branches.
In this manner, the current in all the 12 edges of the cube V 2
I 
are easily written down in terms of I, using Kirchhoff’s first R1  R 0 / 2 2R1  R 0
CURRENT ELECTRICITY 28

The voltage V1 taken from the potentiometer will be the (iii) Putting this value of t in (i), we have
product of current I and resistance R1.
b 1 b2 b2 b2 b2
i=a–b×  c   a   a .
 2V  c 2 c c 2c 2c
V1  IR1     R1
 2R1  R 0  As this value of i is less than that at t = 0, it must be minimum.

Substituting for R1, we have a b2


So minimum value of current = a –
2c
2V R0  R Example - 7
V1  
 R R  R 0  2R
2 0   R0 A steady beam of –particles travelling with kinetic
 R 0  2R  energy E = 83.5 keV carries a current of I = 0.2 A.
–27
Mass of –particle = 6.68 × 10 kg.
2VR (i) If this beam strikes a plane surface at an angle
V1 
2R  R 0  2R q = 60° with normal to the surface, how many
–particles strike the surface in t = 4 second ?
2VR (ii) How many –particles are there in length l = 20
or V1 
R 0  4R cm of the beam ?
Example - 5 –6
Sol. Here, I = 0.02 A = 0.2 × 10 A,
For the given carbon resistor, let the first strip be 3 3 –19
yellow, second strip be red, third strip be orange and E = 83.5 keV = 83.5 × 10 eV = 83.5 × 10 × 1.6 × 10 J
–16
fourth be gold. What is its resistance ? = 83.5 × 1.6 × 10 J
–19
(i) Charge on –particle, q = 2e = 2 × 1.6 × 10 C
Sol. We know that the number for yellow, red and orange are 4,
2 and 3. Gold represents tolerance of ± 5%. Hence the value No. of –particles crossing per second a cross-secton of
3
of the given resistance = 42 × 10  ± 5%. beam,

Example - 6 1 0.2  106


n   6.25  1011 / s .
The change flowing in a conductor varies with time as, q 2  1.6  1019
1 1 No. of –particles striking a surface in time 4 second
a  at  bt 2  ct 3 where a, b, c are positive constants.
2 6 11 12
= nt = 6.25 × 10 × 4 × 2.5 × 10 .
Then, find
Note : Then is no significance of angle q (= 60°) for finding the
(i) the initial current number of –particles striking the surface.
(ii) the time after which the value of curent reaches a (ii) If v is the velocity of –particle while travelling towards a
maximum value surface, then
(iii) the maximum or minimum value of current.
1 2E 2  83.5  1.6  1016
E mv 2 or v  
Sol. (i) Current, 2 m 6.68  1027
6
= 2 × 10 m/s
dq d  1 1  1
i   at  bt 2  ct 2   a  bt  ct 2 ...(i) 6
It means a beam of length v = 2 × 10 m crosses a section in
dt dt  2 6  2
one second. But number of –particle passing through a
11
When t = 0, i=a section in one second is, n = 6.25 × 10
di n
(ii)   b  ct.  No. of –particles in until length of the beam 
dt v
For i to be maximum or minimum,
No. of –particles in length l of the beam
di b
 0   b  ct or t  n 6.25 1011
dt c    0.20 = 6.25 × 104
v 2 106
CURRENT ELECTRICITY 29

Example - 8 1 1
Conductance, G    0.33S
A wire of resistance 5  is drawn out so that its length R 3
is increased by twice its original length. Calculate its
new resistance. 1 
Electrical conductivity,   
 RA
Sol. Here, R1 = 5; l1 = l say ; A1 = A, say
3 5 –1
R2 = ? ; l2 = 2l + l = 3l; A2 = ?  = 1.85 × 10 sm
3  5.4  10 6
If V is the volumne of the wire of length l and area of cross-
section A, then Example - 11

V = Al or A = V/l The resistance of a tungsten filament at 150° C is 133.


What will be its resistance at 500°C ? The temperature
  2 –1
coefficient of resistance of tungsten is 0.0045°C at 0°C.
Now R     R  2
A V/ V
–1
[  and V are constants] Sol. Here; R150 = 133 , R500 = ?,  = 0.0045°C
2 We know, Rt = R0 (1 + t)
R 2  22
Hence,  or R 2   3   9
R1 12  R150 = R0 (1 +  × 150)
5 2
or 133 = R0 (1 + 0.0045 × 150) ...(i)
or R2 = 5 × 9 = 45 .
And R500 = R0 (1 +  × 500)
Example - 9
or R500 = R0 (1 + 0.0045 × 500) ...(ii)
A copper wire is stretched to kake it 0.2% longer. What
Dividing (ii) by (i) we get
is the percentage change in its resistance ?
R 500 1  0.0045  500 3.25
Sol. The mass m of the wire of length l, area of cross section A  
133 1  0.0045  150 1.675
and density d is given by
3.25
m or R 500   1.33  258 
m = Ald or A  1.675
d
The resistance R of the wire of resistivity  is given by Example - 12
It is desired to make a 20.0  coil of wire whose
  2 d temperature coefficient of resistance is zero. To do this,
R   k 2
A m carbon resistor of resistance R1 is placed in series with
where k =  d/m is a constant of the wire. an iron resistor of resistance R2. The proportion of iron
and carbon are so chosen that R1 + R2 = 20 for all
dR 2d
  ; temperatures near 20°C. Find the values of R1 and R2.
R  Temperature coefficient of resistance for carbon,
–3 –3
dR 2  0.2 C = – 0.5 × 10 /°C and that of iron is Fe = 5 × 10 /°C.
% increase in resistance   100   100
R 100 Sol. Let t°C be the rise in temperature. As per question we
= 0.4 % need,
Example - 10 R1 (1 + Ct) + R2 (1 + Fe t) = 20
A wire carries a current of 0.5 A, when a potential Since, R1 + R2 = 20, therefore, R1 C t + R2 Fe t = 0
difference of 1.5 V is applied across it. What is its or R1 C = – R2 Fe
conductance ? If the wire is of length 3 m and area of –3 –3
2 or R1 × (–0.5 × 10 ) = – R2 × 5 × 10
cross-section 5.4 mm , calculate its conductivity.
or R1 = 10 R2.
Sol. Here, I = 0.5 A; V = 1.5 V, l = 3 m; So, 10 R2 + R2 = 20
2 2 –6 2
A = 5.4 mm = 5.4 mm = 5.4 × 10 m or 11 R2 = 20
V 1.5 or R2 = 20/11 = 1.82  and R1 = 10 × 1.82 = 18.2 
Now, resistance, R    3
I 0.5
CURRENT ELECTRICITY 30

Example - 13 Example - 15
(a) The colour coded resistor is shown in figure. Six equal resistances each of 4 ohm are connected to
Compute the resistance in megaohm. form a net work as shown in figure. What is the
Violet Gold resistance between A and B ?
C

A B
Brown Yellow

Sol. We know that number for brown colour is 1, for violet colour
Sol. The upper five resistances between A and B will form a
is 7 and that of yellow is 4. For gold, accuracy is 5%.
balanced. Wheatstone bridge. Hence there will be no current
 Resistance of resistor shown in figure. in arm CO. The equivalent circuit will be as shown in figure.
4
= 17 × 10 ± 5% Here, resistances of arm ACB, AOB and AB are all in
= 0.17 Megaohm ± 5% parallel. The effective resistance R is given by
Example - 14 C
A uniform wire or resistance R is shaped into a regular
n sided polygon where n is even. Find the equivalent
resistance between (i) opposite corners of polygon
O
(ii) adjacent corners of polygon.

Sol. Let the polygon be as shown in figure. The resistance of A B


each side of polygon = R/n.

D E
1 1 1 1
  
R  4  4  4  4 4
C F
11 2 4 1
   or R = 2 
8 8 2
B G
Example - 16

A H In the adjoining circuit, figure

(i) For resistance between opposite corners C and G, we have


two resistance in parallel each of value R/2.
Therefore, the equivalent resistance between opposite corner
is

 R / 2  R / 2 R
  .
 R / 2  R / 2 4
(ii) For resistance between adjacent corner A and H, we have R1 = 100 Ω , R2 = R3 = 50 Ω ,
two resistance of R/n and (n – 1) R/n in parallel. The R4 =75 Ω ,  = 4.75 V..

equivalent resistance is 
 R / n   n  1 R / n  n  1 Work out the equivalent resistance of the circuit and
R the current in each resistor.
 R / n    n  1 R / n n2
CURRENT ELECTRICITY 31

Sol. Here R2, R3 and R4 are connected in parallel. Their effectiv Sol. The four cells are connected in parallel to the parallel
resistance Rp will be given by combination of two 15  resistors as shown in figure.

1 1 1 1 1 1 1
     
R p R 2 R 3 R 4 50 50 75

6  6  4 16 4
  
300 300 75

or Rp = 75/4  .
Total resistance of circuit = R1 + Rp = 100 + 75/4 = 475/4 

4.75
Current, i1   0.04 A;
475 / 4
Pot drop across R1 = i1 = 0.04 × 100 = 4V
Pot. drop. across all other resistances = 4.75 – 4 V = 0.75 V
Current through R2 or R3, i2R1 = i3 = 0.75/50 = 0.015 A;
Current through R4, i4 = 0.75/75 = 0.01 A.
Let r be the internal resistance of each cell. As cells are in para-
Example - 17
llel, the effective emf of all the cells,  = emf of one cell = 2V.
A cell of emf  and internal resistance r gives a current
Terminal voltage, V = 1.6 V,
of 0.5 A with an external resistance of 12  and a current
of 0.25 A with an external resistance of 25 . Calculate Total internal resistance r’ of four cells in parallel is obtained
(a) internal resistance of the cell and (b) emf of the cell. from

1 1 1 1 1 4
Sol. Let R be external resistance in the series with the cell of     
emf  and internal resistance r. The current in circuit is r r r r r r
 r
I or r 
Rr 4
In firs case ,
15  15
I = 0.5 A, R = 12  Total external resistance, R   7.5 
15  15
 Internal resistance r’ of the parallel combination of cells is
0.5  or  = (12 + r) × 0.5 = 6.0 + 0.5 r ...(i)
12  r give by
In second case,
V  2  1.6  7.5
I = 0.25 A, R = 25  r   R     7.5 
 V   1.6  4

 0.25  or  = (25 + r) × 0.25 = 6.25 + 0.25 r r 7.5
25  r or  or r  7.5 
4 4
From (i) and (ii), 6.0 + 0.5 r = 6.25 + 0.25 r
or r = 1 Example - 19
From (i),  = 6.0 + 0.5 × 1 = 6.5 V Draw V–I graph for ohmic and non-ohmic material.
Give one example for each.
Example - 18
Four identical cells each of emf 2V, are joined in parallel Sol. V–I graph for an ohmic material is a straight line passing
providing supply of current to external circuit consisting through origin figure. Example (a) is a resistor made of
of two 15  resistors joined in parallel. The terminal manganin.
voltage of the cells as read by an ideal voltmeter is 1.6 V. V–I graph for a non-ohimc material is a curve, i.e., non-
Calculate the internal resistance of each cell. linear or straight line not passing through origin. Figure (a),
(b) and (c). Example is junction diode or electrolyte.
CURRENT ELECTRICITY 32

I I Applying Kirchhoff’s Second Law to the closed circuit


ARBE1A,
(I1 + I2) × 12 + I1 × 0.5 – 6 = 0
or 12.5 I1 + 12 I2 = 6 ...(i)
V V Applying Kirchhoff’s Second Law to the closed circuit
(a) (b) ARBE2 A, we get
I (I1 + I2) × 12 + I2 × 1 – 10 = 0
or 12 I1 + 13 I2 = 10 ...(ii)
Multiplying (i) by 13 and (ii), we get ...(iii)
162.5 I1 + 156 I2 = 78 ...(iv)
V 144 I1 + 156 I2 = 120
(c)
Substracting (iv) from (iii), we get
Example - 20
42
Write the mathematical relation between mobility and 18.5 I1 = – 42 or I1    2.27 A
18.5
drift velocity of charge carriers in a conductor. Name
the mobile charge carriers responsible for conduction of Negative sign shows that I1 actually flows in a direction
electric current in (a) an electrolyte (b) an ionised gas. opposite to what is shown in figure. Substituting the value
of I1 in (ii), we get
drift velocity vd
Sol. Mobility,    42 12  42 185  504
electric field E 12   13 I 2  10 or 13I 2  10  
18.5 18.5 18.5
(a) The charge carries in an electrolyte are positive and negative
ions. or 689
I2   2.86 A.
18.5  13
(b) The charge carriers in an ionised gas are electrons and
positively charged ions. Example - 22
Example - 21 When two known resistance R and S are connected in
A battery of 6 V and internal resistance 0.5  is joined the left and right gaps of a metre bridge, the balance
in parallel with another of 10 V and internal resistance point is found at a distance l1 from the zero end of the
1. The combination sends a current through an metre bridge wire. An unknown resistance X is now
external resistance of 12. Find the current through connected in parallel to the resistance S and the balance
each battery. point is now found at a distance l2 from the zero end of
the metre bridge wire figure. Obtain a formula for X
Sol. The arrangement of the two batteries and the external in terms of l1, l2 and S.
resitance is shown in figure. Let I1 and I2 be the circuit given
by the two batteries so that the current through the external
resistance is (I1 + I2) as shown in the figure.
E1 = 6V

I1

A B

Sol. When resistance R and S are connected to the left and right
I2
I1 + I2 I1 + I2
gaps of metre bridge and bridge is balanced at length l1 from
zero end, then
E2 = 10V
R 1
 ...(i)
S 100  1 
CURRENT ELECTRICITY 33

When unknown resistance X is connected in parallel to S, Sol. The equivalent cirucit will be as shown in figure, which is a
then effective resistance in right gap is balanced Wheatsone bridge. Therefore the resistance of
arm CD is ineffective. Hence we have the resistance of
SX arm ACB = R + R = 2R in parallel with the resistance of
S 
S X arm ADB = R + R = 2R.
Now, balance point is obtained at length l2,
2 R  2R
Effective resistance between A and B is = = R
R 2 2R  2R
 
S 100   2  Example - 24
Putting the value of S’, we have Two bulbs are marked 220 V – 100 watt and 220 V – 50
watt respectively. They are connected in series to 220 V
R S  X  2 mains. Find the ratio of heats generated in them.
 ...(iii)
SX 100   2 
Sol. Here, P1 = 100 W, P2 = 50 W, V = 220 V
Dividing (iii) by (i), we get Let R1, R2 be the resistances of 100 watt bulb and 50 watt
2
bulb respectively. As R = v /P, therefore,
S X 2 100  1  or S  100  1 
  1  2 2 2
R1 = (200) /100 = 484  and R2 = (220) /50 = 968 .
X 100   2  1 X 1 100   2 
When bulbs are connected in series to mains, there will be
S 100  2  1 2  100 1  1 2 100   2  1  same current I in each bulb. Then
or   2 2 –1 2 2 –1
X 1 100   2  1 100   2  H1 = I R1 = I × 484 Js and H2 = I R2 = I × 968 Js

H1 484 1
1 100   2    
or X S H 2 968 2
100   2  1 
Example - 25
Example - 23 We have 30 watt, 6 volt bulb which we want to glow by
Five equal resistance each of R  are connected in a a supply of 120 V. What will have to be done for it ?
network as shown in figure. Calculate the equivalent
Sol. Here, P = 30 W, V = 6 V
resistance between the points A and B.
2
V2 6
 Resistance of the bulb, R    1.2 
P 30

P 30
Current capacity of the bulb, I    5A
V 6

Supply voltage, V’ = 120 V


Let R’ be the resistance used in series with the bulb to have
a current of 5 A in the circuit
Total resistance = R’ + R = (R’ + 1.2)
 Current, I = V’ /(R’ + 1.2)

120
or 5
R   1.2

or 120 – 1.2 = 22.8  in series.


R 
5
CURRENT ELECTRICITY 34

Example - 26
A heater is designed to operate with a power of 1000
W in a 100 V line. It is connected to two resistance of
10  and R , Figure. If the heater is now operating
with a power of 62.5 W, calculate the value R.

Effective resistance Rp between A and B will be

1 1 1 1 1 31 42
     or R p  
R p 3 14 6 6 42 31

E 1.8
Sol. Resistance of heater, Current from the cell, I    0.89 A
Rp  r 42 2

V 2 1002 31 3
R'   10 
P 1000 Potential diff. between A and B, VA – VB = I Rp
When heater is in the circuit, let pot. diff. across heater be 42
V, then  0.89   1.21 V
31
2
62.5 = V /10
or
2
V = 625 1.21
Current in 3 resistance   0.4 A
3
or V = 25 volt
Power dissipated in the whole circuit (including the cell) is
Therefore, current through heater, I1 = V/R’ = 25/10 = 2.5 A.
Voltage drop across 10  resistor connected in series with 2 2  42 2
the heater = 100 – 25 = 75 volt. P = I (R + r) = (0.89)    = 1.6 watt
 31 3 
75 Example - 28
Main current   7.5 A
10
At room temperature (27.0°C) the resistance of a
 current through resistance R = 7.5 – 2.5 = 5A. heating element is 100 . What is the temperature of
25 the element if the resistance is found to be 117 , give
 Resistance, R   5 that the temperature co-effecient of the material of the
5 –4 –1
resistor is 1.70 × 10 °C .
Example - 27
–4 –1
In figure, the emf of the cell is 1.8 V and internal Sol. Here, R27 = 100 ; Rt = 117 ; t = ?; = 1.70 × 10 °C
resistance is 2/3 , calculate the current in the 3
resistance and the power dissiplated in the whole circuit. R t  R 27
We know that  
R 27  t  27 

R t  R 27
or t  27 
R 27  

R t  R 27 117  100
 t  27   27
R 27   100  1.7  104

= 1000 + 27 = 1027°C
Sol. The equivalent circuit is shown in the figure
CURRENT ELECTRICITY 35

Example - 29
(a) In a meter bridge, thebalance point is found to be
at 39.5 cm from the left end A, if an unknown
resistor X is in the left gap and a known resistor Y
of resistance 12.5  is in the right gap. Determine
the resistance of X. Why are the connection between
resistors in a Wheatstone or metre bridge made to
thick copper strips ?
Sol. (a) For maximum effective resistance, the n resistors must
(b) Determine the balance point of the above bridge if be connected in series.
X and Y are interchanged ?
Maximum effective resistance, in Rs = nR.
(c) Why happens if the galvanometer and cell are
For minimum effective resistance, the n resistors must be
interchanged at the balance point of the bridge ?
connectedd in parallel
Would the galvanometer show any current ?
Maximum effective resistance, Rp = R/n
Sol. (a) Here, l = 39.5 cm; R = X = ?; S = Y = 12.5  Rs nR
   n2
Rp R / n
100   100  39.5
As S  R  12.5  X
 39.5 (b) It is to be noted that (a) the effective resistance of parallel
combination of resistors is less than the individual resistance
or 12.5  39.5 and (b) the effective resistance of series combination of
X  8.16 
60.5 resistors ismore than individual resistance.
Thick copper strips are used to minimise resistance of the Case (i) Parallel combination of 1 and 2  is connected
connections which are not accounted in the formula. in series with 3.
(b) As X and Y are interchanged, therefore, l1 and l2 (i.e., lengths) Effective resistance of 1 and 2  in parallel will be given
are also interchanged.
by R p  1  2  2 
Hence, l = 100 – 39.5 = 60.5 cm. 1 2 3
(c) The galvanometer will show no current.
2
 Equivalent resistance of  and 3 in series
Example - 30 3

(a) Given n resistors each of resistance R. How will you 2 11


 3  
combine them to get the (i) maximum (ii) effective 3 3
resistance ? What is the ratio of the maximum to Case (ii) : Parallel combination of 2  and 3  is
minimum resistance ? connected in series with 1  .
(b) Given the resistances of 1, 2, 3, how will you Equivalent resistance of 2  and 3  in parallel
combine them to get an equivalent them to get an
23 6
equivalent resistance of (i) (11/3)  (ii) (115/)  (iii)   
6 (iv) (6/11)  ? 23 5

(c) Determine the equivalent resistance of network 6


Equivalent resistance of  and 1  in series
shown in figure (a) and (b). 5

6 11
 1  
5 5
Case (iii) : All the resistance are to be connected in series.
Now
 Equivalent resistance = 1 + 2 + 3 = 6.
Case (iv) All the resistance are to be connected in parallel
CURRENT ELECTRICITY 36

1 1 1 1 Example - 33
 Equivalent resistance (R) is given by   
R 1 2 3 Is ohm’s law true for all conductors ? Name two types
6  3  2 11 6 of commercially available resistors.
  or R  .
6 6 11 Sol. No, it is true only for metallic conductors. Commercially
(c) The given net work is a series combination of 4 equal available resistors are : carbon resistors and wire bound
units. Each unit has 4 resistances in which, 2 resistances resistors.
(1  each in series) are in parallel with 2 other resistances Example - 34
(2  each in series). What do you mean by a linear resistor ?
 Effective resistance of two resistances (each of 1) in series
= 1 +1 = 2  Sol. A linear resistor is one which obeys Ohm’s law, i.e., for
which current graph is a straight line.
Effective resistance of two resistances (each of 2) in series
Example - 35
=2+2=4
If Rp is the resistance of one unit of resistances, then Explain why resistance becomes more in series
combination.
1 1 1 3 4
   or R p   Sol. In series combination of resistors, the effective length of
Rp 2 4 4 3
the conductor increases. As R  l, therefore resistance
 Total resistance of net work (4 such units) increases.
4 16 Example - 36
  4    5.33 
3 4
How is the current conducted in metals ? Explain.
For figure, the five resistances each of value R, are connected
in series. Sol. Every metal conductor has large number of free electrons
which move at random at room temperature. Their average
Their effective resistance = 5R.
thermal velocity at any instant is zero. When a pot different
Example - 31 is applied across the ends of a conductor, an electric field is
set up. Due to it, the free electrons of the conductor
Distinguish between static electricity and current
experience force due to electric field and drift towards the
electricity.
positive end of the conductor, causing the electric current
Sol. Static electricity deals with the study of charges at rest and (i.e. conduction current) in the conductor whose direction
current electricity deals with charges in motion. is opposite to the direction of motion of the free electronss
Example - 32 in the conductor.

V–I graoh for a metallic wire at two different Example - 37


temperature T1 and T2 is shown in the figure. Which Explain why resistance becomes less in parallel
of the two temperatures is higher and why ? combination.

V Sol. In parallel combination of resistors, the effective area of


Sol. We know that resistance, R  cross-section of the conductor increases. As R  1/A,
I
therefore, resistance decreases.

I
Example - 38
T1 What are the parameters of resistivity of a metal ? State
the relative for resistivity.
T2
Sol. Resistivity  of a metal conductor depends upon the nature
O V of material of the conductor which inturn depends upon ;
(i) number density of electrons n and (ii) time of relaxation
From figure, for given V, I1 > I2; therefore, R1 < R2.  of free electrons.
As R  T1 so T1 < T2, i.e., T2 is greater than T1. The resistivity is related to n and  according to relation,
CURRENT ELECTRICITY 37

m Example - 44

n e2  When is a Wheatstone bridge said to be balanced ?

This shows that the resistvity of the conductor is independent Sol. Wheatsone bridge is said to be balanced, when no current
of the dimensions of the conductor. flows through the galvanometer arm of Wheatstone bridge.
Example - 39 Example - 45
State the factors on which (i) in ternal resistance If the current flowing in the wire of the potentiometer
(ii) emf of a cell depend. be decreased, what will be effect on the position of zero
deflection in potentiometer ? Explain.
Sol. Internal resistance of a cell depends upon; (i) distance
Sol. If the current in the wire of potentiaometer is decreased, the
between the plates (ii) the nature of electrolyte (iii) the nature
potential gradient will decrease and hence the position of
of electrodes (iv) area of the plates, immersed in the
zero deflection will occur at longer length.
electroyle. If area increases, internal resistance decreases.
Example - 46
E.M.F. of a cell depends upon; (i) nature of electrodes
Prove that in parallel combination of electrical
(ii) nature and concentration of electrolyte used in the cell
applicances, total power consumption is equal to the
(iii) temperature of electrolyte.
sum of the powers of the individual appliances.
Example - 40
Sol. Consider electrical appliances of powers, P1, P2, P3 having
To reduce the brightness of a light bulb, should an
resistances R1, R2, R3 connected in parallel. Suppose they
auxiliary resistance be connected in series with it or in
are operated at main voltage V. Let P be the total power and
parallel ?
R be the total resistance of the combination of all appliances.
Sol. To reduce the brightness of a light bulb, we should decreases Then according to law of resistances in parallel,
the current flowing through the bulb, which is possible when 1 1 1 1
  
an auxiliary resistance is connected in series with the bulb. R R1 R 2 R 3
2
Example - 41 Multiplying both sides by V , we get
Explain how does the resistivity of a conductor depend V2 V2 V2 V2
  
upon (i) number density n of tree electrons, and (ii) R R1 R 2 R 3
relaxation time .
 V2 
or P = P 1 + P 2 + P3   P
Sol. Resistivity  of of a metal conductor is related with n and   R 
m 1 1 Hence total electric power consumed is equal to the sum of
as,   2 , i.e.,   amd   . the powers of the individual appliances.
ne  n 
Example - 47
Example - 42
A wire connected to a bulb does not glow, whereas the
Why do we prefer a potentiometer to measure emf of a filament of the bulb glows when same current flows
cell rather than a voltmeter ? through them. Why ?
Sol. A potentiometer does not draw any current from the cell Sol. Filament of bulb and supply wires are connected in series.
whose emf is to be determined, wheereas a voltmeter always Therefore, the same current flows through them. Since the
draws some little current. Therefore, emf measured by resistance of connecting wires is negligibly small as
voltmeter is slightly less then actual value of emf of the cell. compared to the resistance of filament and heat produced
ue to given current is directly proportional to its resistance
Example - 43 (from Joule’s law of heating), therefore, the heat produced
The emf of the driver cell in the potentiometer in the filament is very large. Hence the bulb glow, but the
experiment should be greater than the emf of the cell connected wires remain practically unheated.
to be determined. Why ? Example - 48
Sol. If its is not so, there will be smaller fall of potential across How does the use of fuse wire save the electrical
the potentiometer wire than the emf of the cell to be installations ?
determined and hence the balance point will not be obtained Sol. A fuse wire is one which has high resistance and low melting
on the potentiometer wire. point. This is connected in series with electrical installations.
CURRENT ELECTRICITY 38

When the supply voltage exceeds the safe limit, more heat Sol. A fuse wire should have (i) high resistivity (ii) low melting
is produced in the fuse wire. (  rate of heat production point and (iii) of suitable current rating corresponding to
2
P = V /R), and it melts. The circuit breaks and the damage the load in the circuit.
to the electrical installations is saved. Example - 54
Example - 49 Name the physical quantity which ahs its unit joule
–1
Name a few practical appliances of heating effect of coulomb . Is it a scalar or vector quantity ?
current. –1
Sol. A physical quantity with unit, joule, coulomb = workdone/
Sol. A large number of electrical appliances whose working is charge = potential difference. It is a scalar quantity.
based on heating effect of current are used in our daily life. Example - 55
A few examples are :
If the current in the electric bulb changes by 1%, then
(i) Safety fuse (ii) Electric lamp (iii) Electric heater (iv) by what percentage will the power change ?
Electric iron (v) Electric oven (vi) Electric geyser (vii) Electric
2
welding (viii) Electric kettle (ix) Thermionic tubes etc. Sol. P = I R or log P = 2 log I + log R; Differentiating it, we get
For the proper functioning of these appliances, the main
P 2I
requirement is the choice of the right kind of resistor and   0 (  R is constant);
proper design of the appliance or device for delivering heat P I
or light as desired.
% change in power  P  100  2I  100  2 1  100
Example - 50 P I 100
Long distance power transmission is carried on high = 2%.
voltage lines. Why ? Example - 56
Sol. When current I is transmitted through a power line of What is the power transferred per unit volume into
resistance R, joule heat in a resistor ?
2
Power loss = I R. 2 2
Sol. Power, P = I R = I l/A; ( R = l/A)
If the power P is transmitted at voltage V, then P = VI
Volume of resistor, V = Al
P2 Power transferred per unit volume
or I = (P/V)  Power loss  2 R 
V
2
For a given power and given line, P and R are constant. P I 2  / A  I 
2        J 2
Hence, power loss  (1/V ) V A A
it means if power is transmitted at high voltage, power loss
where I/A = J = current density.
will be small and vice-versa.
Example - 57
Example - 51
A heating coil of 2000 watt is immersed in an electric
Two bulbs of same wattage,one having a carbon filament
kettle. How much time will it take in raising the
and the other having a metallic filament, are connected
temperature of 1 litre of water from 4°C to 100°C ?
in series to the mains. Which one will glow more ?
Only 80% part of the therml energy produced is used
Sol. Resistance of carbon filament is more than that of metallic in raising the temperature of water.
filament. In series combination, there will be same current
2
through them. Since, heat produced, H = I Rt, therefore H Sol. Here, P = 200 W, t = ? (in second)
 R, thus carbon filament bulb will glow more. Volume of water = 1 litre = 1000 c.c.
Example - 52 Mass of water, m = volume × density = 1000 × 1 = 1000 gram
Of the bulbs in a house, one glows brighter than the Rise in temperature, 2 – 1 = 100 – 4 = 96°C
other, which of the two has a large resistance. –1 –1
We know sp. heat of water, s = 1 cal g °C
Sol. Power of dim bulb is less than the power of bright bulb, i.e.,  Heat taken by water = ms (2 – 1) = 1000 × 1 × 96 = 96000
Pdim < Pbright. At constant voltage P  1/R; so Rdim > Rbright. cal.
Example - 53 Energy spent in heating coil = Pt = 2000 × t
State the characteristics of fuse wire. Useful thermal energy produced = 80%
CURRENT ELECTRICITY 39

2000  t  80 Example - 60
= 2000 × t × 80/100 J = cal.
100  4.2 How can you make a potentiometer of a given length
more sensitive by using a resistance box ?
As this thermal energy is taken by water, therefore,
Sol. The sensitivity of a potentiometer is the smallest potential
2000  t  80
 96000 difference it can measure. It can be increased by reducing
100  4.2
the potential gradient, i.e., potential drops per unit length of
potentiometer wire. The same is possible by decreasing the
96000  100  4.2
or t = 252 seconds. durrent flowing in the potentiometer wire, which in turn can
2000  80
be achieved by using more resistance from resistance box
Example - 58 in series with the potentiometer wire.
A house is fitted with 20 lamps of 60 watt each, 10 fans Example - 61
consuming 0.5 A each and and electric kettle of A copper wire of diameter 0.16 cm is connected in series
resistance 110 . If the energy is supplied at 220 V and to an aluminium wire of diameter 0.25 cm. A current
costs 50 paise per k Wh, calculate bill for November of 10 ampere is passed through them. Find
while running these appliances for 6 hours a day.
(a) current density in copper wire.
2
Sol. Here, n1 = 20 lamps, P1 = 60 W, n = 10 fans, I2 = 0.5 A, (b) drift velocity of electron in the aluminium wire.
n3 = 1 kettle, R3 = 110  The number of free electrons per unit volume of
29 –3
V1 = V2 = V3 = 220 V, t1 = t2 = t3 = 6 h/day, aluminium wire is 10 m .

Rate = 50 paise/k Wh, Total cost = ? Sol. Since the two wires are joined in series, the current through
Electric energy consumed per day each wire is the same, i.e., I = 10 A.
2
(a) For copper wire, A = D /4,
V32 2
= n1P1t1 + n2V2I2t2 + n3 ×  t3 current density J = I/A = I/( D /4)
R3
2
(b) For aluminium wire, A’ =  D’ /4
2 –2 2
  220   (0.25 × 10 ) /4
 20  60  6  10  220  0.5  6  1  6 Wh –6 2 29 –3
 110  = 2.01 × 10 m , n = 10 m
Drift velocity, vd = I/n A’ e
7200  6600  2640
  16.44 kWh
1000 Example - 62
Total energy consumed per month = 16.44 × 30 = 493.2 kwh
A certain electric conductor has a square cross–
50 section, 20.0 mm on a side, and is 12 m long. The
 Total bill = 493.2 × resistance between its ends is 0.072 .
100
(a) What is the resistivity of the material ?
= 246.60 rupees.
(b) If the electric field magnitude in the conductor
Example - 59
is 0.12 V/m, what is the total current ?
Can you express the potential gradient in terms of
specific resistance of the wire ? If yes, find the relation. (c) If the material has 8 × 1028 free electrons per
cubic meter, find the average drift velocity
Sol. Yes, Potential gradient, under the condition of part (b).

potential difference applied (V) Sol. (a) A = (2 × 10–3)2 = 4 × 10–6 m2


k ,
length of wire ()
l = 12 m, R = 0.072 

V  IR  I I RA 0.072  4  10 6
i.e., k      / A   ,  
   A  12
where r is the specific resistance of the material of the wire. = 2.4 × 10–8 m.
CURRENT ELECTRICITY 40

(b) Electric field in the conductor = V/l = 0.12


emf
V = 0.12 × 12 = 1.44 Volts. current through the cell will be = i =
net resistance
i = V/R = 1.44/0.072 = 20 A
(c) Let drift velocity = Vd 50 20
i=  A.
n = 8 × 1028 electrons/m3 20×5 7
10+
Current = i = e n A Vd 20+5

i 20 For || combination of 5 and 20 ,


Vd  
enA 1.6  10  8  1028  4  1016
19
i R1 25 20 20
I2     A.
= 3.9 × 10–4 m/s. R1  R 2 7 20  5 7
Example - 63
Example - 65
The current in a wire varies with time according to
In the circuit shown, the EMF of the cell are :
the relation :
i = 4 A + (2t2) A/s2 E1 = 10 V; E2 = 40 V; R = 15; internal resistances
are r1 = 1 ; r2 = 2. Calculate the current through
(a) How many coulombs pass a cross–section
R and the potential difference across the terminals
of the wire in the time interval between t = 5
and t = 10 s ? of each cell.

(b) What constant current could transport the + – – +


E1 E2
same charge in same time interval ?
R
Sol. i (t) = 4 + 2 t2
10 10
  4  2t  dt  603.33 C
2
(a) q   i dt  Sol. The two cells are equivalent to a single cell of emf
5 5
E = E2 – E1 = 40 V – 10V = 30 V
q 603.33
(b) i c    120.67 A.
r1 r2
t 10  5 A + – – + C
Example - 64 E1 B E2
Calculate the current through the 5 resistance. The
cell has negligible internal resistance. R
E = 50 V
i + –

– +
I1 i 30V

i (1 + 2 + 15)
I2
As E2 > E1, the net emf follows the direction of E2
Sol. Net resistance across the cell
Current through each resistance (all are in series) is
20  5
 10  i = 3/18 = 5/3 A.
20  5
E = 50 V
p.d. across cells :
+ –
p.d. across the terminals of cell E1 is
= VA – VB = E1 + ir1 = 10 + 5/3 × 1 = 35/3 V.
p.d. across the terminals of cell E2 is
= VC – VD = E2 – ir2 = 40 – 5/3 × 2 = 110/3 V.
CURRENT ELECTRICITY 41

Example - 66 Example - 67
Find the current in each branch of the given circuit. The resistance of the galvanometer G in the circuit is
What is the energy supplied by the battery to the 25. The meter deflects full scale for a current of
circuit in one minute ? 10 mA. The meter behaves as an ammeter of three
different ranges. The range is 0–10 A, if the
terminals O and P are taken; range is 0 – 1 A
between O and Q; range is 0 – 0.1 A between O
+
– 14V and R. Calculate the resistance R 1, R 2 and R 3.
G

R1 R2 R3
Sol. Let x, y, z be the currents in the circuit as shown.
+ 10A 1A 0.1A
x x–y y Q
O P R

Sol. Between O and R :


z y
+
–14V x–y+z y–z If
G
R1 R2 R3
x y–z
I
In the left loop containing cell, O R

+ 14 – 1 (x – y) – 2 (x – y + z) = 0 Range = I = 0.1 A
In the top right loop (clockwise),  current through G is 10 mA when I = 0.1 A.
–2y – 1z + 1 (x – y) = 0
In the bottom right loop (clockwise),  R1  R 2  R 3 
 10 103  0.1  
1z – 1(y – z) + 2 (x – y + z) = 0  R1  R 2  R 3  G 
Simiplifying these equations we get :  R1 + R2 + R3 = 25/9  ...(i)
3x –3y + 2z = 14 Between O and Q :
x – 3y – z = 0
If
2x – 3y + 4z = 0 G
R1 R2 R3
Solving these equations, we get
x = 10 A; y = 4 A; z = – 2 A I
 the currents in the resistances are : O Q

i (in upper 1) = x – y = 6 A Range = I = 1 A


i (in upper 2) = y = 4 A
 R1  R 2 
i (in middle 1) = 2A (from left to right)  If  I  
 R1  R 2  R 3  G 
i (in lower 2) = x – y + z = 4 A
i (in lower 1) = y – z = 6 A  
 R  R 
Current in the middle 1 resistance is 2A from left to right.  10  103  1  1 2

 25
All the other currents are from top to the bottom.  25 
 9 
Total current supplied by the battery = x = 10 A
Energy supplied by the battery in one minute  25  1 5
 R1  R 2    25     ...(ii)
 9  100 18
= (Emf) × (current) × (60 s)
From (i) and (ii), we have :
Energy = 14 × 10 × 60 = 8400 J
CURRENT ELECTRICITY 42

R3 = 25/9 – 5/18 = 2.5  C


Between O and P : Sol. A

If
G
B
R1 R2 R3
D
I Let R be the equivalent resistance of the network between
O P A and B. The circuit starts repeating again to the right of
Range = I = 10 A first 3 resistances. Hence R is also the resistance to the
right of C and D.
 R1 
 If  I   3R
 C
 R1  R 2  R 3  G  R AB  2 
R 3 A

  R
 R1  3R
 10  103  10     R  2 B
R 3
 25  25  D
 9  R2 – 2R – 6 = 0

1  25  1
 R1    25   
1000  9
...(iii)  
R  1  7 . 
 36
Example - 70
From (ii) and (iii)
The given network is part of another larger circuit.
R1 = 0.0278; R2 = 0.25; R3 = 2.5 Calculate the potential of point D.
Example - 68 A (33 V)
A voltmeter has a resistance of 20000. When
connected in series with a large resistance R across
110 V line, the meter reads 5 V. Find the resistance R. (42 V) D
B O 3A
Sol. p.d. across voltmeter is 5 V.

110V
C (33 V)
R
V
Sol. Let the potential of point O be x Volts.
Rv 20000 going from D to O, we get :
 5  110   110
R  Rv R  20000 VO – VD = 6 × 3 = 18  VD = x – 18
 R = 420000 = 420 k . Let us assume that the current goes away from point
O to the points A, B, C and D through all branches.
Example - 69
 iOA  i OB  iOC  i OD  0
Find the resistance of the infinite network between
A and B in the figure. VO  VA VO  VB VO  VC VO  VD VO  VD
     0
3 4 5 6 1.2
A
x  33 x  42 x  53 18
   3 0
3 4 5 1.2
B
 x = 18 V
 D is at a potential of 0 Volts.
CURRENT ELECTRICITY 43

EXERCISE - 1 : BASIC OBJECTIVE QUESTIONS


Definition of Current 8. Ohm’s law fails in case of
–19
1. If the electronic charge is 1.6 × 10  C, then the number of (a) Thyristor (b) p-n junction crystal
electrons  passing  through    a  section  of  wire  per  second,
(c) Electrolytes (d) all of these
when the wire carries a current of 2 ampere is
17 17 Variation with length and area
(a) 1.25 × 10 (b) 1.6 × 10
19 19 9. All the edges of a block with parallel faces are unequal. Its
(c) 1.25 × 10 (d)1.6 × 10
longest  edge  is  twice  its  shortest  edge.  The  ratio  of  the
2. A non-conducting wire of radius R has charge q distributed maximum to minimum resistance between parallel faces is
unevenly over it. If it is rotated with an angular velocity of
(a) 2 (b) 4
, the equivalent current will be
(c) 8 (d) 16
(a) zero (b) q
10. Two plates of same material R and S are in the form of a
q q square and have the same thickness. A side of S is twice the
(c)  (d) 
2 2 R side of R. Compare their resistances. The direction of current
is shown by an arrow head.
3. The belt of an electrostatic generator is 50 cm wide and
travels at 30 cm/s .The belt carries charge into the sphere
at a rate which corresponds to 10–4 ampere. What is surface
R S
density of charge on the belt.
(a) 6.7 × 10–5 Cm–2/s (b) 6.7 × 10–4 Cm–2/s
(c) 6.7 × 10–8 Cm–2/s (d) 6.7 × 10–7 Cm–2/s (a) The resistance of R is twice that of S
Mobility of Charge carriers (b) Both have the same resistance
4. A potential difference V is applied to a copper wire of length
(c) The resistance of S is four times that of R
l and thickness d. If V is doubled, the drift velocity.
(a) is doubled (b) is halved (d) The resistance of R is half to that of S.

(c) remain same (d) becomes zero 11. 1 kg piece of copper is drawn into a wire 1 mm thick, and


another piece into a wire 2 mm thick. Compare the resistance
5. A copper wire of length 1 m and radius 1 mm is joined in
of these wires.
series with an iron wire of length 2 m and radius 3 mm and a
current is passed through the wires. The ratio of the current (a) 2 : 1 (b) 4 : 1
density in the copper and iron wires is
(c) 8 : 1 (d) 16 : 1
(a) 2 : 3 (b) 6 : 1
12. The resistance of a 10 m long wire is 10 . Its length is
(c) 9 : 1 (d) 18 : 1 increased  by  25%  by  stretching  the  wire  uniformly.  The
Microscopic form of Ohm’s law resistance of wire will change to (approximately)

6. Counductivity of a conductor depends upon (a) 12.5  (b) 14.5 


(a) length (b) area of cross section (c) 15.6  (d) 16.6 
(c) volume (d) temperature 13. Masses of the three wires of same material are in the ratio of
7. If temperature is decreased, then relaxation time of electrons 1 : 2 : 3 and their lengths in the ratio of 3 : 2 : 1. Electrical
in metals will resistance of these wires will be in the ratio of

(a) increase (b) decrease (a) 1 : 1 : 1 (b) 1 : 2 : 3


(c) fluctuate (d) remain constant (c) 9 : 4 : 1 (d) 27 : 8 : 1
CURRENT ELECTRICITY 44

14. The length of conductor is doubled and its radius is halved, (a) 40oC (b)   50oC


its specific resistance is (c) 60oC (d) 70oC
(a)  unchanged (b) halved 23. On increasing the temperature of a conductor, its resistance
(c) doubled (d) quadrupled increases  because
15. A resistance of 2  is to be made from a copper wire (specific (a) relaxation time decreases
–8
resistance 1.7 × 10   m) using a wire of length 50 cm. The
(b) mass of the electron increases
radius of the wire is
(c) electron density decreases
(a) 0.0116 mm (b) 0.0367 mm
(d) none of the above
(c) 0.116 mm (d) 0.367 mm
3 24. Thermistors are usually made of
16. The specific resistance of a wire is , its volume is 3 m  and
its resistance is 3 ohm, then its length will be (a) metals with low temperature coefficient of resistivity
(b) metals with high temperature coefficient of resistivity
(a)  1/  (b)  3 / 
(c)  metal  oxides  with  high  temperature  coefficient  of
(c)  3 /  (d)   / 3 resistivity
(d)  semiconductors  with  low  temperature  coefficient  of
17. If  an  increases  in  length  of  copper  wire  is  0.5%  due  to
stretching, the percentage increase in its resistance will be resistivity

(a) 0.1% (b) 0.2% 25. A conductor behaves as a super conductor

(c) 1% (d) 2% (a) above critical temperature


2
18. A nichrome wire 1 m long and 1 mm  in cross-section area (b) at critical temperature
draws 4 ampere at 2 volt. The resistivity of nichrome is (c) at 40 C°
–7 –7
(a) 1 × 10  –m (b) 2 × 10  –m (d) at boiling point of that metal
–7 –7
(c) 4 × 10  –m (d) 5 × 10  –m 26. The  temperature  coefficient  of  resistance  for  a  wire  is
–1
Variation with Temperature 0.00125°C . At 300 K its resistance is 1 ohm. The temperature
at which the resistance becomes 1.5 ohm is
19. Which of the following material has negative temperature
coefficient of resistance. (a) 450 K (b) 727 K
(a) brass (b)  copper (c) 454 K (d) 900 K
(c)  porcelain (d)  carbon 27. One end of copper wire of length 2L and cross sectional
20. Which of the following material has nearly zero temperature area A is attached to one end of another  copper  wire  of
coefficient of resistance. length L and cross section area 2A. If the free end of the
(a) carbon (b) porcelain longer wire is at electric potential of 8 volt and free end of
longer wire is at potential of 1 volt ,what is the potential of
(c) copper (d)  manganin
junction of two wires.
21. A  piece  of  copper  and  silicon  are  cooled  from  room
temperature to 100 K. The resistance of (a)1.2 V (b)  2.1 V

(a) each of them increases (c) 2.4 V (d)  3.6 V


28. As the temperature of a conductor increases, its resistivity
(b) each of them decreases
and  conductivity  changes  .The  ratio  of  resistivity  to
(c) copper increases and that of silicon decreases conductivity
(d) copper decreases and that of silicon increases (a) Increases
22. A coil has resistance of 18   when  its mean temperature (b)  decreases
is 20oC and of 20   when its mean temperature is 50 oC.
What will be mean temperature rise when its resistance is (c) remains constant
21  and the surrounding temperature is 15oC. (d) may increase or decrease depending on material
CURRENT ELECTRICITY 45

Series and Parallel of Resistors 35. In the circuit diagram shown below each resistance is of


29. A wire of resistance 6  is cut into three equal pieces, which value 1 ohm, what is the equivalent resistance between the
are  joined  to  form  a  triangle.  The  equivalent  resistance points A and B.
between any two corners of the triangle is
(a) (3/4)  (b) (4/3) 
(c) 2  (d) 4  A B
30. Two resistances are joined in parallel whose resistance is
3/5 . One of the resistance wire is broken and the effective
resistance becomes 3 . The resistance in ohm of the wire (a) 4/7  (b) 8/7 
that got broken was (c) 3/14  (d) 16/7 
(a) 4/3 (b) 2 36. The resistance of all the wires between any two adjacent
dots is R. The equivalent resistance between A and B as
(c) 6/5 (d) 3/4
shown is
31. You are given three equal resistors. How many resistances
can  be  obtained  by  joining  them  in  series  and  parallel A
grouping ?
(a) 6 (b) 4
(c) 3 (d) 2
32. A wire of resistance R is cut into n equal parts. These parts
B
are then connected in parallel. The equivalent resistance of
the combination will be
(a) 7R/13 (b) 7R/6
(a) nR (b) R/n
2
(c) 14R/8 (d) 15R/7
(c) n/R (d) R/n .
37. What  is  the  equivalent  resistance  between  the  points A
33. What will be the resistance between P and Q in the following and B of the circuit shown in figure.
circuit ?

P 5 10
5
A B
10 5

Q
(a) 3 ohm (b) 5 ohm
(a) (1/3)  (b) (2/3)  (c) 7 ohm (d) 15 ohm

(c) 2  (d) 5  38. The effective resistance between points A and B in the circuit


shown in figure is
34. In the figure, find the resistance between points A and B.
Both the circle and the diameter are made of uniform wire of
–4
resistance 1 × 10  ohm per metre. The length AB is 2 meter. R
R R
A R B

A B

–4
(a) (2/3) × 10   (b) 2 × 10  
–4 (a) R/3 (b) R/2
–4
(c) 14.56 × 10   (d) 0.88 × 10  
–4 (c) 2 R/5 (d) 3 R/5
CURRENT ELECTRICITY 46

39. What is the equivalent resistance between A and B in the 43. The  electrical  resistance  between  points A  and  B  of  the
circuit of figure, if R = 3 , figure shown is

A B
R R A B

R
C D
(a) (2/3)  (b) 2 
R R
(c) (3/2)  (d) 6 
Heating effect

(a)  8  (b) 9  44. A  100watt,110  volt  and  a  50  watt,110  volt  lamps  are
connected  in  series  across  220  volt  d.c.  source.  If  the
(c) 12  (d) 15  resistances of two lamps are assumed to remain constant,
40. The effective resistance between A and B in figure is the voltage across 100 watt lamps is
(a) 100 volt (b) 143.3 volt
(c)  73.3 volt (d)  200 volt
45. Three  resistances are connected in series across 12 volt
battery.The first resistor has value of 1  ohm second has
A B voltage drop of 4 volt and third has power dissipation of
12 watt. What is circuit current.
(a)  1A (b) 2A
(a) 10  (b) 12 
(c)   3A (d) 4A
(c) 9.85  (d) 10.85 
Wheatstone Bridge
41. What is the total resistance of the circuit ?
46. What  is the  equivalent resistance  of  the  network  across
4V points A and B shown in figure below.
+ –
A
A B

(a) 4 ohm (b)  36/7 ohm

A (c) 23/7 ohm (d)  12/7 ohm
47. The Wheatstone’s bridge  shown in figure is balanced.  If
(a) 6  (b) 7  the  position  of  the  cell  E  and  galvanometer  G  are  now
(c) 8  (d) 9  interchanged, G will show zero deflection
A
42. A uniform wire of resistance 36  is bent in the form of a
circle. The effective resistance across the points A and B is R4 R1

I E D G B
A

30°
R3 R2
O B I C
(a) only if all resistor are equal
(b) only if R1 = R3 and R2 = R4
(a) 36  (b) 18  (c) only if R1R4 = R2R3
(c) 9  (d) 2.75  (d) in all cases
CURRENT ELECTRICITY 47

48. In an experiment on Wheatstone’s bridge, the position of 52. A resistance of 2  is connected across one gap of a meter


the cell and galvanometer are interchanged once balance bridge,  the  length  of  wire  is  1  meter,  and  an  unknown
point is obtained then resistance, greater than 2  is connected across the other
(a) balance point shifts gap. When these resistances are interchanged, the balance
point shifts by 20 cm. Neglecting any correction, find the
(b) balance point remains unchanged
unknown  resistance.
(c) balance point depends on internal resistance of cell.
(a) 3  (b) 4 
(d) All of these
(c) 5  (d) 6 
49. AB is a wire  of uniform resistance. The galvanometer G
53. A wire connected in the left gap of a meter bridge balance
shows no current when the length AC = 20 cm and CB = 80
a 10 in the right gap at a point which divides the bridge
cm. The resistance R is equal to
wire in the ratio of 3:2.What is the resistance of the wire
R (a) 10 (b) 1.2
(c) 15 (d) 1
B 54. Consider  the  following statements regarding  the  network
shown in the figure
A B
C R R
A B
R'
(a) 2  (b) 8  2R 2R

(c) 20  (d) 40  E

50. Determine the value of resistance R if the current in branch
OA is zero. (i) The  equivalent  resistance  of  the  network  between A
and B is independent of the value of R’
(ii) The equivalent resistance of the network between points
A and B is 4 R/3
(iii) The current flowing through R’ is zero
Which of the above statement(s) is/are correct ?
R
(a) (i) alone
(b) (ii) alone
(c) (ii) and (iii)
(a) 6  (b) 8  (d) (i), (ii) and (iii)
(c) 10  (d) 12  55. In the circuit shown, some potential difference is applied
51. In meter bridge or wheatstone bridge for measurement of between A & B. The equivalent resistance between A & B is
resistance,  the  known  and  the  unknown  resistances  are
interchanged. The error so removed is
(a) end correction A B

(b) index error
(c) due to temperature effect
(a) 14  (b) 12.5 
(d) random error
(c) 3.6  (d) 2.1 
CURRENT ELECTRICITY 48

Kirchhoff’s Laws (a) 2 : 1 : 6 (b) 3 : 2 : 1


56. The figure shows currents in a part of an electric circuit, (c) 2 : 3 : 6 (d) 1 : 2 : 3
then current I is 60. What is the current I in the circuit as shown in figure

2A
1A I
1.3A
3V
2A
I I

(a) 1.7 A (b) 3.7 A
(a) 2A (b) 1.2 A
(c) 1.3 A (d) 1 A
(c) 1 A (d) 0.5 A
57. The figure shows a network of currents. The magnitude of
currents is shown here. The current I will be 61. Total current supplied to the circuit by the battery is

12A
3A

6V 2
8A

I 1.5 3 6
5A

(a) – 3A (b) 3A
(c) 13 A (d) 20 A (a) 4A (b) 6A
58. In  the  figure  shown  below,  the  electric  current  flowing (c) 2A (d) 1A
through 2R resistor is 62. In the circuit shown in figure potential difference between
X and Y will be
B x y
A D
R 2R C R

+ –

120V

(a) from left to right (a) zero (b) 20 V


(b) from right to left (c) 60 V (d) 120 V
(c) no current flows 63. For the simple ladder network ,find input voltage which
produces  a  current  of  0.25 A  in  3  ohm  resistor. All  the
(d) double to that through any other resistors
resistors are in ohm.
59. In the given circuit RB = 6, RC = 3, RD = 1. The ratio of
current through resistors C, B and D will be in the ratio
A
7 5 1 0.25A
RB
8 6 3

RA RC RD 9 2
B

E (a) 20 volt (b) 15 volt
(c)   26 volt (d) 12 volt
CURRENT ELECTRICITY 49

64. The potential difference between points A and B in the circuit 68. In the circuit as shown in figure, the current in ammeter is


shown in figure will be
3V

A 10V,

I A

B 5V, 6V

(a) 1 V (b) 2 V 42 27
(a)  A (b)  A
32 32
(c) 3 V (d) 4 V
15 3
65. Electric current is passed through a circuit containing two (c)  A (d)  A
32 10
wires of the same material, connected in parallel. If the lengths
and radii are in the ratio of 4/3 and 2/3 respectively, then the Potential difference across cell
ratio of current passing through the wires will be 69. A battery is charged at a potential of 15 volt for 8 hours
when  current  flowing  is  10 A.The  battery  on  discharge
(a) 8/9 (b) 1/3
supplies a current of 5A for 15 hours .The mean terminal
(c) 3 (d) 2 voltage  during  discharge  is  14  volt.  What  is  watt  hour
66. What is current through 3 ohm resistor(BD branch) in the efficiency of the battery.
shown electric circuit ? (a) 50% (b) 67.8%
(c) 89.6% (d) 87.5%
70. In  an  electric  circuit  consisting  of  a  battery,  the
charge(assumed positive) inside the battery
(a) Always move from positive to negative terminal
(b) may move from negative to positive terminal
(c) Always move from negative to positive terminal
(d) does not move at all
(a) 1.23 A (b) 2.27 A
71. If a  variable resistance is connected to  a cell  of constant
(c) 4.3 A (d) 3.46 A e.m.f., then which one of the following graphs represents
67. Find out the value of current through 2 resistance for the the relationship between current, I and resistance, R ?
given circuit.
I I

(a)  (b) 
R R
O O

I I

(a) zero (b) 2A
(c) 5A (d) 1 A (c)  (d) 
R R
O O
CURRENT ELECTRICITY 50

72. The potential difference across the terminals of a battery is 78. Two  identical  cells  connected  in  series  send  1.0  amp  of
8.4 volt when there is current of 1.50 A in the battery from current  through  a  5  ohm  resistance.When  they  are
negative to positive terminal .When the current is 3.5 A in connected in parallel, they send current of 0.8 amp through
reverse direction ,the potential difference becomes 9.4 volt the same resistor. What is the internal resistance of the cell ?
.The internal resistance and e.m.f. of battery are (a) 0.5  (b) 1
(a) 0.2 ohm, 8.7 V (b) 0.4 ohm, 6.7 V (c) 2.5  (d) 4
(c) 0.5 ohm, 9.5 V (d) 0.2 ohm, 10.4 V 79. Two  real  batteries  are  connected  in  series.  Consider  the
73. An idealized voltmeter is connected across the terminals of following statements (i) The equivalent e.m.f. is larger than
a 15 volt battery and a 7.5 ohm appliance is also connected either of two emf’s (ii) The equivalent internal resistance is
across its terminals .If the voltmeter reads 12.5 volt .What smaller than either of two internal resistances.
is the internal resistance of the battery. (a) Both statements are correct
(a) 1.0  (b) 1.2  (b) Statement (i) is correct but statement (ii) is wrong
(c) 1.4  (d) 1.5  (c) Statement (ii) is correct but statement (i) is wrong
74. A cell of negligible resistance and e.m.f. 2 volt is connected (d) Both are wrong statements
to  series  combination  of  2,  3  and  5  ohm.  The  potential 80. How will you connect 24 cells each of internal resistance of
difference in volt across the 3 ohm resistance will be 1so as to get maximum power output across a load of
(a) 6 V (b) 3 V 1.50 

(c) 2/3 V (d) 0.6 V (a) 4 rows, 6 cells in a row


(b) 12 rows, two cells in a row
75. For a cell terminal potential difference is 2.2 volt when circuit
is open and reduces to 1.8 volt when cell is connected to an (c) 2 rows, 12 cells in a row
external resistance of 5 ohm.What is the internal resistance (d) 3 rows, 8 cells in a row
of the cell ? 81. To draw a maximum current from a combination of cells, how
should the cells be grouped ?
 10   10 
(a)     (b)     (a) Parallel
9  19 
(b) Series
1 5
(c)     (d)     (c) Mixed grouping
9 9
(d) Depends upon the relative values of internal and external
76. A cell of internal resistance r is connected across an external       resistances
resistance nr.Then the ratio of terminal voltage to the e.m.f.
82. The heating element of an electric heater should be made
of the cell is
with a material which should have
1 1 (a) High specific resistance and high melting point
(a)  (b)   n  1
n (b) High specific resistance and low melting point
(c) Low specific resistance and high melting point
 n  1 n
(c)  (d)  (d) Low specific resistance and low melting point
n n 1
83. 50 electric bulbs are connected in series across 200 volt
Series and Parallel of Cells
supply and illumination produced is I1. 5 bulbs get fused. If
77. Five  cells  each  of  e.m.f.  E  and  internal  resistance  r  are
the remaining bulbs are again connected across the source
connected  in  series.  If  due  to  oversight  one  cell  gets
in  series,  the  illumination  produced  is  I2.Which  of  the
connected wrongly ,then the equivalent e.m.f. and internal
following is true.
resistance of this combination is
(a) I1= I2 (b) I1> I2
(a) 5E,5r (b) 4E,4r
(c) 3E,4r (d) 3E,5r (c) I1< I2 (d) anything is possible
CURRENT ELECTRICITY 51

84. A coil takes 15  minutes to boil certain amount of water, 92. In potentiometer a balance point is obtained, when


another coil takes 20 minutes for the same process .Time
(a) the e.m.f. of the battery becomes equal to the e.m.f. of
taken to boil the same amount of water when both the coils
are connected in series across same source is the experimental cell

(a) 5 min (b) 8.6 min (b) the  p.d.  of  the  wire  between  the  +ve  end  to  jockey
(c) 35 min (d)12 min becomes equal to the e.m.f of the experimental cell
85. If two bulbs of wattage 25 and 30 watt each rated at 220 volt (c) the  p.d.  of  the  wire  between  +ve  point  and  jockey
are connected in series with 440 volt supply. Which bulb becomes equal to the e.m.f. of the battery
will fuse ?
(d) the p.d. across the potentiometer wire becomes equal to
(a) 25 watt bulb (b) 30 watt bulb the e.m.f. of the battery.
(c) neither of them (d) both of them
93. If the resistivity of a potentiometer wire be  and area of
Joule’s Heating Effect cross  section  be  A.  If  I  is  the  current  through  the
86. A 200 volt,1000 watt bulb is connected across 100 volt main potentiometer wire then what will be the potential gradient
supply.What will be power consumed. along the wire ?
(a) 200 watt (b) 250 watt
(a) 1/A (b) I/A
(c) 500 watt (d) 750 watt
(c) IA/ (d) IA
87. When three identical bulbs of 60 watt, 200 volt rating are
connected in series to a 200 volt supply, the power drawn 94. If the radius of a potentiometer wire is increased four times,
by them keeping its length constant then the value of its potential
(a) 60 watt (b)180 watt gradient will become
(c) 10 watt (d) 20 watt (a) half (b) two times
Potentiometer (c) four times (d)  unchanged
88. In a potentiometer experiment,  there is no current at the
95. 125 cm of potentiometer wire balances the e.m.f. of a cell and
balance point in
100 cm of the wire is required for balance the poles of a cell
(a) main battery circuit
which are joined by a 2  resistors. The internal resistance
(b) galvanometer circuit of a cell is
(c) potentiometer circuit
(a) 0.25  (b) 0.50 
(d) both main and galvanometer circuit
(c) 0.75  (d) 1.25 
89. The length of a potentiometer wire is 5 metres. An electron
–19
isthiswireexperiencesaforceof 4.8 × 10  newton, e.m.f. of 96. In the following circuit figure, the resistance of wire AB is
the main cell used in potentiometer is 10  and its length is 1m. Rest of the quantities are shown in
(a) 3 volt (b) 15 volt the figure. The potential gradient on potentiometer wire will be
(c) 1.5 volt (d) 5 volt 2V
90. In a potentiometer, experiment, the balancing length is at
240 cm with a cell. On shunting the cell with a resistance of
2  the  balancing  length  becomes  120  cm  .What  is  the
internal resistance of the cell. J
A B
(a) 0.5  (b) 1
(c) 2 (d) 4 G
91. The length of potentiometer wire is l. A cell of e.m.f. E is
balanced at the length l/3 from the positive end of the wire.
If the length of the wire is increased by l/2, at what distance 1.5V
will the same cell give a balance point.
(a)  l/2 (b)   l/3 (a) 0.8 V/m (b) 0.08 V/m
(c)  l/4 (d)   l/5 (c) 0.008 V/m (d) none of the above
CURRENT ELECTRICITY 52

97. In the above problem, the length of the wire AJ at which null 103. What is the final charge on the capacitor in steady state in


points is obtained, will be the above question ?
(a) 3.75 cm (b) 37.5 cm
1 2
(a)  CE (b)  CE
(c) 75 cm (d) 27.5 cm 3 3
98. The current in the primary circuit of a potentiometer wire is
–7
0.5A, specific resistanceof wire is4 × 10  –m and area of 3
(c) CE (d)  CE
–6 2
cross-section of wire is 8 × 10  m . The potential gradient in 2
the wire would be 104. A resistor with resistance of 10 M and a capacitor with
(a) 2.5 mV/metre (b) 25 mV/metre capacitance 1F are connected in series as shown with a
switch .The capacitor is given initial charge of 5C and
(c) 25 V/metre (d) 10 V/metre
then discharged by closing the switch at t = 0. After what
99. A 4 m long wire of resistance 8  is connected in series with
time charge will be equal to 0.5C.
a battery of e.m.f. 2 V and a resistor of 7 . The internal
resistance of the battery is 1 . What is the potential gradient
along the wire ?
–1 –1
S C
(a) 1.00 Vm (b) 0.75 Vm
–1 –1
(c) 0.50 Vm (d) 0.25 Vm R
100. Two resistors of value 1 K   and 4 K are connected in
series  across  a  constant  voltage  supply  of  100  volt. A (a) 12 s (b) 23 s
voltmeter has internal resistance of 12 K   voltmeter is
(c) 45 s (d) 0.14 s
connected across 4 K  resistor. What is percentage error
in voltmeter reading. 105. The circuit is shown in figure, the cell is ideal with e.m.f. of
(a) –12.5% (b)  –3.25% 15 volt each resistance R =3 ohm .What is the potential
difference across the capacitor.
(c)  –6.25% (d)  +4.5%
101. The infinity resistance plug in a post office box (resistance C
box) has
(a) an air gap only R
R R
(b) a resistance coil of infinite resistance
(c) largest resistance value in box
R R
(d) resistance of value –100 ohm
102. In the given R and C circuit what is final steady state current ?
15V

2R (a) 2 volt (b) 4 volt
C
(c) 12 volt (d) zero
2R Conversion of Galvanometer
R E
106. Mark the correct option.
(a) an ammeter should have small resistance

E E (b) an ammeter should have high resistance
(a)  (b) 
2R 3R (c) a voltmeter has low resistance

E E (d) galvanometer has low resistance than a voltmeter
(c)  (d) 
R 5R
CURRENT ELECTRICITY 53

107. A galvanometer of coil resistance 20 gives a full scale (a) The reading of both the ammeter and voltmeter remains


deflection with a current of 5 mA. What arrangement should constant
be made in order to measure current upto 1.0 A. (b) Reading of both ammeter and voltmeter will increase
(a) add a series resistance of 2 (c)  Reading  of  ammeter  remains  constant  and  that  of
(b) add a parallel resistance of 2 voltmeter increases
(c) add a series resistance of 0 (d)  Reading  of  ammeter  remains  constant  and  that  of
(d) add a parallel resistance of 0.1 voltmeter decreases

108. The net resistance of an ammeter should be small to ensure 111. A candidate connects a moving coil voltmeter V, a moving


coil ammeter A and a resistor R as shown in figure. If the
(a) it does not get overheated
voltmeter  reads  20  volt  and  the  ammeter  reads  4 A,  the
(b) it does not draw excessive current value of R is
(c) it does not appreciably change the current to measure
(d) it can measure large current
V
109. The deflection of galvanometer decreases from 25 divisions +
to 5 divisions when a resistor of 20 is connected in series
.What is galvanometer resistance. R
+ A
(a) 4 (b) 5
(c) 6 (d) 7
110. In the given circuit, the sliding contact C is moved from A (a) equal to 5 ohm
to B (b) greater than 5 ohm
(c) less than 5 ohm
V
(d) may be greater or less than 5 ohm
C
A B

A
CURRENT ELECTRICITY 54

EXERCISE - 2 : PREVIOUS YEAR JEE MAINS QUESTIONS

1. In  the  given  circuit  all  resistances  are  of  value  R  ohm 4. A  constant  voltage  is  applied  between  two  ends  of  a
each. The equivalent resistance between A and B is : metallic wire. If the length is halved and the radius of the
(2018 Online Set-1) wire is doubled, the rate of heat developed in the wire will
be : (2018 Online Set-2)
(a) Doubled (b) Halved
(c) Unchanged (d) Increased 8 times
5. A  heating  element  has  a  resistance  of  100    at  room
temperature. When it is connected to a supply of 220 V, a
steady current of 2 A passes in it and temperature is 5000C
(a) 2R (b) 3R more  than  room  temperature.  What  is  the  temperature
coefficient of resistance of the heating element ?
5R 5R (2018 Online Set-3)
(c) (d) 
3 2
(a) 0.5×10-4 0 C  -1 (b)  5×10-4 0 C  -1
2. In a meter bridge, as shown in the figure, it is given that
resistance Y =12.5   and that the balance is obtained at a (c) 1×10-4 0 C  -1 (d)  2×10-4 0 C  -1
distance  39.5  cm  from  end  A  (by  Jockey  J).  After
6. A galvanometer with its coil resistance  25   requires a
interchanging the resistances X and Y, a new balance point
current of 1 mA for its full deflection. In order to construct
is found at a distance   2 from end A. What are the values an ammeter to read up to a current of 2 A, the approximate
of X and   2 ? (2018 Online Set-1) value of the shunt resistance should be :
(2018 Online Set-3)
(a) 2.5×10-3  (b) 1.25×10-2 

(c) 1.25×10-3  (d)  2.5×10-2 


7. A power transmission line feeds input power at 2300 V to
a step down transformer with its primary windings having
4000 turns, giving the output power at 230 V. If the current
in the primary of the transformer is 5 A, and its efficiency
is 90%, the output current would be :
(2018 Online Set-3)
(a) 50 A (b) 45 A
(a) 8.16  and 60.5 cm (c) 25 A (d) 20 A
(b) 19.15 and 39.5 cm 8. In a circuit for finding the resistance of a galvanometer by
(c) 8.16 and 39.5 cm half deflection method, a 6V battery and a high resistance
(d) 19.15 and 60.5 cm of 11k   are used. The figure of merit of the galvanometer
3. A copper rod of cross-sectional area A carries a uniform is 60   /division. In the absence of shunt resistance, the
current I through it. At temperature T, if the volume charge galvanometer  produces  a  deflection  of   = 9 divisions
density of the rod is  ,  how long will the charges take to when current flows in the circuit. The value of the shunt
travel a distance d ? (2018 Online Set-2) 
resistance that can cause the deflection of , is closest
2 d A 2 d A 2
(1) (2)  I T to : (2018 Online Set-3)
I
(a) 550  (b)  220 
d A d A
(3) (4)  I T (c) 55 (d) 110 
I
CURRENT ELECTRICITY 55

9. A 9 V battery with internal resistance of 0.5 is connected 11. A uniform wire of length l and radius r has resistance of


across  an  infinite  network  as  shown  in  the  figure. All
r
ammeters A1, A2, A3 and voltmeter V are ideal. 100. It is recast into a wire of radius  .  The resistance of
2
(2017 Online Set-1)
new wire will be : (2017 Online set-2)
(a) 1600  (b) 400 
(c) 200  (d) 100 
12. The  figure  shows  three  circuits  I,  II  and  III  which  are
connected to a 3V battery. If the powers dissipated by the
configurations I, II and III and P1, P2 and P3 respectively,
then : (2017 Online Set-2)
Choose correct statement.
(a) Reading of A1 is 2A
(b) Reading of A1 is 18 A
(c) Reading of V is 9 V
(d) Reading of V is 7V
10. A potentiometer PQ is set up to compare two resistances
as shown in the figure. The ammeter A in the circuit reads (a)  P1  P2  P3 (b)  P1  P3  P2
1.0 A when two way key K3 is open. The balance point is
at a length l1 cm from P when two way key K3 is plugged in (c)  P1  P2  P3 (d)  P3  P2  P1
between 2 and 1, while the balance point is at a length l2 13. In  the  circuit  shown,  the  resistance  r  is  a  variable
cm from P when key K3 is plugged in between 3 & 1. The resistance. If for r = f R, the heat generation in r is maximum
R1 then the value of f is : (2016 Online Set-1)
ratio of two resistances  ,  is found to be :
R2

(2017 Online Set-1)

1 1
(a) (b) 
4 2
3
(c) (d) 1
4
14. A  50  &!  resistance  is  connected  to  a  battery  of  5  V. A
galvanometer of  resistance 100  &!   is to be used as an
ammeter  to  measure  current  through  the resistance,  for
this a resistance rs is connected to the galvanometer. Which
1 2 of the following connections should be employed if the
(a)     (b)     measured current is within 1% of the current without the
1 2 2 1
ammeter in the circuit? (2016 Online Set-1)
1 1 (a) rs = 0.5 in parallel with the galvanometer
(c)     (d)    
1 2 2 1 (b) rs = 0.5 in series with the galvanometer
(c) rs = 1  in series with galvanometer
(d) rs = 1  in parallel with galvanometer
CURRENT ELECTRICITY 56

15. To  know  the  resistance  G  of  a  galvanometer  by  half 20. In the experiment of calibration of voltmeter, a standard
deflection method, a battery of emf VE and resistance R is cell  of  e.m.f  1.1  volt  is  balanced  against  440  cm  of
used to deflect the galvanometer by angle θ. If a shunt of potentiometer  wire.  The  potential  difference  across  the
resistance S is needed to get half deflection then G,  R and ends of resistance is found to balance against 220 cm of
S are related by the equation: (2016 Online Set-1) wire. The corresponding reading of voltmeter is 0.5 volt.
(a) 2S (R + G) = RG (b) S (R + G) = RG The error in the reading of voltmeter will be:
(2014 Online Set-2)
(c) 2S = G (d) 2G = S
(a) – 0.15 volt (b) 0.15 volt
16. The resistance of an electrical toaster has a temperature
(c) – 0.05 volt (d) 0.5 volt
dependence given by R(T) = R0 [1 + (T  T0 ) ] in its range 21. The circuit shown here has two batteries of 8.0 V and 16.0
of operation. At  T0  300K, R  100  and at T = 500 K, V and three resistors 3, 9 and 9 a capacitor 5.0 F.

R = 120 .  The toaster is connected to a voltage source at
200 V and its temperature is raised at a constant rate from
300 to 500 K in 30 s. The total work done in raising the
temperature is : (2016 Online Set-2)

6 2
(a) 60000 l n   J (b)  200 ln J
5 3

5
(c) 400 ln J (d) 300 J
6
How much is the current I in the circuit in steady state?
17. A galvanometer having a coil resistance of 100  gives a
(2014 Online Set-2)
full  scale  deflection  when  a  current  of  1  mA  is  passed
through it. The value of the resistance which can convert (a) 0.67 A (b) 1.6 A
this galvanometer into ammeter giving full scale deflection (c) 0.25 A (d) 2.5 A
for a current of 10 A is: (2016 Online Set-2)
22. In the circuit shown, current (in A) through the 50 V and
(a) 0.1  (b) 0.01 
30 V batteries are, respectively: (2014 Online Set-3)
(c) 100  (d) 0.001 
18. In  the  electric  network  shown,  when  no  current  flows
through  the  4  resistor  in  the  arm  EB,  the  potential
difference between the points A and D will be :
(2015 Online)

(a) 4.5 and 1 (b) 2.5 and 3
(c) 3 and 2.5 (d) 3.5 and 2
23. Four bulls B1, B2, B3 and B4 of 100 W each are connected
to 220 V main as shown in the figure. The reading in an
(a) 6 V (b) 3 V ideal ammeter will be : (2014 Online Set-4)

(c) 5 V (d) 4 V
19. A d.c. main supply of e.m.f. 220 V is connected across a
storage battery of e.m.f 200 V through a resistance of 1.
The  battery  terminals  are  connected  to  an  external
resistance ‘R’. The minimum value of ‘R’, so that a current
passes through the battery to charge it is:
(2014 Online Set-1)
(a) 7 (b) 9 (a) 0.90 A (b) 1.80 A
(c) 11 (d) Zero (c)  1.35 A (d) 0.45 A
CURRENT ELECTRICITY 57

24. In a meter bridge experiment resistances are connected as 28. In the below circuit, the current in each resistance is:


shown in the figure. Initially resistance P = 4 and the (2017)
neutral  point  N  is  at  60  cm  from A.  Now  an  unknown
resistance  R  is  connected  in  series  to  P  and  the  new
position of the neutral point is at 80 cm from A. The value
of unknown resistance R is : (2017)

(a) 1A (b) 0.25A
(c) 0.5 A (d) 0A
29. When a current of 5 mA is passed through a galvanometer
having  a  coil  of  resistance 15 ,  it  shows  full  scale
deflection. The value of the resistance to be put in series
33
(a)   (b)  6 with  the  galvanometer  to convert  it  into  a  voltmeter  of
5
range0-10 V is: (2017)
20
(c)  7 (d)   (a) 1.985  103  (b)  2.045 103 
3

25 Two batteries with e.m.f 12V and 13V are connected in (c)  2.535 103  (d)  4.005 103 


parallel  across  a  load  resistor  of  10.  The  internal
30. Which of the following statements is false? (2017)
resistance of the two batteries are 1 and 2 respectively.
(a) Wheatstone bridge is the most sensitive when all the
The voltage across the load lie between: (2018)
four resistances are of the same order of magnitude
(a) 11.4 V and11.5 V
(b)  In a  balanced Wheatstone bridge  if the cell and the
(b) 11.7 V and11.8 V
galvanometer are exchanged, the null point is disturbed
(c) 11.6 V and 11.7 V
(c) A rheostat can be used as a potential divider
(d) 11.5 V and 11.6 V
(d) Kirchhoff’s second law represents energy conservation
26. In a potentiometer experiment, it is found that no current
31. A galvanometer having a coil resistance of 100  gives a
passes  through  the  galvanometer  when  theterminals  of
full scale deflection, when a current of 1  mA is passed
the cell are connected across 52cm of the potentiometer
through it. The value of the resistance, which can convert
wire. If the cell is shunted by resistance of 5, a balance is
this galvanometer into ammeter giving a full scale deflection
found when the cell is connected across 40 cm of the wire.
for a current of 10 A, is : (2016)
Find the internal resistance of the cell. (2018)
(a) 2  (b) 0.1
(a) 2 (b) 2.5
(c) 3  (d) 0.01 
(c) 1 (d) 1.5
32. The  temperature  dependence  of  resistance  of  Cu  and
27. On interchanging the resistances, the balance point of a undoped Si in the temperature range 300–400 K, is best
meter bridge shifts to the left by 10 cm. The resistance of described by : (2016)
their  series  combination  is  1K.  How  much  was  the
(a) Linear increase for Cu, exponential increase for Si.
resistance  on  the  left  slot  before  interchanging  the
(b) Linear increase for Cu, exponential decrease for Si.
resistances  ? (2018)
(c) Linear decrease for Cu, linear decrease for Si.
(a) 500 (b) 910
(d) Linear increase for Cu, linear increase for Si
(c) 990 (d) 505
CURRENT ELECTRICITY 58

33. When 5V potential difference is applied across a wire of Statement-I : Higher the range, greater is the resistance of


length 0.1 m, the drift speed of electrons is 2.5 × 10–4 ms–1. ammeter.
If  the  electron  density  in  the  wire  is  8  ×  1028  m–3,  the Statement-II : To increase the range of ammeter, additional
resistivity of the material is close to : (2015) shunt needs to be used across it. (AIEEE 2013)
(a) 1.6 × 10–6 m (b) 1.6 × 10–5 m (a) If Statement-I is true, Statement-II is true; Statement-II
(c) 1.6 × 10–8 m (d) 1.6 × 10–7 m is the correct explanation for Statement-I.

34. In the circuit shown, the current in the 1 resistor is : (b) If Statement-I is true, Statement-II is true; Statement-II


is not a correct explanation for Statement-I.
(c) If Statement-I is true; Statement-II is false.
(d) If Statement-I if false; Statement-Ii is true.
38. The supply voltage in a room is 120 V. The resistance of
the lead wires is 6. A 60W bulb is already switched on.
What is the decrease of voltage across the bulb, when a
240 W heater is switched on in parallel to the bulb ? (2013)
(a) 0.13 A, from Q to P (2015) (a) zero (b) 2.9 V
(b) 0.13 A, from P to Q (c) 13.3 V (d) 10.04 V
(c) 1.3 A, from P to Q 39. Two electric bulbs marked 25 W–220 V and 100 W-220 V are
connected in series to a 440 V supply. Which of the bulbs
(d) 0A
will fuse ? (AIEEE 2012)
35. A conductor lies along the z–axis at –1.5 < z < 1.5 m
(a) Both (b) 100 W
and carries a fixed current of 10.0 A in  â z  direction (see
(c) 25 W (d) Neither

figure).  For  a  field  B  3.0  104 e0.2x   â y  T,  find the 40. Resistance  of a  given wire is obtained  by measuring the
power  required  to  move  the  conductor  at  constant current  flowing  in  it  and  the  voltage  difference  applied
speed to x = 2.0 m, y = 0 m in 5 × 10–3 s. Assume parallel across it. If the percentage errors in the measurement of the
motion along the x–axis. (2014) current and the voltage difference are 3% each, then error
in the value of resistance of the wire is (AIEEE 2012)
(a) 6% (b) zero
(c) 1% (d) 3%
41. If  400    of  resistance  is  made  by  adding  four  100  
resistance  of  tolerance  5%,  then  the  tolerance  of  the
combination is (AIEEE 2011)
(a) 20% (b) 5%
(c) 10% (d) 15%

(a) 2.97 W (b) 14.85 W 42. The current in the primary circuit of a potentiometer is 0.2 A.


The  specific  resistance  and  cross-section  of  the
(c) 29.7 W (d) 1.57 W
potentiometer  wire  are  4  ×  10–7  –m  and  8  ×  10–7  m2
36. In a large building, there are 15 bulbs of 40 W, 5 bulbs of respectively. The potential gradient will be equal to :
100 W, 5 fans of 80 W and 1 heater of 1 kW. The voltage of (AIEEE 2011)
the electric mains is 220 V. The minimum capacity of the
(a) 0.2 V/m (b) 1 V/m
main fuse of the building will be : (2014)
(a) 10 A (b) 12 A (c) 0.3 V/m (d) 0.1 V/m
(c) 14 A (d) 8 A 43. If a wire is stretched to make it 0.1% longer, its resistance
37. This  question  has  Statement-I  and  Statement-II.  Of  the will (AIEEE 2011)
four  choices  given  after  the  statements,  choose  the  one (a) increase by 0.2% (b) decrease by 0.2%
that best describes the two Statements.
(c) decrease by 0.05% (d) increase by 0.05%
CURRENT ELECTRICITY 59

44. In the circuit shown below, the key K is closed at t = 0. The 47. Shown in the figure adjacent is a meter-bridge set up with


current through the battery is (AIEEE 2010) null  deflection  in  the  galvanometer.  The  value  of  the
unknown resistor R is (AIEEE 2008)

VR1R 2 V G
(a)   at t = 0 and   at t = 
R2 20 cm
R12  R 22

V V  R1  R 2 
(b)  R  at t = 0 and   at t = 
2 R 1R 2
(a) 13.75  (b) 220 
V VR1R 2
(c)   at t = 0 and   at t =  (c) 110  (d) 55 
R2 R12  R 22
48. A 5 V battery with internal resistance 2  and a 2 V battery
V  R1  R 2  V with internal resistance 1  are connected to a 10  resistor
(d)   at t = 0 and   at t = 
R1R 2 R2 as shown in the figure. (AIEEE 2008)

45. Two conductors have the same resistance at 0°C but their P2
temperature coefficients of resistance are 1 and 2. The
respective temperature coefficients of their series and parallel 5V 2V
combinations are nearly. (AIEEE 2010)
P1
1   2 1   2
(a)  , 1   2 (b)  1   2 ,
2 2 The current in the 10  resistor is
1 2 1   2 1   2 (a) 0.27 A, P2 to P1 (b) 0.03 A, P1 to P2
(c)  1   2 ,    (d)  ,
1 2 2 2 (c) 0.03 A, P2 to P1 (d) 0.27 A, P1 to P2
46. This question contains Statement–I and Statement–II. Of 49. The resistance of a wire is 5  at 50°C and 6  at 100°C.
the four choices given after the statements, choose the one The resistance of the wire at 0°C will be (AIEEE 2007)
that best describes the two statements. (a) 2  (b) 1 
Statement-I : The temperature dependence of resistance is (c) 4  (d) 3 
usually given as R = R0 (1 + t). The resistance of a wire
50. A  material  B  has  twice  the  specific  resistance  of A. A
changes from 100  to 150  when its temperature is increased
circular wire made of B has twice the diameter of a wire
from 27°C to 227°C. This implies that  = 2.5 × 10–3/°C.
made  of A.  Then  for  the  two  wires  to  have  the  same
Statement-II : R = Ri (1 + t) is valid only when the change resistance, the ratio B/A of their respective lengths must
in the temperature T is small and R = (R – R0) << R0. be (AIEEE 2006)
(a) Statement-I is true, Statement-II is false. (a) 1 (b) 1/2
(b) Statement-I is true; Statement-II is true; Statement-II is (c) 1/4 (d) 2
the correct explanation of Statement-I. 51. An electric bulb is rated 220 V-100 W. The power consumed
(c) Statement-I is true, Statement-II is true; Statement-II is by it when operated on 110 V will be (AIEEE 2006)
not the correct explanation of Statement-I. (a) 75 W (b) 40 W
(d) Statement-I is false, Statement-II is true. (c) 25 W (d) 50 W
CURRENT ELECTRICITY 60

52. The resistance of a bulb filament is 100  at a temperature 57. Two sources of equal emf are  connected to an external


of 100°C. If its temperature coefficient of resistance be resistance R. The internal resistances of the two sources
0.005  per  °C,  its  resistance  will  become  200    at  a are R1 and R2 (R2 > R1). If the potential difference across
temperature of (AIEEE 2006) the source having internal resistance R2, is zero, then
(a) 300°C (b) 400°C (AIEEE 2005)
(c) 500°C (d) 200°C R 2   R1  R 2 
(a)  R  (b) R = R2 – R1
53. In a Wheatstone’s bridge, three resistances P, Q and R are  R 2  R1 
connected in the three arms and the fourth arm is formed by
R1R 2 R 1R 2
two resistances S1 & S2 connected in parallel. The condition (c)  R  (d)  R 
for the bridge to be balanced will be (AIEEE 2006)  R1  R 2   R 2  R1 
58. A heater coil is cut into two equal parts and only one part
P 2R P R  S1  S2 
(a)   (b)   is now used in the heater. The heat generated will now be
Q S1  S2 Q S1S2
(AIEEE 2005)
P R (a) doubled (b) four times
P R  S1  S2 
(c)   (d)  Q  S  S (c) one-fourth (d) halved
Q 2S1S2 1 2
59. An energy source will supply a constant current into the
54. The  Kirchhoff ’s first law (i =  0) and  second law load, if its internal resistance is (AIEEE 2005)
(iR = E) where the symbols have their usual meanings,
(a) equal to the resistance of the load
are respectively based on (AIEEE 2006)
(b) very large as compared to the load resistance
(a) conservation of charge, conservation of momentum
(c) zero
(b) conservation of energy, conservation of charge
(d) non-zero but less than the resistance of the load
(c) conservation of momentum, conservation of charge
60. In a potentiometer experiment the balancing with a cell is
(d) conservation of charge, conservation of energy
at length 240 cm. On shunting the cell with a resistance of
55. The current I drawn from the 5 V source will be
2 , the balancing length becomes 120 cm. The internal
(AIEEE 2006)
resistance of the cell is (AIEEE 2005)
(a) 1  (b) 0.5 
(c) 4  (d) 2 
61. The resistance of hot tungsten filament is about 10 times
I
the cold resistance. What will be the resistance of 100 W
and 200 V lamp, when not in use ? (AIEEE 2005)
+ –
5V (a) 40  (b) 20 

(a) 0.33 A (b) 0.5 A (c) 400  (d) 200 


(c) 0.67 A (d) 0.17 A 62. The total current supplied to the circuit by the battery is
56. In the circuit, the galvanometer G shows zero deflection. If (AIEEE 2004)
the batteries A and B have negligible internal resistance,
the value of the resistor R will be (AIEEE 2005)
6V
G 2V
A
R
12V  B

(a) 1 A (b) 2 A

(a) 200  (b) 100  (c) 4 A (d) 6 A


(c) 500  (d) 1000 
CURRENT ELECTRICITY 61

63. The resistance of the series combination of two resistances 68. The length of a given cylindrical wire is increased by 100%.


is S. When they are joined in parallel, the total resistance Due to the consequent decrease in diameter the change in
is P. If S = nP, then the minimum possible value of n is the resistance of the wire will be (AIEEE 2003)
(AIEEE 2004)
(a) 200% (b) 100%
(a) 4 (b) 3
(c) 50% (d) 300%
(c) 2 (d) 1
69. A 220 V, 1000 W bulb is connected across a 110 V mains
64. In a meter bridge experiment, null point is obtained at 20 cm
supply. The power consumed will be (AIEEE 2003)
from one end of the wire when resistance X is balanced
against another resistance Y. If X < Y, then where will be (a) 750 W (b) 500 W
the new position of the null point from the same end, if (c) 250 W (d) 1000 W
one decides to balance a resistance of 4X against Y ?
70. A wire when connected to 220 V mains supply has power
(AIEEE 2004)
dissipation P1. Now the wire is cut into two equal pieces
(a) 50 cm (b) 80 cm
which are connected in parallel to the same supply. Power
(c) 40 cm (d) 70 cm dissipation in this case is P2. Then P2 : P1 is
65. Time taken by a 836 W heater to heat one litre of water (AIEEE 2002)
from 10°C to 40°C is     (AIEEE 2004) (a) 1 (b) 4
(a) 50 s (b) 100 s (c) 2 (d) 3
(c) 150 s (d) 200 s 71. If in the circuit, power dissipation is 150 W, then R is
66. The length of a wire of a potentiometer is 100 cm, and the (AIEEE 2002)
emf of its standard cell is E volt. It is employed to measure
the emf of a battery whose internal resistance is 0.5 . If R
the balance point is obtained at  = 30 cm from the positive
end, the emf of the battery is (AIEEE 2003)

30 E 30 E 15 V
(a)  (b) 
100.5 100  0.5

30  E  0.5i  (a) 2  (b) 6 


(c)  ,  where  i  is  the  current  in  the
100 (c) 5  (d) 4 
        potentiometer wire
30 E
(d) 
100
67. A  3  V  battery  with  negligible  internal  resistance  is
connected in a circuit as shown in the figure. The current
I, in the circuit will be (AIEEE 2003)

3V

(a) 1 A (b) 1.5 A

1
(c) 2 A (d)  A
3
CURRENT ELECTRICITY 62

EXERCISE - 3 : ADVANCED OBJECTIVE QUESTIONS


Definition of Current
1. Current  of  4.8  amperes  is  flowing  through  a  conductor.

log I

log I
The  number  of  electrons  crossing  a  cross-section  per
second will be (c)  (d) 
19 21
(a) 3 × 10 (b) 7.68 × 10 O O
log V log V
20 20
(c) 7.68 × 10 (d) 3 × 10
5. The V–I graph for a conductor makes an angle  with
Microscopic form of Ohms law V–axis. Here V denotes the voltage and I denotes current.
2. An ionization chamber with parallel conducting plates as The resistance of conductor is given by
7
anode  and  cathode  has  5  ×  10   electrons  and  the  same (a) sin  (b) cos 
3
number  of  singly-charged  positive  ions  per  cm .  The (c) tan  (d) cot 
electrons are moving at 0.4 m/s. The current density from
2 6. I-V characteristic of a copper wire of length L and area of
anode  to  cathode  is  4  A/m .  The  velocity  of  positive
cross-section A is shown in figure. The slope of the curve
ions moving towards cathode is
becomes
(a) 0.4 m/s (b) 16 m/s
I
(c) Zero (d) 0.1 m/s
Ohmic Conductors
3. Which of the adjoining graphs represents ohmic resistance
O V

(a) More if the experiment is performed at higher temperature
V V
(b) More if a wire of steel of same dimension is used
(a)  (b) 
(c) More if the length of the wire is increased
O I O I
(d) Less if the length of the wire is increased
7. Current flowing through metallic conductor whose area of
cross section increases in the direction of current. If we
V V
move in this direction then
(c)  (d)  (a) the current will change

O I O I (b) current density will change
(c) the drift velocity will increase
4. When  a  current  I  is  passed  through  a  wire  of  constant (d) drift velocity will decrease
resistance, it produces a potential difference V across its Electrical Resistance
ends. The graph drawn between log I and log V will be
8. The resistivity of a wire
(a) Increases with the length of the wire
(b) Decreases with the area of cross-section
log I

log I

(c) Decreases with the length and increases with the cross-
(a)  (b)  section of wire
O log V O log V (d) None of the above statement is correct
CURRENT ELECTRICITY 63

Variation with length and area Temperature Effect


9. Two wires are made up of different materials whose specific 15. A resistance wire has a resistance R. Half of this wire is
resistances are in the ratio of 2:3, lengths in the ratio of 3:4 stretched to double its length and half is twisted to double
and area 4:5.The ratio of their resistances is the thickness ,then its new resistance becomes
(a) 5 : 6 (b) 5 : 8
65 R 45 R
(c) 1 : 4 (d) 1 : 3 (a)  (b) 
32 32
10. A  spherical  shell,  made  of  material  of  electrical
35 R 15 R
109 1 (c)  (d) 
conductivity  m   has thickness of 2 mm and radius 32 32

Variation with Temperature
of 10 cm. In an arrangement its inside surface is kept at a
lower  potential  than  its  outside  surface  .The  resistance 16. Two resistances R1 and R2 are made of different materials.
offered by the shell will be The temperature coefficient of the material of R1 is  and
(a) 5 × 10–11  (b) 5 × 10–11  of the material of R2 is –. The resistance of the series
(c) 2 × 10–12  (d) 2 × 10–11  combination of R1 and R2 will not change with temperature,
11. Two wires of the same dimensions but resistivities 1 and if R1/R2 equals
2  are  connected  in  series.The  equivalent  resistivity  of
  
the combination is (a)  (b) 
  
(a)  2   1  2  (b)  12
 2  2 
(c)  (d) 
 1   2   
(c)    (d)   1  2 
 2  17. The resistance Rt of a conductor varies with temperature t
12. Length of a hollow tube is 5 m, it’s outer diameter is 10 cm as shown in the figure. If the variation is represented by
2
and thickness of it’s wall is 5 mm. If resistivity of the material Rt = R0 [1 + t + t ], then
–8
of the tube is 1.7 × 10  ×m then resistance of tube will be
–5 –5 Rt
(a) 5.6 × 10   (b) 2 × 10  
–5
(c) 4 × 10   (d) none of these
13. In order to quadruple the resistance of a uniform wire, a
part of its length was uniformly stretched till the final length O t
of the entire wire was 1.5 times the original length, the part
of the wire with stretched fraction equal to (a) and are both negative

l (b) and are both positive
(c) is positive and is negative
(d) is negative and are positive
(a) 1/8 (b) 1/6
18. The  resistance  of  a  wire  of  iron  is  10  ohms  and  temp.
(c) 1/10 (d) 1/4 –3
coefficient of resistivity is 5 × 10 /°C. At 20°C it carries
14. The  resistance  of  a  metallic  conductor  increases  with
30  milliamperes  of  current.  Keeping  constant  potential
temperature due to
difference between its ends, the temperature of the wire is
(a) change in carrier density
raised to 120°C. The current in milliamperes that flows in
(b) change in dimensions of the conductor the wire is
(c) increase in number of collisions among the carriers
(a) 20 (b) 15
(d) increase in rate of collisions between charge carriers
and vibrating atoms (c) 10 (d) 40
CURRENT ELECTRICITY 64

19. The voltage V and current I graph for a conductor at two
different temperatures T1 and T2 are shown in the figure. x
The relation between T1 and T2 is

V
T1
V 2V
V (a)  x (b)  x
T2 2L 0 L2

O I 2V 2V 3
(c)  x (d)  x
L2 3L2

23. Two  long  co–axial  and conducting  cylinders  of  radius  a


(a) T1 > T2 (b)  T1  T2
and b are separated by a material of conductivity  and a
(c) T1 = T2 (d) T1 < T2 constant potential difference V is maintained between them
20. The V–i graph for a conductor at temperature T1 °C and by a battery. Then current per unit length of the cylinder
T2 °C are as shown in the figure. (T2 – T1) is proportional from one cylinder to other is
to (Take Ref  at T = 0°C)
12V 4V
(a)  n  b / a  (b)  n  b / a 
T2
V
T1 V 4n  b / a 
(c)  n  a / b  (d) 
V
O i
24. If two wires having resistance R and 2R are joined in series
(a) cos 2 (b) sin  and  then in  parallel.Then the  ratio of  heat generated  in
this situation is
(c) cot 2 (d) tan 
(a) 2 : 1 (b) 1 : 2
21. Two  wires  of  resistance  R1  and  R2  have  temperature
(c) 2 : 9 (d) 5 : 2
coefficient  of  resistance  1  and  2,  respectively.  These
are joined in series. The effective temperature coefficient Joules law (Heating Effect)
of resistance is 25. Consider four circuits shown in the figure below. In which
circuit power dissipated is greatest  (Neglect the internal
1   2 resistance of the power supply)
(a)  (b)  1 2
2

R
1R1   2 R 2 R1R 2 1 2
(a)  E R R  (b)  E
(c)  R  R (d) 
1 2 R12  R 22 R

22. A cylinder solid has length L and radius r is having varying
resistivity given by relation  = 0x, where 0 is positive
constant and x is measured from left to right of the solid. A R R R
cell shown in figure is having e.m.f. V and negligible internal E R
resistance  .The  electric  field  as  a  function  of  x  is  best (c)    (d)  E
R R
described  by
CURRENT ELECTRICITY 65

26. In the following circuit, bulb rated as 1.5 V, 0.45 W. If bulbs (a) 3 (b) 4


glows with full intensity then what will be the equivalent (c) 6 (d) 7
resistance between X and Y ? 31. The effective resistance between the points A and B of the
shown network where the value of each resistor is R is
6 V

X R
Y

(a) 0.45  (b) 1 
(c) 3  (d) 5 
27. A fuse wire with a radius of 1 mm blows at 1.5 A. If the fuse
wire of the same material should blow at 3.0 A ,the radius
(a) 8R/11 (b) 6R/11
of the wire should be
(c) 6R/5 (d) 2R/5
(a)  3 4 mm (b)  3 3 mm 32. The potential difference across 8 ohm resistance is 48 volt
as shown in the figure. The value of potential difference
(c)  4 mm (d) 8 mm
across X and Y points will be
Series and Parallel of Resistors
X
28. In an electrical cable there is a single wire of radius 9 mm of
copper. Its resistance is 5 . The cable is replaced by 6
different insulated copper wires in parallel, the radius of
each wire is 3 mm. Now the total resistance of the cable
will be 48V

(a) 7.5  (b) 45 
Y
(c) 90  (d) 270 
–6 (a) 160 volt (b) 128 volt
29. The resistance of a wire is 10   per metre. It is bend in
the form of a circle of diameter 2 m. A wire of the same (c) 80 volt (d) 62 volt
material  is  connected  across  its  diameter.  The  total 33. What  is the  equivalent resistance  between  the  points A
resistance across its diameter AB will be and B of the network

2m A
A B

4 2
(a)    106  (b)    106 
3 3 B
–6 –6
(c) 0.88 × 10   (d) 14 × 10  
30. There are three resistance coils of equal resistance. The 57
(a)   (b) 8 
maximum  number  of  resistances  you  can  obtain  by 7
connecting them in any manner you choose, being free to 57
use any number of the coils in any way is (c) 6  (d)  
5
CURRENT ELECTRICITY 66

34. A wire of resistor R is bent into a circular ring of radius r.
V
Equivalent resistance between two points X and Y on its B
circumference, when angle XOY is , can be given by
A

X O i

W O Z
(a) A (b) B

Y (c) A and B both (d) Neither A nor B
Wheatstone Bridge
R R 39. The equivalent resistance between the points P and Q in
(a)  2  2    (b)   2   
4 2 3
the network given here is equal to   given r   
4  2 
(c) R (2 – ) (d)   2   
R
35. A  cylindrical  conductor  has  uniform  cross-section. r r
Resistivity of its material increases linearly from left end r
to right end. If a constant current is flowing through it and r r
P Q
at a section distance X from left end, magnitude of electric
field intensity is E, which of the following graphs is correct r
r r
E E

1
(a)  (b)  (a)   (b) 1 
2
O X O X 3
(c)   (d) 2 
E E 2
Kirchhoff’s Laws
(c)  (d)  40. In the circuit element given here, if the potential at point B,
VB = 0, then the potentials of A and D are given as
O X O X
36. A piece of wire of resistance 4 ohms is bent through 180° at
1 amp 2V
its mid point and the two halves are twisted together, then
the resistance is A B C D
(a) 8 ohms (b) 1 ohm
(c) 2 ohms (d) 5 ohms (a) VA = –1.5 V, VD = +2 V (b) VA = +1.5 V, VD = +2 V
37. Two uniform wires A and B are of the same metal and (c) VA = +1.5 V, VD = +0.5 V (d) VA = +1.5 V, VD = –0.5 V
have equal masses. The radius of wire A is twice that of
41. What is the value of R so that ammeter reads zero in the
wire B. The total resistance of A and B when connected
circuit shown below.
in parallel is
(a) 4  when the resistance of wire A is 4.25  A
(b) 5  when the resistance of wire A is 4.25 
(c) 4  when the resistance of wire B is 4.25  6V R 2V
(d) 4  when the resistance of wire B is 4.25 
38. V-i  graphs  for  parallel  and  series  combination  of  two
(a) 600  (b) 300 
identical  resistors are  as  shown  in figure.  Which  graph
represents parallel combination (c) 250  (d) 150 
CURRENT ELECTRICITY 67

Potentiometer (a) 3  (b) 4 


42. The length of wire of potentiometer is 100 cm and the e.m.f. (c) 4.5  (d) 5 
of the standard cells is E volt. It is employed to measure 46. In the following (figure), the equivalent resistance between
the e.m.f. of the battery whose internal resistance is 0.5 the points A and H will be [cos 72° = 0.31] (wire has uniform
ohm. If the balance point is obtained at l = 30 cm from the resistance)
positive end, the e.m.f. of the battery will be

30 E 70 E
(a)  (b)  A
100 100
r r 72°
30 E 30  E  0.5  r C D r
(c)  (d)  B E
100.5 100 r r
F J
Multiple Cells Circuit H
r r
43. You are given several identical resistances each of value r r
G I
R = 10  and each capable of carrying maximum current of
1 ampere. It is required to make a suitable combination of
these resistances to produce a resistance of 5  which can
carry  a  current  of  4  amperes.  The  minimum  number  of (a) 1.944 r (b) 0.973 r
resistances of the type R that will be required for this job (c) 0.486 r (d) 0.243 r
(a) 4 (b) 10 Potential Difference across cell
(c) 8 (d) 20 47. When connected across the terminals of a cell, a voltmeter
measures 5V and a connected ammeter measures 10 A of
44. The reading of the ideal voltmeter in the adjoining diagram
current. A resistance of 2 ohms is connected across the
will be
terminals  of  the  cell.  The  current  flowing  through  this
resistance will be
A
(a) 2.5 A (b) 2.0 A

10V (c) 5.0 A (d) 7.5 A


V
4V 48. Two cells A and B each of e.m.f. 2 volt, are connected in
series to an external resistance R = 1 ohm. If the internal
B N C
resistance of the cell is 1.9 ohm and that of battery B is 0.9
(a) 4 V (b) 8 V ohm. What is the potential difference across the terminals
of cell A
(c) 12 V (d) 14 V
45. Seven resistances are connected as shown in the figure. + – – +
The equivalent resistance between A and B is
A B

R
A B

(a) 2.0 volt (b) 3.8 vol
(c) 1.90 volt (d) 2.5 volt
CURRENT ELECTRICITY 68

49. A wire of resistance 10  is bent to form a circle. P and Q 52. In the circuit of adjoining figure, the current through 12 


are points on the circumference of the circle dividing it resistor will be
into a quadrant and are connected to a Battery of 3 V and
internal resistance 1  as shown in the figure. The currents
in the two parts of the circle are

P 5V 5V
A C

3V E F
Q

1
(a) 1 A (b)  A
5
6 18
(a)  A and A 2
23 23 (c)  A (d) 0 A
5
5 15 53. If in the circuit shown below, the internal resistance of the
(b)  A and A
26 26 battery is 1.5  and VP and VQ are the potentials at P and
Q respectively, what is the potential difference between
4 12
(c)  A and A the points P and Q
25 25

3 9
(d)  A and A 20V
25 25 + –

50. Identical cells each of e.m.f. E and internal resistance r are P
joined in series to form a closed circuit. One cell A is joined
with reverse polarity. The potential difference across each Q
cell except A is

2E  n  1 E
(a)  (b)  (a) Zero (b) 4 volts (VP > VQ)
n n
(c) 4 volts (VQ > VP) (d) 2.5 volts (VQ > VP)
 n  2 E 2nE
(c)  (d)   n  2  54. In the circuit shown in figure reading of voltmeter is V1
n when only S1 is closed, reading of voltmeter is V2 when
51. Potential  difference  across  the  terminals  of  the  battery only S2 is closed and reading of voltmeter is V3 when both
shown in figure is (r = internal resistance of battery) S1 and S2 are closed. Then

3R
R S1
10 V 6R
S2
V

(a) 8 V (b) 10 V (a) V3 > V2 > V1 (b) V2 > V1 > V3

(c) 6 V (d) Zero (c) V3 > V1 > V2 (d) V1 > V2 > V3


CURRENT ELECTRICITY 69

55. In  an  experiment,  a  graph  was  plotted  of  the  potential 59. To get the maximum current through a resistance of 2.5
difference V between the terminals of a cell against the ohm one can use m rows of cells, each row having n cells.
circuit  current  i  by  varying  load  rheostat.  Internal The internal resistance of each cell is 0.5 ohm. What are
conductance of the cell is given by the value of n and m if total number of cells is 45.
(a) 15, 3 (b) 3, 15
V (c) 9, 5 (d) 5, 9
60. In the circuit shown here, E1 = E2 = E3 = 2 V & R1 = R2 = 4 ohms.
x V R
The  current  flowing  between  points A  and  B  through
battery E2 is
y
i
E1 R1
(a) xy (b) y/x

(c) x/y (d) (x – y) E2
A B
Work and Power of Battery in Circuit
56. A torch bulb rated as 4.5 W, 1.5 V is connected as shown E3 R2
in the figure. The e.m.f. of the cell needed to make the bulb
glow at full intensity is (a) Zero (b) 2 amp from A to B
(c) 2 amp from B to A (d) None of these
4.5 W
1.5 V 61. In the circuit shown below E1 = 4.0 V, R1 = 2 , E2 = 6.0 V,
R2 = 4  and R3 = 2 . The current I1 is

E1 = 4V
I1
(a) 4.5 V (b) 1.5 V
I2
(c) 2.67 V (d) 13.5 V
Series and Parallel of Cells E2 = 6V
57. Two batteries of different emf’s and internal resistances
(a) 1.6 A (b) 1.8 A
are  connected  in  series  with  each  other  and  with  an
(c) 1.25 A (d) 1.0 A
external load resistor.The current is 3 ampere. When the
polarity of one battery is reversed the current becomes 1 62. A group of N cells whose emf varies directly with the internal
ampere.The ratio of e.m.f.’s of two batteries is resistance as per the equation EN = 1.5 rN are connected as
shown in the figure below. The current I in the circuit is
(a) 3.0 (b) 2.5
(c) 2.0 (d) 1.5 1
2
58. Two cells of equal e.m.f. and of internal resistances r1 and r1
r2
r2  (r1  >  r2)  are  connected  in  series.  On  connecting  this
N rN
combination  to  an  external  resistance  R,  it  is  observed r3
3
that the potential difference across the first cell becomes r4
zero. The value of R will be 4
(a) r1 + r2 (b) r1 – r2
(a) 0.51 amp (b) 5.1 amp
r1  r2 r1  r2
(c)  (d)  (c) 0.15 amp (d) 1.5 amp
2 2
CURRENT ELECTRICITY 70

63. A battery consists of a variable number ‘n’ of identical (a) 6 A, 60 V (b) 0.6 A, 6 V


cells having internal resistances connected in series. The
(c) 6/11 A, 60/11 V (d) 11/6 A, 11/60 V
terminals of battery are short circuited and the current i is
measured.  Which  of  the  graph  below  shows  the 67. The resistance of a galvanometer is 10 ohm. It gives full
relationship between i and n scale  deflection  when  1  mA  current  is  passed.  The
i i resistance to be connected in series for converting it into a
voltmeter of 2.5 volt will be

(a)  (b)  (a) 249  (b) 2490 


(c) 299  (d) 2999 
O n O n
i i 68. A galvanometer has a resistance of 30 ohm and a current
of 2 mA is needed to give full scale deflection. What is the
resistance needed to convert it into a voltmeter of 0.2 volt
(c)  (d)  range.
O n O n (a) 20  (b) 30 
Galvanometer (c) 60  (d) 70 
64. The ammeter has range 1 ampere without shunt. The range 69. The graph which represents the relation between the total
can  be  varied  by  using  different  shunt  resistances.  The resistance R of a multi range moving coil voltmeter and its
graph between shunt resistance and range will have the full scale deflection V is
nature R R

Q
R
S
 
Shunt

P
S V V
R (i) (ii)
Q
R R
O 1 2 3 4
Range Ampere

(a) P (b) Q
V V
(c) R (d) S (iii) (iv)
Voltmeter (a) (i) (b) (ii)
65. The net resistance of a voltmeter should be high to ensure (c) (iii) (d) (iv)
(a) it does not get overheated R-C Circuit
(b) it does not draw excessive current
70. When the switch is closed then initial current through 1
(c) it can measure large potential difference ohm resistor will be
(d) it does not appreciably change the potential difference
to measure
66. In the circuit shown here, the readings of the ammeter and
voltmeter are
12 V F

V A S
3 
(a) 12 A (b) 4A
(c) 10/7 A (d) 3A
CURRENT ELECTRICITY 71

71. A capacitor is charged then made to discharge through a 76. When a cell is short circuited then


resistance R. The time constant is . In what time will the
(a) current drawn from cell is maximum
potential difference across the capacitor decreases by 10%.
(b)  potential  difference  across  cell  terminals  become
(a)  ln (0.1) (b)  ln (0.9)
maximum
(c)  ln (11/10) (d)  ln (10/9)
(c) potential difference across cell terminals become zero
72. In  the  circuit  shown  in  figure  ,the  charge  on  the  upper
plate is (d) current drawn from cell is zero

R A 2R 77. If three  resistors R1,  R2,  R3 are joined in parallel  across


some potential source then
C (a)  equivalent  resistance  is  less  than  lowest  among  the
2R R individual resistors
B (b)  applied  potential  difference  is  equal  to  potential
difference across each resistor
(c) current supplied by the source is shared by resistors
in inverse ratio
(a) positive (b) negative
(d) current supplied by the source is shared directly among
(c) zero (d) oscillating
the  resistors
73. When  the  key K  is  pressed  at  time  t  =  0, which  of  the
78. In potentiometer experiment
following statements about the current I in the resistor AB
of the given circuit is true (a) the wire must be uniform and must be long
(b) e.m.f. of battery in main circuit must be greater than
A B e.m.f. to measure
2V K (c) positive terminal of battery and cell must be connected
at same point
C
(d) none of these
79. A  constant  voltage  is  applied  between  two  ends  of  a
uniform metallic wire. If both the length and radius of wire
(a) I = 2 mA at all t are doubled
(b) I oscillates between 1 mA and 2 mA (a) heat developed in wire will be doubled
(c) I = 1 mA at all t (b) heat developed remains the same
(d) At t = 0, I = 2 mA and with time it goes to 1 mA (c) electric field in the wire will be halved
Multiple Correct Option (d) electric field in the wire will be doubled
74. E.m.f. of a cell is independent of 80. In the circuit, the battery is ideal. A voltmeter of resistance
(a) quantity of electrolyte present in cell 600  ohm is  connected in  turn acrosss  R1 and  R2 giving
readings V1 and V2 respectively.
(b) nature of the electrode and electrolyte used in cell
(c) size of the electrode used in cell
(d) distance between the electrodes
75. Potential difference between the terminals of a cell
E = 120 Volt
(a) is greater than e.m.f. of the cell when cell is discharged
(b) is greater than e.m.f. when cell is being charged
(c) is less than e.m.f. if the cell is charged (a) V1 = 80 volt (b) V1 = 60 volt
(d) is equal to its e.m.f. if the cell is in open circuit (c) v2 = 40 volt (d) V2 = 30 volt
CURRENT ELECTRICITY 72

81. A  current  is  passed  through  wire  of  non-uniform  cross 86. Two  cells  of  unequal  e.m.f.  E 1  and  E 2  with  internal
section. Which of the following quantities are independent resistances r1 and r2 are joined as shown in figure. If VA and
of cross section. VB are the potential at A and B respectively then
(a) charge crossing in a given time interval
+ –
(b) drift speed
(c) current density E1, r1
A B
(d) free electron density
82. Which of the following statements are correct. + –
(a) resistance of filament of bulb is inversely proportional
E2, r2
to power of bulb
(b) resistance is directly proportional to power of the bulb (a) one cell will continuously supply energy to other
(c) higher the wattage of bulb, higher is the current that (b) potential difference across both the cells will be
can be allowed to pass
equal
(d) higher the wattage of bulb, lower is the current that
(c) The potential difference across one cell is will be
can be allowed to pass
more than its e.m.f.
83. A given number of cells each of same e.m.f. and internal
resistance r are to be grouped in such a way that the current  E1r2  E 2 r1 
flowing through external resistance R is maximum. Which (d)  VA  VB 
r1  r2
of the following statement is correct.
87. For the circuit shown, select the correct statements from
(a) series combination if R > r and parallel combination if R < r
the following
(b) series combination if R < r and parallel combination if R > r
(c) mixed combination for series combination if R > r and
parallel combination if R = r
(d) none of these 2 2
84. The charge flowing in a conductor varies with time as 2
A B
q = at – bt2,then the current
(a) decreases linearly with time 2 2
(b) reaches a maximum and then decreases

a
(c) falls to zero after a time  t 
2b (a) A and B are equi-potential points
(d) changes at a rate of –2b (b) Effective resistance between x and y is 2 ohm
85. A voltmeter and an ammeter are connected  in series to an (c) Effective resistance between x and y is 1 ohm
ideal cell of e.m.f. E. The voltmeter reads V and current (d) None of these
reading is I. Then
88. A  voltmeter  reads  the  potential  difference  across  the
(a) V < E
terminals of an old battery as 1.40 volt while a potentiometer
V reads its voltage to be 1.55volt.the voltmeter resistance is
(b) the voltmeter resistance is 280 ohm then
I
(c) the potential difference across the ammeter is (E-V) (a) e.m.f. of the battery is 1.55 volt

(d) voltmeter resistance plus ammeter resistance equal (b) The e.m.f. of the battery is 1.40 volt

E (c) Internal resistance of the battery is 5 ohm
to
I (d) Internal resistance of the battery is 30 ohm
CURRENT ELECTRICITY 73

89. The potential difference across the terminals of a non ideal Assertion Reason Type Questions


battery  is (a) Statement–1 is True, Statement–2 is True; Statement–2 is
(a) zero when it is short circuited a correct explanation for Statement–1.
(b) less than its e.m.f. when current flows from negative to (b) Statement–1 is true, Statement–2 is True; Statement–2 is
positive terminal inside the battery NOT a correct explanation for Statement–1.
(c) zero when no current is drawn from battery (c) Statement–1 is True; Statement–2 is False.
(d) Greater than its e.m.f. when current flows from positive (d) Statement–1 is False, Statement–2 is True.
to negative terminal inside the battery
92. Statement–1 : At  cryogenic  temperatures,  the  electrical
90. A battery of e.m.f. E and internal resistance r is connected resistivity in metallic conductors diminishes.
across a resistance R. Resistance R can be adjusted to any
Statement–2 : Thermal oscillations of atoms which hinder
value greater than equal to zero. A graph is plotted between
motion of free electrons under the influence of an external
the  current  passing  through  the  resistance  and  potential
difference across it. Select the correct alternative. field become insignificant.
(a) A (b) B
(c) C (d) D
10 volt

93. Statement– 1 :  In the circuit shown assume both cell are


ideal and of fixed e.m.f., the resistor R1 is fixed, the resistor
R2 is non zero variable .Then the electric power delivered to
V the resistor R1 is independent of value of resistance R2
O I 2

R1
(a) internal resistance of the battery is 5 ohm
E1 E2
(b) e.m.f. of the battery is 10 volt R2
(c) maximum current which can be taken from battery is 2
ampere
Statement–2 :  If  potential  difference  across  a  fixed
(d) V versus I graph can never be straight line as shown
resistance is unchanged, the power delivered to the resistor
here.
remains unchanged.
91. In the circuit shown in the figure
(a) A (b) B
(c) C (d) D
94. Statement–1 : A current flows in a conductor when there is
electric field within the conductor.
2A
R3 R2 R1 Statement–2 : The drift velocity of electrons in presence of
electric field decreases.
V
0.8 A 0.4 A (a) A (b) B
(c) C (d) D
(a) voltage V=10 volt 95. Statement–1 :  When  the  battery  is  short  circuited  the
terminal potential difference is zero.
(b) resistance R1=10 ohm
Statement–2 : In the situation of short circuit, the current
(c) resistance R2=20 ohm
is zero.
(d) equivalent resistance of the circuit is 10 ohm (a) A (b) B
(c) C (d) D
CURRENT ELECTRICITY 74

96. Statement–1 : Potentiometer is better for e.m.f. measurement 103. Statement–1 :  Current  density  at  any  point  in  ohmic
of a cell than a voltmeter. resistor is in the direction of electric field at that point.
Statement–2 : A potentiometer draws  no current while Statement–2 :  A point charge when released from rest in
measuring e.m.f. of a cell. a region having only electrostatic field always move along
electric field lines.
(a) A (b) B
(a) A (b) B
(c) C (d) D
(c) C (d) D
97. Statement–1 : Two unequal resistors are connected  in
104. Statement–1 : The drift velocity of electrons in metallic
series  across  a  cell.  Then  the  potential  drop  across  the
wire will decrease if the temperature of the wire is increased.
larger resistor is more.
Statement–2 : On increasing temperature, conductivity
Statement–2 : Current will be same in both the resistors.
of metallic wire decreases.
(a) A (b) B
(a) A (b) B
(c) C (d) D (c) C (d) D
98. Statement–1 : If the current in the lamp decreases by 20%, 105. Statement–1 : Voltmeter always gives e.m.f. of a cell if it is
the percentage decrease in its illumination is 40%. connected across the terminals of a cell.
Statement–2 : Illumination is directly proportional to the Statement–2 :  Terminal potential difference of a cell is
square of current flowing through lamp. given by V=E-Ir.
(a) A (b) B (a) A (b) B
(c) C (d) D (c) C (d) D
99. Statement–1 : In a simple battery circuit the point of lowest 106. Statement–1 : Potential measured by voltmeter across a
potential is negative terminal of battery. wire  is  always  less  than  the  actual  potential  difference
across it.
Statement–2 : Current flows towards the point of higher
potential  as  it  does  in  such  a  circuit  from  negative  to Statement–2 :  Finite  resistance  of  voltmeter  changes
positive terminal. current  flowing  through  the  resistance  across  which
potential difference is to be measured.
(a) A (b) B
(a) A (b) B
(c) C (d) D
(c) C (d) D
100. Statement–1 : Fuse wire must have high resistance and
107. Statement–1 : When two conducting wires of different
low melting point.
resistivity  have  same  cross  sectional  area  are  joined  in
Statement–2 : Fuse is used for small current flow only. series ,the electric field in them would be equal when they
(a) A (b) B carry current.
(c) C (d) D Statement–2 : When wires are in series they carry equal
current.
101. Statement–1 : A 200 watt bulb glows with more brightness
than 100 watt bulb. (a) A (b) B

Statement–2 : 100 watt bulb has more resistance than 200 (c) C (d) D


watt. Comprehensive type questions
(a) A (b) B Passage - 1
(c) C (d) D Using the following passage, solve Q. 108 to 111
102. Statement–1 : It is advantageous to transmit electric power A potential difference is applied across a copper wire of
at high voltage. radius 0.5 mm. It results in a uniform electric field of 1.5 V/m
along the length of the wire. Consequently there is a current
Statement–2 : High voltage implies high current.
in the wire. Temperature of the wire is  60oC.Assuming each
(a) A (b) B copper  atom  contributes  one  free  electron  .Given  that
(c) C (d) D density of copper is 8.9 gm/cm3 and resistivity of copper at
CURRENT ELECTRICITY 75

20oC is 1.7×10–8 ohm.m and its temperature coefficient of Passage -3
resistivity at 20oC is 3.9×10–3/OC  . Atomic mass of copper is Using the following passage, solve Q. 116 to 118
63.5. Answer the following questions.
The potentiometer basically consists of a uniform wire of
108. Resistivity of copper under experimental condition is resistance R through which a steady current flows when a
(a) 4.5 × 106 m (b) 1.96 × 10–8 m driver cell source is connected between the points A and
(c) 3.2 × 10–8 m (d) 5.1 × 10–8 m B (ends of potentiometer wire) and tapping is adjusted till
no current passes through galvanometer. Using this null
109. Current density in the wire is
deflection method, unknown e.m.f. of a cell is determined.
(a) 11.24 × 106 A/m2 (b) 5.4 × 107 A/m2 Using this concept answer the following questions.
(c) 7.65 × 107 A/m2 (d) 52.5 × 106 A/m2 116. The  current  in  potentiometer  wire  is adjusted  to give a
110. Drift speed of the electron is null point at 56 cm with a standard cell of e.m.f. 1.02 volt.
(a) 2.3 × 10–2 m/s (b) 4.5 × 10–3 m/s The e.m.f. of another cell for which null point is obtained
at 70 cm is
(c) 7.5 × 103 m/s (d) 5.7 × 103 m/s
(a) 0.78 volt (b) 1.275 volt
111 Potential difference  that  need to be  applied  between the
ends of a 4 m long wire to produce the given field is (c) 1.35 volt (d) 1.54 volt

(a) 3 volt (b) 4.5 volt 117. The total length of the potentiometer wire is 10 m. The


distance  between  the  null  points  on  potentiometer  wire
(c) 6 volt (d) 1.5 volt
for two cells is 60 cm.If the difference between e.m.f.’s of
Passage - 2 the cells be 0.4 volt, the potential gradient along the wire
Using the following passage, solve Q. 112 to 115 is
In the circuit shown the resistances are given in ohms and 4
the battery is assumed to be ideal with e.m.f. equal to 3.0 (a)  volt/m (b) 1 volt/m
3
volt.
2 1
(c)  volt/m (d)  volt/m
3 2
118. How  can  sensitivity  of  this  potentiometer  be  further
increased.
3.0 V
(a) increasing the e.m.f. of driver cell
(b) increasing the length of potentiometer wire
(c) decreasing the length of potentiometer wire
112. The resistor that dissipated maximum power will be (d) increasing the cross sectional area of wire
(a) R1 (b) R2 Passage - 4
(c) R3 (d) R5 Using the following passage, solve Q. 119 to 123
113. The electric current passing through 3 volt battery is A  Galvanometer  is  an  instrument  that  can  be  used  to
construct an ammeter for measuring current .It can be used
(a) 30 mA (b) 40 mA
to convert to a voltmeter. It can also be used as a multimeter.
(c) 50 mA (d) 60 mA
In all cases resistance must be connected to galvanometer
114. Electric potential drop across R4 is either in series or in parallel to effect the change .To turn
(a) 0.5 V (b) 1.0 V into an ammeter a low resistance in parallel of suitable value
and to convert to a voltmeter a resistance is connected in
(c) 1.5 V (d) 2.0 V
series. You are given a galvanometer for which a current of
115. The potential difference across the resistor R5 is
10 mA is required for full deflection .The internal resistance
(a) 0.4 V (b) 0.5 V of the galvanometer is 100 ohm. Then answer the following
(c) 0.6 V (d) 1.0 V questions.
CURRENT ELECTRICITY 76

119. Which  of  the  following  formula  correctly  applies  to  the Kettle A : specific heat capacity;1680 J/kg–k, mass = 200 gm
ammeter circuit. cost = Rs. 400
Kettle B : specific heat capacity = 2450 J/kg–K mass = 400 gm
G cost = Rs 400. When kettle A is switched on with constant
Ir I
potential source, the tea begins to boil in 6 minutes. When
IR kettle B is switched on with same source separately, tea
begins  to  boil  in  8  minutes.  They  made  discussion  on
R specification and efficiency of kettles and prepared a list
(a) VR>Vr (b) Vr>VR of questions to draw a conclusion. Some of these are as
under (take specific heat of tea liquid = 4200 J/kg–k, and
(c) VR=Vr (d) more information required density = 1000 kg/m3)
120. What  resistance  must  be  connected  in  series  to  the 124. Efficiency of kettle A is
galvanometer in order to convert it to a 100 volt voltmeter? (a) 93.34% (b) 83.34%
(a) 600 ohm (b)1000 ohm (c) 73.34% (d) 100%
(c) 9900 ohm (d) 13,900 ohm 125. Efficiency of kettle B is
121. To double the full scale voltage reading of any galvanometer (a) 93.5% (b) 83.5%
turned into a voltmeter, you must (c) 73.5% (d) 62.5%
126. If the resistances of coil of kettle A and B are RB and RB
respectively, then we can say
G
Ig I R (a) RA > RB

Voltmeter (b) RA > RB

(a) doubles the resistance R extra connected (c) RA = RB

(b) half the resistance R (d) no relation can be predicted

(c) Increase the resistance to 4R 127. If both the kettles are joined with the same source in series


one after the other .The boiling starts in kettle A & B after
(d) more than 2R
(a) four times of their original time
122. In voltmeter circuit the current in resistor R must be
(b) two times of their original time
(a)  negligible  so  that  it only  has  small effect  on voltage
reading (c) equal to their original time

(b)  substantial  but  does  not  have  any  effect  on  voltage (d) can not be ascertained from this data.
reading Match the Column Type Question
(c)  substantial,  but  does  have  some  effect  on  voltage 128. For an electric conductor, Match column I with Column II
reading Column-I Column-II
(d) none of these (a) Electric conductivity of a (p) Electric field
123. What  resistance  must  be  connected  in  parallel  to  the conductor  depends  on strength
galvanometer to turn it into an ammeter capable of reading (b) Conductance of a (q) Temperature
electric current up to 10.01 A. conductor  depends  on
(a) 0.1 ohm (b) 1 ohm (c) For a given conductor (r) Nature of the
(c) 1.01 ohm (d) 10.01 ohm at a given temperature conductor
Passage - 5 current  density  depends  on
Using the following passage, solve Q. 124 to 127 (d) For a given potential (s) Dimensions of
Bohr and Biot purchased two electric kettles A and B of the difference applied across a conductor (area/
same size and thickness and same volume of 0.4 litre. They conductor, current in it will length)
studied the specification of kettles as under depends  on
CURRENT ELECTRICITY 77

129. In  an  electric  circuit  with  two  batteries  in  series  with (d) Equivalent resistance (s) On removing
opposite  polarities  are  shown  in  the  figure,  match  the between C and D resistance  between
following quantities in column I with column-II.
B and D equivalent
B1 B2
resistance  becomes
5R/8
131. Consider two identical cells each of e.m.f. E and internal
resistance  of  r  connected  to  a  load  resistance  R.  Match
column-1 with column-2.
Column-1 Column-2
Column-I Column-II
E2
(a) Potential difference across (p) B1 (a) For parallel combination (P)
4r
B1 battery
of cells
(b) Potential difference across (q) 9 volt
r
B2 battery (b) For series combination (Q) Eeq = E,  req 
2
(c) power is supplied by battery (r) B2
of cells
(d) Powerr is consumed by (s) 14 volt
(c) For maximum power (R) Eeq = 2E, req = 2q
battery (t) none transfer to load when
130. Referring to a circuit  shown  below, match the column I cells in series
with Column II
E2
(d) For maximum power (S)
2r
transfer when cells
are in parallel
Integer type questions
132. In the circuit shown below , calculate the current flowing
through 3 ohm resistance.

A B
Column-I Column-II
6V
(a) Equivalent resistance (p) The circuit is a 2V
10V
between A and C balanced  wheatstone D C
 bridge plus a E
resistance parallel to
bridge 133. Figure below shows a bridge circuit of the five resistors.
(b) Equivalent resistance (q) R/2 What is potential difference Vba in volt in the circuit shown.

between B and D
1
(c) Equivalent resistance (r) On removing the
1
A and B resistance  between 1
1.3 a b
B and D equivalent
Volt 1
resistance  becomes 2
R/2
CURRENT ELECTRICITY 78

134. The potential difference between the points A and B shown 136. Three  2  ohm  resistors  are  connected  as  shown  in  figure
in the circuit will be …..Volt. below. Each can withstand a maximum power of 32 watt
without becoming excessive hot .What maximum power in
watt that can be delivered to the combination of resistors.
A
C F

10V 4V 9V
A B

D G
B 137. An ideal battery sends a current of 5 A in a resistor. When
another resistor of value 10 ohm is connected in parallel
the current through battery increases to 6  ampere.  Find
135. In the given circuit, what is the value of R in ohm for which
the resistance of first resistor.
current flowing through 4 ohm resistor is zero.
138 A  galvanometer  has  full  scale  range  of  50A  with  a
A B C resistance of 100  When 1 In parallel, it can be used as
a higher range ammeter. What is its range in mA.
2 R

10V 4
4V

6V

D
CURRENT ELECTRICITY 79

EXERCISE - 4 : PREVIOUS YEAR JEE ADVANCED QUESTIONS

Fill in the Blanks


1. An electric bulb rated for 500 W at 100 V is used in a circuit
having a 200 V supply. The resistance R that must be put
in series with the bulb, so that the bulb delivers 500 W is
......... (JEE 1987)
2. The equivalent resistance between points A and B of the The heat generated in the 4  resistor is
circuit given below is ......... (JEE 1997) (a) 1 cal/s (b) 2 cal/s
(c) 3 cal/s (d) 4 cal/s

2R 2R R 7. The current i in the circuit (see figure) is (JEE 1983)


A B

I
2V
3. In the circuit shown below, each battery is 5 V and has an
internal  resistance  of  0.2  .  The  reading  in  the  ideal
voltmeter V is ........V. (JEE 1997)

1 1
(a)  A (b)  A
45 15
V
1 1
(c)  A (d)  A
10 5
True/False 8. A piece of copper and another of germanium are cooled
4. Electrons in a conductor have no motion in the absence of from room temperature to 80 K. The resistance of
a potential difference across it. (JEE 1982) (a) each of them increases (JEE 1988)
5. The  current-voltage  graphs for a  given metallic  wire  at (b) each of them decreases
two  different  temperatures  T1  and  T2  are  shown  in  the
(c) copper increases and germanium decreases
figure. The temperature T2 is greater than T1.
(d) copper decreases and germanium increases
(JEE 1985)
9. Read the following statements carefully (JEE 1993)
I T1 Y  :  The  resistivity  of  semiconductor  decreases  with
increase of temperature.
T2
Z : In a conducting solid, the rate of collisions between free
electrons and ions increases with increase of temperature.
V
Select the correct statement(s) from the following :
Objective Questions (Only one correct option) (a) Y is true but Z is false
6. In the circuit shown in figure the heat produced in the 5  (b) Y is false but Z is true
resistor due to the current flowing through it is 10 cal/s.
(c) Both Y and Z are true
(JEE 1981)
(d) Y is true and Z is the correct reason for Y
CURRENT ELECTRICITY 80

10. A  battery  of  internal  resistance  4  is  connected  to  the 14. In  the  given  circuit,  it  is  observed  that  the  current  I  is
network of resistances as shown. In order that the maximum independent of the value of the resistance R6. Then, the
power can be delivered to the network, the value of R in  resistance  values must  satisfy (JEE 2001)
should  be (JEE 1995)
R5

I R1 R3
R R R6
R 6R R
E R 4R R2 R4

(a) 4/9 (b) 2 (a) R1R2R5 = R3R4R6

(c) 8/3 (d) 18 1 1 1 1
(b)  R  R  R  R  R  R
11. A  steady  current  flows  in  a  metallic  conductor  of  non- 5 6 1 2 3 4

uniform  cross-section.  The  quantity/quantities  constant


(c) R1R4 = R2R3 (d) R1R3 = R2R4
along the length of the conductor is/are (JEE 1997)
15. A wire of length L and 3 identical cells of negligible internal
(a) current, electric field and drift speed resistances  are  connected  in  series.  Due  to  the  current,
(b) drift speed only the temperature of the wire is raised by T in a time t. A
number N of similar cells is now connected in series with a
(c) current and drift speed
wire of the same material and cross-section but of length
(d) current only 2L. The temperature of the wire is raised by the same amount
12. In the circuit shown in the figure, the current through T in the same time. The value of N is (JEE 2001)
(JEE 1998) (a) 4 (b) 6
(c) 8 (d) 9
16. In the given circuit, with steady current, the potential drop
9V across the capacitor must be (JEE 2001)

V R

V C

(a) the 3  resistor is 0.50 A 2V 2R

(b) the 3  resistor is 0.25 A
(a) V (b) V/2
(c) the 4  resistor is 0.50 A
(c) V/3 (d) 2V/3
(d) the 4  resistor is 0.25 A
17. The  effective resistance between points P and  Q of the
13. In the circuit shown P  R, the reading of galvanometer is
electrical circuit shown in the figure is (JEE 2002)
same with switch S open or closed. Then (JEE 1999)
2R 2R
P Q
2R
r r
S P Q
R G 2R
2R 2R

V 2 Rr 8 R R  r 
(a)  (b) 
Rr 3R  r
(a) IR = IG (b) IP = IG
5R
(c) 2r + 4R (d)   2r
(c) IQ = IG (d) IQ = IR 2
CURRENT ELECTRICITY 81

18. A 100 W bulb B1, and two 60 W bulbs B2 and B3, are
P
connected to a 250 V source, as shown in the figure. Now
W1, W2 and W3 are the output powers of the bulbs B1, B2
and B3 respectively. Then (JEE 2002)

Q R

B1 B2
(a) P and Q (b) Q and R
(c) P and R (d) any two points
B3
22. A capacitor  is charged  using an external battery with a
250V
resistance x in series. The dashed line shows the variation
of ln I with respect to time. If the resistance is changed to
(a) W1 > W2 = W3 (b) W1 > W2 > W3 2x, the new graph will be  (JEE 2004)

(c) W1 < W2 = W3 (d) W1 < W2 < W3
19. In the shown arrangement of the experiment of the meter l n I S
R
bridge  if  AC  corresponding  to  null  deflection  of
galvanometer is x, what would be its value if the radius of Q
the wire AB is doubled ? (JEE 2003) P
t
O

(a) P (b) Q
R1 R2 (c) R (d) S

G 23. A moving coil galvanometer of resistance 100  is used as an
A B ammeter using a resistance 0.1 . The maximum deflection
x C current in the galvanometer is 100 A. Find the minimum
current in the circuit, so that the ammeter shown maximum
(a) x (b) x/4
deflection. (JEE 2005)
(c) 4x (d) 2x
(a) 100.1 mA (b) 1000.1 mA
20. Three  resistances  of  equal  value  are  arranged  in  the
different  combinations  shown  below. Arrange  them  in (c) 10.01 mA (d) 1.01 mA
increasing order of power dissipation. (JEE 2003) 24. A rigid container with thermally insulated walls contains a
coil of resistance 100 , carrying current 1A. Change in
i
internal energy after 5 min will be (JEE 2005)
i (a) zero (b) 10 kJ
(I) (II)
(c) 20 kJ (d) 30 kJ
25. Find out the value of current through 2  resistance for
i i
the given circuit. (JEE 2005)
(III) (IV)

(a) III < II < IV < I (b) II < III < IV < I 10V 20V


(c) I < IV < III < II (d) I < III < II < IV
21. Six equal resistances are connected between points P, Q
and R as shown in the figure. Then, the net resistance will (a) 5 A (b) 2 A
be maximum between (JEE 2004) (c) zero (d) 4 A
CURRENT ELECTRICITY 82

26. A 4 F capacitor, a resistance of 2.5 M is in series with 12 V (a) zero (b) 54 C


battery. Find the time after which the potential difference
(c) 27 C (d) 81 C
across  the  capacitor  is  3  times  the  potential  difference
across the resistor. [Given ln (2) = 0.693] (JEE 2005) 30. A  resistance  of  2    is  connected  across  one  gap  of  a
(a) 13.86 s (b) 6.93 s metre-bridge (the length  of the  wire  is 100  cm)  and  an
(c) 7 s (d) 14 s unknown resistance, greater than 2 , is connected across
27. Two bars of radius r and 2r are kept in contact as shown. the other gap. When these resistances are interchanged,
An electric current I is passed through the bars. Which the  balance  point  shifts  by  20  cm.  Neglecting  any
one of following is correct ? (JEE 2006) corrections, the unknown resistance is (JEE 2007)
(a) 3  (b) 4 
I/2
I/2
2r r (c) 5  (d) 6 
I
C 31. Figure shows three resistor configurations R1, R2 and R3
A B
connected to 3 V battery. If the power dissipated by the
(a) Heat produced in bar BC is 4 times the heat produced configuration R1, R2 and R3 is P1, P2 and P3, respectively,
in bar AB then (2008)
(b) Electric field in both halves is equal
(c) Current density across AB is double that of across BC
(d) Potential difference across AB is 4 times that of across BC
28. Find the time constant for the given RC circuits in correct
order. (JEE 2006)

V V
R1
C1
C1 R1
R2 C2

(a) P1 > P2 > P3 (b) P1 > P3 > P2
C2 R2
V (c) P2 > P1 > P3 (d) P3 > P2 > P1
R1 C1
32. Consider  a  thin  square  sheet  of  side  L  and  thickness  t,
made of a material of resistivity . The resistance between
R2 C2 two  opposite  faces,  shown  by  the  shaded  areas  in  the
figure is (2010)
R1 = 1, R2 = 2, C1 = 4F, C2 = 2F
(a) 18, 4, 8/9 (b) 18, 8/9, 4
(c) 4, 18, 8/9 (d) 4, 8/9, 18
29. A  circuit  is  connected  as  shown  in  the  figure  with  the
switch S open. When the switch is closed, the total amount
of charge that flows from Y to X is  (JEE 2007)

X (a) directly proportional to L

S (b) directly propotional to t

(c) independent of L
Y
(d) independent of t
9V
CURRENT ELECTRICITY 83

33. Incandescent bulbs are designed by keeping in mind that
the resistance of their filament increases with the increase
in temperature. If at room temperature, 100W, 60W and 40W
bulbs have filament resistance R100, R60 and R40, respectively,
the relation between these resistances is (2010)
1 1 1
(a)  R  R  R (b) R100 = R40 + R60 (a)  60  0.15 (b) 135  0.56
100 40 60
(c)  60  0.25 (d) 135  0.23
1 1 1 37. In  an  aluminium  (Al)  bar  of  square  cross  section,  a
(c) R100 > R60 > R40 (d)  R  R  R
100 40 60 square  hole  is  drilled  and  is  filled  with  iron  (Fe)  as
shown in the figure. The electrical resistivities of Al and
34. To  verify  Ohm’s  law,  a  student  is  provided  with  a  test
Fe are 2.7 × 10 m and 1.0 × 10 m, respectively. The
resistor RT, a high resistance R1, a small resistance R2, two
electrical resistance between the two faces P and Q of the
identical galvanometers G1 and G2, and a variable voltage
composite bar is (2015)
source V. The correct circuit to carry out the experiment is
(2010)

(a)  (b) 

2475 1875
(a)   (b)  
(c)  (d)  64 64

1875 2475
(c)   (d)  
49 132
35. A meter bridge is set-up as shown in figure, to determine 38. An infinite line charge of uniform electric charge density
an unknown resistance X using a standard 10  resistor.  lies along the axis of an electrically conducting infinite
The galvanometer shows null point when tapping-key is cylindrical shell of radius R. At time t = 0, the space inside
at 52 cm mark. The end-corrections are 1 cm and 2 cm the cylinder is filled with a material of permittivity  and
respectively for the ends A and B. The determined value electrical conductivity . The electrical conduction in the
of X is (2011) material follows Ohm’s law. Which one of the following
graphs  best  describes  the  subsequent  variation  of  the
magnitude  of  current  density  j(t)  at  any  point  in  the
material? (2016)

(a) 10.2  (b) 10.6  (a)  (b) 


(c) 10.8  (d) 11.1 
36. During an experiment with a metre bridge, the galvanometer
shows a null point when the jockey is pressed at 40.0 cm
using a standard resistance of 90, as shown in the figure.
The least count of the scale used in the metre bridge is 1 (c)  (d) 
mm. The unknown resistance is (2014)
CURRENT ELECTRICITY 84

Assertion and Reason 43. For the circuit shown in the figure (2009)

(a) If Statement-I is true, Statement-II is true; Statement-II is
the correct explanation for Statement-I.
(b) If Statement-I is true, Statement-II is true; Statement-II is
not a correct explanation for Statement-I.
(c) If Statement-I is true; Statement-II is false.
(d) If Statement-I if false; Statement-Ii is true.
39. Statement-I : In a meter bridge experiment, null point for an (a) the current I through the battery is 7.5 mA
unknown  resistance is  measured.  Now,  the  the  unknown (b) the potential difference across RL is 18 V
resistance is put inside an enclosure maintained at a higher (c) ratio of powers dissipated in R1 and R2 is 3
temperature.  The null point can be obtained  at the  same
(d) if R1 and R2 are interchanged, magnitude of the power
point  as  before  by  decreasing  the  value  of  the  standard
dissipated in RL will decrease by a factor of 9
resistance.
44. For the resistance network shown in the figure, choose the
Statement-II : Resistance of a metal increases with increase
correct option(s). (2012)
in temperature. (2003)
(a) A (b) B
(c) C (d) D
Objective Question (One or more correct answer)
40. Capacitor  C1  of  capacitance  1  F  and  capacitor  C2  of
capacitance 2 F are separately charged fully by a common
battery. The two capacitors are then separately allowed to
discharge through equal resistors at time t = 0.(JEE 1989)
(a) The current in each of the two discharging circuits is
zero at t = 0.
(a) The current through PQ is zero
(b) The currents in the two discharging circuits at t = 0
(b) I1 = 3A
are equal but not zero.
(c) The currents in the two discharging circuits at t = 0 (c) The potential at S is less than that at Q
are unequal. (d) I2 = 2A
(d) Capacitor C1, losses 50% of its initial charge sooner 45. Two ideal batteries of emf V1 and V2  and three resistances
than C2 loses 50% of its initial charge. R1, R2 and R3 are connected as shown in the figure. The
41. A microammeter has a resistance of 100  and full scale current in resistance R2 would be zero if (2014)
range of 50 A. It can be used as a voltmeter or as a higher
range ammeter provided a resistance is added to it. Pick
the correct range and resistance combination (s)
(JEE 1991)
(a) 50 V range with 10 k resistance in series
(b) 10 V range with 200 k resistance in series
(c) 5 mA range with 1  resistance in parallel
(d) 10 mA range with 1  resistance in parallel
42. When a potential difference is applied across, the current
passing  through (JEE 1999)
(a) an insulator at 0 K is zero (a) V1 = V2 and R1 = R2 = R3
(b) a semiconductor at 0 K is zero (b) V1 = V2 and R1 = 2R2 = R3
(c) a metal at 0 K is finite (c) V1 = 2V2 and 2R1 = 2R2 = R3
(d) a p-n diode at 300 K is finite, if it is reverse biased (d) 2V1 = V2 and 2R1 = R2 = R3
CURRENT ELECTRICITY 85

46. A  conductor  (shown  in  the  figure)  carrying  constant (d) The maximum current range is obtained when the


current I is kept in the x-y plane in a uniform magnetic two  galvanometers  are  connected  in  series  and  the
 combination  is  connected  in  parallel  with  both  the
field   B . If F is the magnitude of the total magnetic force
resistors
acting on the conductor, then the correct statement(s) is
(are) (2015) 49. A  conducting  loop  in  the  shape  of  a  right  angled
isosceles triangle of height 10 cm is kept such that the
90° vertex is very close to an infinitely long conducting
wire (see the figure). The wire is electrically insulated
from the loop. The hypotenuse of the triangle is parallel
to  the  wire.  The  current  in  the  triangular  loop  is  in
 counterclockwise direction and increased at a constant
(a)  If B is along z,
ˆ F  (L  R)
rate  of  10 As–1.  Which  of  the  following  statement(s)
 is(are) true? (2016)
(b)  If B is along x,
ˆ F0

(c)  If B is along y,
ˆ F  (L  R)

(d)  If B is along z,
ˆ F0
47. In the circuit shown below, the key is pressed at tme t = 0. (a) There is a repulsive force between the wire and the
Which of the following statement(s) is(are) true?(2016) loop
(b)  The  induced  current  in  the  wire  is  in  opposite
direction to the current along the hypotenuse
(c) If the loop is rotated at a constant angular speed

 0 
about the wire, an additional emf of     volt is
  
induced in the wire

 0 
(a) The voltmeter displays –5 as soon as the key is pressed, (d) The magnitude of induced emf in the wire is   
  
and displays +5 V after a long time
volt
(b) The voltmeter will display 0 V at time t = ln 2 seconds
(c) The current in the ammeter becomes 1/e of the initial Integer Type Questions
value after 1 second 50. In the circuit shown in figure E1 = 3 V, E2 = 2 V, E3 = 1 V and
(d) The current in the ammeter becomes zero after a long R = r1 = r2 = r3 = 1 . (JEE 1981)
time
r1
48. Consider two identical galvanometers and two identical + –
resistors with resistance R. If the internal resistance of E1
the  galvanometers  RC  <  R/2,  which  of  the  following R r2
A + – B
statement(s) about any one of the galvanometers is(are) E2
true? (2016) r3
+ –
(a) The maximum voltage range is obtained when all the E3
components are connected in series
(i) Find  the  potential  difference  between  the  points A
(b) The maximum voltage range is obtained when the
and B and the currents through each branch.
two  resistors  and  one galvanometer  are  connected  in
series,  and  the  second  galvanometer  is  connected  in (ii) If r2 is short circuited and the point A is connected to
parallel to the first galvanometer point B, find the currents through E1, E2, E3 and the
(c) The maximum current range is obtained when all the resistor R.
components are connected in parallel
CURRENT ELECTRICITY 86

51. Calculate the steady state current in the 2  resistor shown 56. An infinite ladder network of resistances is constructed


in the circuit (see figure). The internal resistance of the with 1  and 2  resistances, as shown in figure.
battery is negligible and the capacitance of the condenser The 6 V battery between A and B has negligible internal
C is 0.2 F. (JEE 1982) resistance : (JEE 1987)

6V
C
B

(i) Show that the effective resistance between A & B is 2 .
V=6 volt (ii)  What  is  the  current  that  passes  through  the  2  
52. A steady current passes through a cylindrical conductor. resistance nearest to the battery ?
Is there an electric field inside the conductor ?(JEE 1982) 57. In the given circuit  : (JEE 1988)
53. Two resistors, 400 , and 800  are connected in series E1 = 3E2 = 2E3 = 6 V R1 = 2R4 = 6 
with a 6 V battery. It is desired to measure the current in R3 = 2R2 = 4  C = 5 F.
the circuit. An ammeter of 10  resistance is used for this
Find the current in R3 and the energy stored in the capacitor.
purpose. What will be the reading in the ammeter ? Similarly,
if a voltmeter of 1000  resistance is used to measure the R1 E1
potential difference across the 400  resistor, what will be
the reading in the voltmeter ? (JEE 1982) C

54. In the circuit shown in figure E, F, G, H are cells of emf 2, 1,
E2 R2 R3
3 and 1 V respectively, and their internal resistances are 2,
1, 3 and 1  respectively. Calculate :
(i) the potential difference between B and D and E3 R4
(ii) the potential difference across the terminals of each 58. An  electrical  circuit  is  shown  in  figure.  Calculate  the
cells G and H. (JEE 1984) potential difference across the resistor of 400  as will be
A
E
B measured by the voltmeter V of resistance 400  either by
applying Kirchhoff’s rules or otherwise. (JEE 1996)
H
F V

D C
G

55. A part of circuit in a steady state along with the currents I2
flowing in the branches, the values of resistances etc., is I1
shown  in  the  figure.  Calculate  the  energy  stored  in  the
capacitor C (4 F). (JEE 1986) I 10V

1A 59. Find the  emf (V)  and  internal  resistance (r)  of a  single


battery which is equivalent to a parallel combination of
4V two batteries of emfs V1 and V2 and internal resistance r1
and r2 respectively, with polarities as shown in figure
2A
(JEE 1997)
C
3V
r2 V2
2A + –
A B
1A r1 V1
CURRENT ELECTRICITY 87
6 –3
60. A leaky parallel plane capacitor is filled completely with a 10   and 10  respectively. Clearly show the positions
material  having  dielectric  constant  k  =  5  and  electrical of the voltmeter and the ammeter. (JEE 2004)
–12 –1 –1
conductivity  = 7.4 × 10    m . If the charge on the 65. R1, R2, R3 are different values of R. A, B, C are the null
plane at instant t = 0 is q = 8.85 mC, then calculate the points  obtained  corresponding  to  R 1 ,  R 2   and  R 3
leakage current at the instant t = 12 s. (JEE 1997) respectively. For which resistor, the value of X will be the
61. In the circuit shown in figure, the battery is an ideal one, most accurate and why ? (JEE 2005)
with emf V. The capacitor is initially uncharged. The switch
X
S is closed at time t = 0. (JEE 1998) R
G
S A R
A B C

V R C

66. At t = 0, switch S is closed. The charge on the capacitor is
R B –t
varying with time as Q = Q0 (1 – e ). Obtain the value of
(a) Find the charge Q on the capacitor at time t. Q0 and  in the given circuit parameters. (JEE 2005)

(b) Find the current in AB at time t. What is its limiting
R1
value as t  ?
62. A thin uniform wire AB of length 1 m, an unknown resistance S
C R2
X  and  a  resistance  of  12    are  connected  by  thick
conducting strips, as shown in the figure. A battery and V
galvanometer (with a sliding jockey connected to it are
also available). Connections are to be made to measure 67. When  two  identical  batteries  of  internal  resistance  1
the  unknown  resistance  X  using  the  principle  of each are connected in series across a resistor R, the rate of
Wheatstone bridge. Answer the following questions. heat  produced  in  R  is  J1.  When  the  same  batteries  are
connected in parallel across R, the rate is J2. If J1 = 2.25 J2
(JEE 2002)
then the value of R in  is (2010)

68. At time t = 0, a battery of 10 V is connected across points
X
A  and  B  in  the  given circuit.  If  the  capacitors  have  no
charge initially, at what time (in second) does the voltage
A B C D across them become 4 V ?
(a) Are  there  positive  and  negative  terminals  on  the [Take : ln 5 = 1.6, ln 3 = 1.1] (2010)
galvanometer ?
(b) Copy the  figure in your answer book and  show the
battery and the galvanometer (with jockey) connected
at appropriate points.
(c) After appropriate connections are  made, it is found
that no deflection takes place in the galvanometer when 69. Two  batteries  of  different  emfs  and  different  internal
the sliding jockey touches the wire at a distance of 60 resistances are connected as shown. The voltage across
cm from A. Obtain the value of the resistance X. AB in volt is. (2011)
63. Show  by  diagram,  how  can  we  use  a  rheostat  as  the
potential divider ? (JEE 2003)
64. Draw the circuit for experimental verification of Ohm’s law
using a source of variable DC voltage, a main resistance
of 100 , two galvanometers and two resistances of values
CURRENT ELECTRICITY 88

70. A galvanometer gives full scale deflection with 0.006 A 73. A moving coil galvanometer has 50 turns and each turn


current. By connecting it to a 4990  resistance, it can be has an area 2×10–4 m2. The magnetic field produced by the
converted into a voltmeter of range 0-30 V. If connected to magnet inside the galvanometer is 0.02T. The torsional
constant of the suspension wire is 10–4 N m rad–1. When a
2n current  flows  through  the  galvanometer,  a  full  scale
a   resistance, it becomes an ammeter of range 0-1.5
249 deflection  occurs  if  the  coil  rotates  by  0.2  rad.  The
A. The value of n is (2014) resistance of the coil of the galvanometer is 50. This
71. Two parallel wires in the plane of the paper are distance X0 a galvanomter is to be converted into an ammeter capable
part. A  point  charge  is  moving  with  speed  u between  the of  measuring  current  in  the  range  0–1.0 A.  For  this
wires in the same plane at a distance X1 from one of the wires. purpose, a shunt resistance is to be added in parallel to
When  the wires carry current of  magnitude I  in the same the galvanometer. The value of this shunt resistance, in
direction,  the radius  of  curvature of the path of the  point ohms, is.................. (2018)
charge is R1. In contrast, if the currents I in the two wires have
directions opposite to each other the radius of curvature of
x0 R
the path is R2. If   3 , the value of  1 is (2014)
x1 R2

72. In the following circuit, the current through the resistor R
(=2) is I amperes. The value of I is (2015)


CURRENT ELECTRICITY 89

ANSWER KEY
EXERCISE - 1 : BASIC OBJECTIVE QUESTIONS

1. (c) 2. (c) 3. (b) 4. (a) 5. (c) 6. (d) 7. (a) 8. (d)


9. (b) 10. (b) 11. (d) 12. (c) 13. (d) 14. (a) 15. (b) 16. (b)
17. (c) 18. (d) 19. (d) 20. (d) 21. (d) 22. (b) 23. (a) 24. (c)
25. (b) 26. (b) 27. (c) 28. (a) 29. (b) 30. (d) 31. (b) 32. (d)
33. (d) 34. (d) 35. (b) 36. (b) 37. (c) 38. (c) 39. (a) 40. (c)
41. (c) 42. (d) 43. (a) 44. (c) 45. (b) 46. (b) 47. (d) 48. (b)
49. (c) 50. (a) 51. (a) 52. (a) 53. (c) 54. (d) 55. (c) 56. (a)
57. (d) 58. (b) 59. (a) 60. (a) 61. (a) 62. (d) 63. (a) 64. (d)
65. (b) 66. (b) 67. (a) 68. (d) 69. (d) 70. (b) 71. (d) 72. (a)
73. (d) 74. (d) 75. (a) 76. (d) 77. (d) 78. (c) 79. (b) 80. (a)
81. (d) 82. (a) 83. (c) 84. (c) 85. (a) 86. (b) 87. (d) 88. (b)
89. (b) 90. (c) 91. (a) 92. (b) 93. (a) 94. (d) 95. (b) 96. (a)
97. (b) 98. (b) 99. (d) 100. (c) 101. (a) 102. (b) 103. (b) 104. (b)
105. (c) 106. (a) 107. (d) 108. (c) 109. (b) 110. (c) 111. (b)

EXERCISE - 2 - PREVIOUS YEAR JEE MAIN QUESTIONS

1. (a) 2. (a) 3. (c) 4. (d) 5. (d) 6. (b) 7. (b) 8. (d)


9. (a) 10. (d) 11. (a) 12. (c) 13. (b) 14. (a) 15. (b) 16. (a)
17. (b) 18. (c) 19. (c) 20. (c) 21. (a) 22. (a) 23. (c) 24. (d)
25. (d) 26. (d) 27. (a) 28. (d) 29. (a) 30. (b) 31. (d) 32. (b)
33. (b) 34. (a) 35. (a) 36. (b) 37. (d) 38. (d) 39. (c) 40. (a)
41. (b) 42. (d) 43. (a) 44. (b) 45. (d) 46. (d) 47. (b) 48. (c)
49. (c) 50. (d) 51. (c) 52. (a) 53. (b) 54. (d) 55. (b) 56. (b)
57. (b) 58. (a) 59. (c) 60. (d) 61. (a) 62. (c) 63. (a) 64. (a)
65. (c) 66. (d) 67. (b) 68. (d) 69. (c) 70. (b) 71. (b)

EXERCISE - 3 - ADVANCED OBJECTIVE QUESTION

1. (a) 2. (d) 3. (a) 4. (a) 5. (d) 6. (d) 7. (b, d) 8. (d)


9. (b) 10. (a) 11. (c) 12. (a) 13. (a) 14. (d) 15. (a) 16. (d)
17. (b) 18. (a) 19. (a) 20. (c) 21. (c) 22. (c) 23. (a) 24. (c)
25. (a) 26. (b) 27. (a) 28. (b) 29. (c) 30. (d) 31. (a) 32. (a)
33. (b) 34. (a) 35. (a) 36. (b) 37. (a) 38. (a) 39. (b) 40. (d)
41. (b) 42. (a) 43. (c) 44. (b) 45. (b) 46. (b) 47. (b) 48. (a)
49. (a) 50. (a) 51. (d) 52. (d) 53. (d) 54. (b) 55. (b) 56. (d)
57. (c) 58. (b) 59. (a) 60. (b) 61. (b) 62. (d) 63. (d) 64. (b)
CURRENT ELECTRICITY 90

65. (d) 66. (c) 67. (b) 68. (d) 69. (d) 70. (b) 71. (d) 72. (a)
73. (d) 74. (a, c, d) 75. (b, d) 76. (a, c) 77. (a, b, c) 78. (a, b, c) 79. (a, c) 80. (b, d)
81. (a, d) 82. (a, c) 83. (a, c) 84. (a, c, d) 85. (a, b, c, d) 86. (a, b, c, d) 87. (a,c) 88. (a, d)
89. (a,b,d) 90. (a,b,c) 91. (b, c, d) 92. (a) 93. (a) 94. (c) 95. (c) 96. (a)
97. (a) 98. (a) 99. (c) 100. (c) 101. (b) 102. (c) 103. (c) 104. (b)
105. (d) 106. (a) 107. (d) 108. (b) 109. (c) 110. (d) 111. (c) 112. (a)
113. (b) 114. (b) 115. (c) 116. (b) 117. (c) 118. (b) 119. (c) 120. (c)
121. (d) 122. (a) 123. (a) 124. (b) 125. (d) 126. (c) 127. (a)
128. (a–q, r; b–q, r, s; c – p; d – q, r, s) 129. (a–s; b – t; c – p; d – r) 130. (a – p, q, r; b – p, q; c – s; d – s)
131. (a–q; b–r; c–s; d–s) 132. 0004 133. 0001 134. 0006 135. 0001 136. 0048
137. 0002 138. 0005

EXERCISE - 4 - PREVIOUS YEAR JEE ADVANCED QUESTIONS


1. 20  2. R/2 3. Zero 4. F 5. T 6. (b) 7. (c) 8. (d)
9. (c) 10. (b) 11. (d) 12. (d) 13. (a) 14. (c) 15. (b) 16. (c)
17. (a) 18. (d) 19. (a) 20. (a) 21. (a) 22. (b) 23. (a) 24. (d)
25. (c) 26. (a) 27. (a) 28. (b) 29. (c) 30. (a) 31. (c) 32. (c)
33. (d) 34. (c) 35. (b) 36. (c) 37. (b) 38. (c) 39. (d) 40. (b, d)
41. (b, c) 42. (a, b, d) 43. (a, d) 44. (a,b,c,d) 45. (a, b, d) 46. (a, b, c) 47. (a,b,c,d) 48 (b,c)
49. (a,d) 50. (i) 2 V, 1 A, 0, – 1A ; (ii) 1 A, 2 A, – 1A, 2 A 51. 0.9 A 52. Yes 53. 4.96 mA, 1.58 V
2 21 19 –5
54. (i)  V  ; (ii)  V, V 55. 0.288 mJ 56. (ii) 1.5 A 57. 1.5 A from right to left, 1.44 × 10  J
13 13 13

20 V1r2  V2 r1 rr CV V V 2t / 3RC V


58.
3
V 59. V 
r1  r2
, r 12
r1  r2
60. 0.198 A 61. (a)  Q 
2
 
1  e 2t / 3RC  ; (b)  i 2 
2R

6R
e ,
2R
CVR 2 R  R2
62. (a) No ; (c) 8  65. B is most accurate answer. 66. Q0  ,  1
R1  R 2 CR1R 2
67. 0004 68. 0002 69. 0005 70. (0005) 71. (0003) 72. (0001) 73. 5.55

Dream on !!


You might also like